Тепер Ви можете повідомити нас про будь-яку помилку в тесті, натиснувши кнопку
Гарячі клавішіВідкрити/закрити список тестів: SpaceГортати тести: ← →Перекласти тест: Shift
Понято
Акушерство і гінекологія
1/1500
Дівчинка 13 років надійшла до гінекологічного відділення зі скаргами на значні кров'янисті виділення зі статевих шляхів протягом 10 днів. З анамнезу відомо: менархе з 11 років, менструальний цикл нерегулярний. Під час ректо-абдомінального обстеження патології не виявлено. Який попередній діагноз?A 13-year-old girl came to the gynecological department with complaints of significant bleeding from the genital tract for 10 days. It is known from the anamnesis: menarche since the age of 11, menstrual cycle irregular. During the recto-abdominal examination, no pathology was detected. What was the previous diagnosis?
АденоміозAdenomyosis
Травма зовнішніх статевих шляхівInjury of the external genital tract
35-річний чоловік звернувся до лікаря зі скаргами на сильний біль у правому оці, світлобоязнь, сльозотечу, погіршення зору. Скарги з'явились після роботи зі зварювальним апаратом. Контактні лінзи не використовує. Під час огляду: очна щілина звужена, помірна перикорнеальна ін'єкція, гострота зору збережена. Після інстиляції розчину анестетика, стан полегшився, що дало можливість провести огляд у щілинній лампі та виявити дефект рогівки розміром 3 мм, що зафарбувався флюоресцеїном. Яке лікування буде найбільш доречним?A 35-year-old man consulted a doctor with complaints of severe pain in the right eye, photophobia, lacrimation, impaired vision. The complaints appeared after working with a welding machine. He does not use contact lenses. During the examination: the eye slit is narrowed, the pericorneal injection is moderate, the visual acuity is preserved. After instillation of the anesthetic solution, the condition was relieved, which made it possible to conduct a slit-lamp examination and reveal a 3 mm corneal defect stained with fluorescein. . What treatment would be most appropriate?
У хворого 60-ти років скарги на задуху, парестезії, запаморочення, втомлюваність. В анамнезі: атрофічний гастрит. Об'єктивно: шкіра та слизові бліді, іктеричні. Сосочки язика згладжені. Печінка, селезінка не збільшені. ЗАК: Нb- 77 г/л, ер.- 1,65 Г/л, КП- 1,4, ретикулоцити - 0,2%, Л- 2,8 Г/л; ШОЕ- 22 мм/год, макроцити. Яке додаткове обстеження дозволить вия-вити причину анемії?A 60-year-old patient complains of suffocation, paresthesias, dizziness, fatigue. History: atrophic gastritis. Objectively: skin and mucous membranes are pale, icteric. The papillae of the tongue are smoothed. ; ESR - 22 mm/h, macrocytes. What additional examination will reveal the cause of anemia?
Визначення рівня вітаміну B_12Determining the level of vitamin B_12
Визначення рівня залізаIron level determination
Визначення рівня кальціюCalcium level determination
Визначення рівня мідіDetermining copper level
Визначення рівня вітаміну DDetermination of vitamin D level
Акушерство і гінекологія
4/1500
Пацієнтка віком 25 років скаржиться на значні пінисті виділення із піхви, з неприємним запахом, печіння та свербіж в ділянці геніталій. З анамнезу відомо: хворіє протягом тижня, не має постійного статевого партнера. Під час піхвового дослідження виявлено: слизова оболонка піхви гіперемійована, під час дотику кровоточить, виділення піністого характеру. Який найімовірніший діагноз?A 25-year-old patient complains of significant foamy discharge from the vagina, with an unpleasant odor, burning and itching in the genital area. It is known from the anamnesis: she has been ill for a week, she has no a permanent sexual partner. During the vaginal examination, the vaginal mucosa is hyperemic, bleeding when touched. What is the most likely diagnosis?
Трихомонадний кольпітTrichomonad colpitis
Бактеріальний вагінозBacterial vaginosis
Хламідійний цервіцитChlamydial cervicitis
Гонорейний цервіцитGonorrheal cervicitis
Кандидомікоз піхвиVaginal candidiasis
Хірургія
5/1500
Пацієнт 15 років відстає у фізичному розвитку, періодично відзначається пожовтіння шкіри. Об'єктивно спостерігається: селезінка 16х12х10 см, холецистолітіаз, виразка шкіри нижньої третини лівої гомілки. У крові виявлено: еритроцити - 3,0·10^12/л, Hb - 90 г/л, КП - 1,0, мікросфероцитоз, ретикулоцитоз. Білірубін сироватки загальний - 56 мкмоль/л, непрямий - 38 мкмоль/л. Який метод лікування найдоцільніший?The patient is 15 years behind in physical development, yellowing of the skin is periodically noted. Objectively observed: spleen 16x12x10 cm, cholecystolithiasis, skin ulcer of the lower third of the left leg. In the blood revealed: erythrocytes - 3.0·10^12/l, Hb - 90 g/l, microspherocytosis, reticulocytosis. Serum bilirubin - 56 μmol/l, indirect - 38 μmol/l. What is the treatment method the most appropriate?
Портокавальний анастомозPortocaval anastomosis
Пересадження селезінкиSpleen transplant
ОментогепатопексіяOmentohepatopexy
СпленектоміяSplenectomy
ОментоспленопексіяOmentosplenopexy
Терапія
6/1500
Хвора 47-ми років, яка 2 дні тому повернулась з Перу, скаржиться на біль та збільшення лімфатичних вузлів у правій пахвинній ділянці. Діагностована бубонна форма чуми. Який з препаратів необхідно призначити контактним особам для екстреної профілактики захворювання?A 47-year-old patient who returned from Peru 2 days ago complains of pain and enlarged lymph nodes in the right inguinal region. The bubonic form of the plague is diagnosed. Which of the drugs should be prescribed to contact persons for emergency prevention of the disease?
Імуноглобулін людиниHuman immunoglobulin
Гетерологічна сироваткаHeterologous serum
ХлорохінChloroquine
ДоксициклінDoxycycline
ФлуконазолFluconazole
Гігієна
7/1500
Сім'я звернулася за прогнозом потомства у зв'язку з народженням дитини з тяжким дефектом закриття невральної трубки. Дитина померла у неонатальному періоді. Ваша порада родині:The family applied for the prognosis of the offspring due to the birth of a child with a severe neural tube closure defect. The child died in the neonatal period. Your advice to the family:
Провести преконцепційну профілактику, запланувати вагітність, провести пренатальну діагностикуConduct preconception prevention, plan pregnancy, conduct prenatal diagnosis
Провести цитогенетичне обстеження батьківConduct cytogenetic examination of parents
Відмовитися від народження дітейRefuse to have children
У разі вагітності провести біопсію хоріону, кордоцентезIn case of pregnancy, conduct a chorionic biopsy, cordocentesis
Планувати вагітність, оскільки це більше не повторитьсяPlan pregnancy because it won't happen again
Терапія
8/1500
Пацієнт віком 27 років скаржиться на болі в симетричних суглобах рук, лихоманку, ранкову скутість. Захворів гостро. Лабораторно встановлено: ревматоїдний фактор, високий титр С-реактивного білка, високий титр антитіл до цитрулінованого віментину. Рентгенологічно виявлено одиничні ерозії міжфалангових суглобів. Яке початкове базисне лікування необхідно призначити у цьому разі?A 27-year-old patient complains of pain in the symmetrical joints of the hands, fever, morning stiffness. He became acutely ill. Laboratory tests revealed: rheumatoid factor, high titer of C-reactive protein, high titer of citrullinated vimentin. Single erosions of the interphalangeal joints were detected. What initial treatment should be prescribed in this case?
У хворого 38-ми років скарги на переймоподібний біль в животі, часті рідкі випорожнення з домішками слизу та свіжої крові. Хворіє 2 роки. Схуд на 12 кг. Об'єктивно: Ps - 92/хв, АТ - 100/70 мм рт.ст., температура тіла - 37,4^oC. Живіт м'який, болючий за ходом товстого кишечника. Сигмоподібна кишка спазмована. В крові: еритроцити - 3,2·10^12/л, Нb - 92 г/л, лейкоцити - 10,6·10^9/л, ШОЕ - 32 мм/год. Іригоскопія: товста кишка звужена, гаустри відсутні, контури нечіткі, симптом ''водопровідної труби''. Який найбільш імовірний діагноз?A 38-year-old patient complains of spasm-like abdominal pain, frequent loose stools with impurities of mucus and fresh blood. He has been ill for 2 years. He has lost 12 kg. About Objectively: BP - 100/70 mmHg, body temperature - 37.4 °C. Abdomen is painful, colon is spasmodic. Blood: erythrocytes ,2·10^12/l, Hb - 92 g/l, leukocytes - 10.6·10^9/l, ESR - 32 mm/h. Irigoscopy: colon is narrowed, haustra are absent, contours are unclear, symptom water pipe''. What is the most likely diagnosis?
Амебна дизентеріяAmoebic dysentery
Хвороба КронаCrohn's disease
Рак кишечникаIntestinal cancer
Дивертикульоз кишечникаIntestinal diverticulosis
Виразковий колітUlcerative colitis
Акушерство і гінекологія
10/1500
Вагітна жінка віком 19 років перебуває у відділенні патології вагітності з приводу вагітності в терміні 36 тижнів, тазове передлежання, великий плід, важка форма цукрового діабету. На КТГ у плода зареєстрована брадикардія до 90/хв. Пологової діяльності немає. Яка тактика ведення вагітності в цій ситуації?A 19-year-old pregnant woman is in the department of pregnancy pathology due to a 36-week pregnancy, breech presentation, large fetus, severe form of diabetes mellitus. CTG of the fetus registered bradycardia up to 90/min. There is no labor activity. What are the tactics of pregnancy management in this situation?
Терміновий кесарів розтинUrgent cesarean section
РодостимуляціяRodostimulation
Виведення плода за тазовий кінецьExpulsion of the fetus by the pelvic end
Продовження вагітностіContinuation of pregnancy
Розродження через природні пологові шляхиBirth through natural birth canals
Педіатрія
11/1500
Доношена дівчинка народилася від ІІ вагітності, яка перебігала з загрозою переривання на 25-27 тижнях, з хронічною фетоплацентарною недостатністю. Під час пологів було одноразове туге обвиття пуповини навколо шиї плода, утруднення виведення плечиків. Після народження дитина була оцінена за шкалою Апгар на першій хвилині: дихання відсутнє, ЧСС - 50/хв., тотальний ціаноз, атонія, арефлексія. Яка це оцінка за шкалою Апгар?A full-term girl was born from the II pregnancy, which ran with a threat of termination at 25-27 weeks, with chronic fetoplacental insufficiency. During childbirth, there was a single tight wrapping of the umbilical cord around the neck of the fetus, difficulty in withdrawing the shoulders. After birth, the child was assessed according to the Apgar scale: no breathing, heart rate - 50/min., total cyanosis, atony, areflexia. What is this assessment according to the Apgar scale?
1 бал1 point
0 балів0 points
3 бали3 points
4 бали4 points
2 бали2 points
Терапія
12/1500
Хлопчик віком 10 років скаржиться на задишку під час ходіння, підвищення температури тіла до 38^oС , біль і припухлість обох колінних суглобів. Два тижні тому переніс ангіну. Об'єктивно спостерігається: припухлість, гіперемія, обмеження рухів в колінних суглобах, розширення меж серця вліво, тахікардія, приглушення тонів серця, систолічний шум на верхівці. Який найімовірніший діагноз?A 10-year-old boy complains of shortness of breath while walking, an increase in body temperature up to 38°C, pain and swelling of both knee joints. Two weeks ago he suffered from angina. About objectively observed: swelling, hyperemia, limitation of movements in the knee joints, expansion of the heart borders to the left, tachycardia, muffled heart sounds, systolic murmur at the apex. What is the most likely diagnosis?
Для боротьби з тютюнопалінням Україна ратифікувала Рамкову конвенцію Всесвітньої організації охорони здоров'я з боротьби з тютюном, у зв'язку із чим було заборонено палити в громадських місцях. До якого виду профілактики належать зазначені заходи?In order to combat tobacco smoking, Ukraine ratified the World Health Organization's Framework Convention on Tobacco Control, in connection with which smoking in public places was prohibited. what type of prevention do these measures belong to?
Вторинна профілактикаSecondary prevention
Індивідуальна профілактикаIndividual prevention
Третинна профілактикаTertiary prevention
Первинна профілактикаPrimary prevention
Громадська профілактикаPublic prevention
Акушерство і гінекологія
14/1500
Здорова жінка 47-ми років прийшла до свого гінеколога на планове взяття мазку Папаніколау. Скарг не має. Викурює 20 сигарет на день протягом 30-ти років (індекс паління - 30 пачко/років). Рік тому самостійно зробила аналіз крові на визначення концентрації загального холестерину, ЛПНЩ, ЛПВЩ та тригліцеридів. Усі показники аналізу в межах норми. При фізикальному обстеженні температура - 37,1^oC, пульс - 76/хв., частота дихання - 13/хв., артеріальний тиск - 120/70 мм рт.ст. Аускультативних змін у легенях не виявлено. Лабораторні показники без відхилень. Який наступний крок є найбільш доречним?A healthy 47-year-old woman came to her gynecologist for a routine pap smear. She has no complaints. She has smoked 20 cigarettes a day for 30 years (smoking index - 30 pack/years) A year ago, she did a blood test to determine the concentration of total cholesterol, HDL and triglycerides. During the physical examination, the temperature was 37.1°C, the heart rate was 76/min. respiratory rate - 13/min, blood pressure - 120/70 mm Hg. No abnormalities in laboratory parameters. What is the most appropriate step?
Визначення загального холестерину та ЛПВЩ через 2 рокиDetermination of total cholesterol and HDL after 2 years
Колоноскопія через 1 рікColonoscopy in 1 year
Мамографія через 3 рокиMammography in 3 years
Низькодозова КТ легень через 3 рокиLow-dose lung CT after 3 years
Кольпоскопія після ПАП-тестуColposcopy after Pap test
Терапія
15/1500
За 2 місяці після операційного втручання з приводу парапроктиту жінка віком 53 роки звернулась до лікаря зі скаргами на перианальний дискомфорт, тенезми, слизисто-гнійні виділення. Під час ректального обстеження відмічається зовнішній норицевий отвір, при надавлюванні на який виходить гній, по ходу отвору відмічається шнуроподібне ущільнення в напрямку прямої кишки. Під час анаскопії встановлено: слизова облонка кишківника по типу бруківки. Який імовірний діагноз у пацієнтки?2 months after surgery for paraproctitis, a 53-year-old woman consulted a doctor with complaints of perianal discomfort, tenesmus, mucopurulent discharge. During a rectal examination an external fistula opening is noted, on which pus comes out, a cord-like compaction is noted in the direction of the rectum. During the anascopy, the mucous membrane of the intestine of the cobblestone type is found. What is the probable diagnosis of the patient?
У жінки віком 25 років після пологів посилилися хиткість під час ходи та слабкість в ногах. Хворіє протягом 6 років, зазначає погіршення стану щороку восени. Об'єктивно спостерігається: жінка ейфорична, знижена критичність до свого стану, горизонтальний ністагм, високі сухожилкові рефлекси, клонус стоп, патологічні стопні рефлекси, черевні рефлекси відсутні, атаксія при пробі Ромберга, інтенційний тремор та промахування під час виконання координаторних проб. На очному дні спостерігається темпоральне збліднення дисків зорових нервів. Який діагноз найімовірніший?A 25-year-old woman after giving birth has increased unsteadiness while walking and weakness in her legs. She has been sick for 6 years, notes that her condition worsens every fall. Objectively observed: the woman is euphoric, reduced criticality of her condition, horizontal nystagmus, high tendon reflexes, foot clonus, pathological foot reflexes, absent abdominal reflexes, ataxia during Romberg's test, intention tremor and lapses during coordination tests nerves. What is the most likely diagnosis?
Пацієнт віком 50 років скаржиться на задишку, що посилюється під час фізичного навантаження. З анамнезу відомо що палить протягом 30 років. Об'єктивно спостерігається: температура тіла 36,5^oС, ЧД - 22/хв, ЧСС - 88/хв, АТ - 130/85 мм рт. ст. Грудна клітка бочкоподібної форми, під час аускультації вислуховується послаблене везикулярне дихання з коробковим відтінком над усією поверхнею легень. Який найімовірніший діагноз?A 50-year-old patient complains of shortness of breath, which worsens during physical exertion. It is known from the anamnesis that he has been smoking for 30 years. Objectively observed: body temperature 36, 5^oС, heart rate - 88/min, barrel-shaped chest, during auscultation weak vesicular breathing is heard over the entire surface of the lungs. What is the most likely diagnosis?
Хронічне обструктивне захворювання легеньChronic obstructive pulmonary disease
Терапія
18/1500
Чоловік 58-ми років, скаржиться на постійний біль в епігастрії, що іррадіює в спину і посилюється після прийому жирної їжі. Пацієнт відмічає слабкість, за останні 2 місяці втратив у вазі до 9 кг. Місяць тому з'явилась жовтяниця, що поступово зростає. Живіт м'який, печінка не збільшена. При лабораторному дослідженні крові гемоглобін - 68 г/л, лейкоцити - 9,0·10^9/л, швидкість зсідання еритроцитів - 36 мм/год. На УЗД: жовчний міхур великих розмірів, конкрементів не містить, холедох розширений, в головці підшлункової залози - округле утворення до 4 см в діаметрі з нечіткими контурами. Який діагноз є найбільш імовірним?A 58-year-old man complains of constant pain in the epigastrium, which radiates to the back and worsens after eating fatty food. The patient notes weakness, over the past 2 months he has lost in weight up to 9 kg. Jaundice appeared a month ago, the abdomen is soft, the liver is not enlarged. In the laboratory examination of blood - 68 g/l, leukocytes - 9.0·10^9/l sedimentation of erythrocytes - 36 mm/h. On ultrasound: the gallbladder is large, does not contain calculi, the choledochus is enlarged, in the head of the pancreas - a rounded formation with a diameter of 4 cm. What is the most likely diagnosis?
Рак шлунка з метастазами в печінкуGastric cancer with liver metastases
Терапія
19/1500
У пацієнта віком 16 років уперше раптово виникли біль у потиличній ділянці, миготіння <<мушок>>, одноразове блювання, що не принесло полегшення. Об'єктивно виявлено: пацієнт у свідомості, гіперемія шкіри обличчя, АТ - 160/110 мм рт. ст., пульс - 122/хв, напружений. Сухожилкові рефлекси рівномірні. Стенокардія напруги. Акцент ІІ тону над аортою. Який препарат потрібно насапмеред призначити для лікування цього пацієнта?A 16-year-old patient first developed sudden pain in the back of the head, flickering <>, single vomiting, which did not bring relief. Objectively revealed: the patient is conscious, the blood pressure is 160/110 mm Hg, the tendon reflexes are tense. What drug should be prescribed for the treatment of this patient?
1%-й розчин дибазолу 0,1 мл/рік життя в ізотонічному розчині натрію хлориду в/в1% solution of dibazol 0.1 ml/year of life in isotonic solution of sodium chloride IV
Пропранолол 2 мг в/в крапельно в ізотонічному розчині натрію хлоридуPropranolol 2 mg IV drip in isotonic sodium chloride solution
2%-й розчин еуфіліну 3 мг/кг у 100 мл ізотонічного розчину натрію хлориду в/в2% solution of euphilin 3 mg/kg in 100 ml of isotonic sodium chloride solution IV
Гігієна
20/1500
Робітниця 40 років працює на фабриці з виробництва льону, стаж 12 років. Пред'являє скарги на задишку, кашель, слабкість. Загальний стан поліпшується у вихідні дні, але знову стає гіршим у разі виходу на роботу. У легенях вислуховуються розсіяні сухі хрипи з обох сторін. У крові спостерігається незначний лейкоцитоз. Який найвірогіднішиий діагноз?A 40-year-old female worker works at a flax factory, 12 years of experience. She complains of shortness of breath, cough, weakness. Her general condition improves on weekends, but becomes worse when going to work. Scattered wheezes are heard on both sides. What is the most likely diagnosis?
Пильовий бронхітDust bronchitis
БісинозBisinosis
СилікозSilicosis
Алергійний альвеолітAllergic alveolitis
Бронхіальна астмаBronchial asthma
Терапія
21/1500
Чоловік 69 років протягом декількох останніх років спостерігав прогресуюче порушення сечовипускання. Протягом останньої доби зазначив відсутність сечовипускання, постійний розпираючий біль над лоном. Які діагностичні заходи слід запропонувати у цьому разі?A 69-year-old man has had a progressive urinary incontinence for the past several years. During the last day, he noted the absence of urination, constant throbbing pain above the pubic area. What diagnostic measures should be offered in this case ?
Чоловік 62 років скаржиться на помірний біль у лівій ступні в ділянці плюснефалангового суглоба, який посилюється під час рухів. Хворіє протягом 12 років, захворювання почалося з <<гострого нападу>> болю. 2 роки тому під шкірою суглоба з'явився жовтувато-білий вузлик. Під час огляду виявлено суглоб деформований, синюшний. На рентгенограмі суглоба спостерігаються звуження суглобової щілини та чітко обмежені дефекти кісткової тканини в епіфізі (<<симптом пробійника>>). Який діагноз найімовірніший?A 62-year-old man complains of moderate pain in the left foot in the area of the metatarsophalangeal joint, which worsens during movements. He has been ill for 12 years, the disease began with an <> pain. A yellowish-white nodule appeared under the skin of the joint. During the examination, the joint was found to be bluish. Narrowing of the joint space and clearly limited bone tissue defects in the epiphysis were observed (<>). . What is the most likely diagnosis?
Подагричний артритGouty arthritis
Ревматоїдний артритRheumatoid arthritis
ОстеоартритOsteoarthritis
Синдром РейтераReiter syndrome
Реактивний артритReactive arthritis
Терапія
23/1500
До лікаря звернувся чоловік зі скаргами на біль у нижній щелепі. Об'єктивно спостерігається: маса тіла знижена, шкірні покриви бліді, неприємний запах з рота, ясна кровоточать. На рентгенограмі щелепи виявлено явища остеопорозу. Пацієнт протягом 30 років працював на хімічному підприємстві. Для якого патологічного стану характерні ці симптоми?A man went to the doctor with complaints of pain in the lower jaw. Objectively observed: body weight is reduced, skin is pale, bad breath, gums are bleeding. An x-ray of the jaw revealed osteoporosis. The patient worked at a chemical plant for 30 years. What pathological condition are these symptoms characteristic of?
Отруєння перхлоратом каліюPoisoning with potassium perchlorate
Отруєння метилртуттюMethylmercury poisoning
Хронічного остеомієліту нижньої щелепиChronic osteomyelitis of the lower jaw
ФлюорозуFluorosis
Терапія
24/1500
Хлопець 18 років перебуває на лікуванні в терапевтичному відділенні з приводу загострення хронічного бронхіту. З 6 років страждає епілепсією. На фоні підвищення температури до фебрильних цифр розвинувся епілептичний статус у вигляді тоніко-клонічних судом. Яка Ваша найперша тактика у цьому разі?An 18-year-old boy is being treated in the therapeutic department for an exacerbation of chronic bronchitis. He has been suffering from epilepsy since he was 6 years old. Against the background of an increase in temperature to febrile figures, he developed a status epilepticus in the form of tonic-clonic seizures. What is your first tactic in this case?
Переведення до реанімаційного відділенняTransfer to intensive care unit
Переведення до неврологічного відділенняTransfer to the neurology department
Організація консультації психіатраOrganization of psychiatrist consultation
Переведення до психіатричного стаціонаруTransfer to a psychiatric hospital
Надання невідкладної допомоги на місціProviding emergency care on site
Акушерство і гінекологія
25/1500
У вагітної жінки, що страждає на гіпертонічну хворобу I ступеня, в терміні 35 тижнів з'явилися набряки на нижніх кінцівках і передній черевній стінці, в добовій сечі кількість білка - до 5 г/л, АТ підвищився до 170/120 мм рт.ст., став турбувати головний біль і погіршився зір. Проведене впродовж чотирьох годин інтенсивне лікування не дало ефекту. Яка тактика необхідна в цій ситуації?A pregnant woman suffering from hypertension of the first degree, in the period of 35 weeks, edema appeared on the lower limbs and the anterior abdominal wall, the amount of protein in the daily urine - up to 5 g/l, blood pressure rose to 170/120 mm Hg, headache and vision worsened. Intensive treatment for four hours did not give any effect. What tactics are necessary in this situation?
Негайне розродження шляхом кесаревого розтинуImmediate cesarean delivery
Проведення родозбудженняConducting genealogy
Консервативне розродженняConservative birth
Підготовка шийки матки для дострокового розродженняPreparation of the cervix for premature birth
Продовження інтенсивної терапіїContinuation of intensive care
Терапія
26/1500
Під час обстеження пацієнта виявлено такі симптоми: прогресуюча кволість, зниження працездатності, швидка фізична та психічна втомлюваність, відчуття мерзлякуватості та голоду, втрата маси тіла. Установіть вид аліментарного захворювання:During the examination of the patient, the following symptoms were revealed: progressive frailty, reduced work capacity, rapid physical and mental fatigue, feeling of coldness and hunger, loss of body weight. Establish the type of alimentary disease :
Недостатність поліненасичених жирних кислотDeficiency of polyunsaturated fatty acids
Акушерство і гінекологія
27/1500
В гінекологічне відділення онкодиспансеру доставлено хвору 35-ти років зі скаргами на кров'янисті виділення після статевого акту. При дослідженні в дзеркалах: на шийці матки навколо зовнішнього вічка дефект епітелію. Матка, додатки і параметрії без патологічних змін. При гістологічному дослідженні біоптату шийки матки виявлено рак in situ. Яке лікування буде найбільш доречним?A 35-year-old patient was brought to the gynecological department of the oncology clinic with complaints of bleeding after sexual intercourse. When examined in mirrors: a defect is found on the cervix around the outer eye epithelium. Uterus, appendages without pathological changes. Cancer in situ was detected during histological examination of the cervix. What treatment would be most appropriate?
Променева терапіяRadiotherapy
Конусоподібна електроексцизія шийки маткиCone-shaped electroexcision of the cervix
Кріодеструкція шийки маткиCryodestruction of the cervix
Розширена екстирпація матки з додаткамиAdvanced uterine extirpation with attachments
Хіміотерапевтичне лікуванняChemotherapy treatment
Хірургія
28/1500
Чоловік 35 років збитий автомобілем, ударився головою об асфальт. Об'єктивно встановлено: стан важкий, без свідомості, у комі, дихання уривчасте, шумне, на больові подразники не реагує. У ділянці лоба велика рана м'яких тканин голови, видно фрагменти кістки, витікає мозковий детрит, є кровотеча. Визначте лікувальну тактику на догоспітальному етапі:A 35-year-old man was hit by a car, hit his head on the asphalt. Objectively established: the condition is serious, unconscious, in a coma, breathing is intermittent, noisy, on painful stimuli does not react. There is a large wound of the soft tissues of the head, brain debris is leaking out. Determine the treatment tactics at the pre-hospital stage:
Видалення фрагментів кістокRemoving bone fragments
Чоловік 33-х років прийшов до лікаря на профілактичний прийом. Скарг не має. При фізикальному обстеженні лікарем випадково виявлено грубий систолічний шум в тoчці Боткіна, який проводиться в точку аускультації верхівки та на судини шиї. Від подальшого обстеження пацієнт відмовився. Через 2 місяці на фоні грипу у пацієнта розвинулися симптоми серцевої недостатності. Яку ваду найбільш імовірно було виявлено лікарем?A 33-year-old man came to the doctor for a preventive appointment. He has no complaints. During the physical examination, the doctor accidentally detected a rough systolic murmur at Botkin's point, which is performed at the point auscultation of the apex and the vessels of the neck. The patient refused further examination. After 2 months, the patient developed symptoms of heart failure.
Хвора 17-ти років має скарги на біль в колінних та гомілковостопних суглобах, підвищення температури тіла до 39^oC. 2 тижні тому перенесла гострий тонзиліт. Об'єктивно: припухлість та різка болючість, обмеження рухів в суглобах. На шкірі тулуба та кінцівок є ледве помітні кільцеподібні блідо-рожеві плями. ЧСС- 95/хв., АТ- 90/60 мм рт.ст., тони ослаблені, м'який систолічний шум над верхівкою. Поставте попередній діагноз:A 17-year-old patient complains of pain in the knee and ankle joints, an increase in body temperature to 39°C. She suffered acute tonsillitis 2 weeks ago. Objectively : swelling and sharp pain in the joints. On the skin of the trunk and limbs there are barely visible ring-shaped spots. HR- 95/min., BP- 90/60 mm Hg, tones are weakened, soft systolic make a preliminary diagnosis:
Ревматоїдний артритRheumatoid arthritis
Хвороба РейтераReiter's disease
Гостра ревматична лихоманкаAcute rheumatic fever
Системна склеродерміяSystemic scleroderma
Вузлувата еритемаErythema nodosum
Терапія
31/1500
Дівчинка 14-ти років звернулася до лікаря зі скаргами на відчуття стиснення у ділянці шиї, покашлювання, потовщення шиї. При фізикальному обстеженні щитоподібна залоза дифузно збільшена, щільна при пальпації, неболюча, поверхня гладка. При ультразвуковому дослідженні тканина щитоподібної залози неоднорідна. Яке лабораторне дослідження буде найбільш доречним?A 14-year-old girl went to the doctor with complaints of a feeling of compression in the neck area, coughing, thickening of the neck. On physical examination, the thyroid gland is diffusely enlarged, dense on palpation , painless, smooth surface. Thyroid tissue is heterogeneous during ultrasound examination. Which laboratory test would be most appropriate?
Титр антитіл до тиреоглобулінуTiter of antibodies to thyroglobulin
Вміст йоду в добовій сечіIodine content in daily urine
Вміст кальцитонінуCalcitonin content
Вміст паратгормонуParathormone content
Вміст гормону ростуContent of growth hormone
Терапія
32/1500
Пацієнтка віком 26 років протягом тижня почала виконувати багато нових і незвичних для неї справ зокрема: самотужки взялась фарбувати стіни в під'їзді, ночами писала вірші, водночас роблячи до них ілюстрації. Об'єктивно спостерігається: балакуча, швидко змінює теми розмови, активно жестикулює, жартує, кокетує з чоловіками. Вважає, що в ній загинула велика актриса, письменниця і художниця, запрошує всіх у гості на 'мистецькі вечори'. Який психопатологічний стан спостерігається у пацієнтки?A 26-year-old patient began doing many new and unusual things for her during the week, in particular: she started painting the walls in the entrance hall on her own, wrote poems at night, while doing They are objectively observed: she is chatty, she is actively gesticulating, she is flirting with men. She thinks that she is a great actress, writer and artist observed in the patient?
У ході проведення медичного огляду студентів, лікар звернув увагу на хлопця 24-х років з ростом 176 см та масою тіла 68 кг, у якого спостерігалась сухість шкіри та гіперкератоз. При офтальмологічному обстеженні виявлене суттєве зниження гостроти сутінкового зору. У харчуванні студента практично ніколи не були включені в раціон молочні продукти, овочі та фрукти. Яке із зазначених захворювань можна припустити?During the medical examination of students, the doctor noticed a 24-year-old boy with a height of 176 cm and a body weight of 68 kg, who had dry skin and hyperkeratosis During the ophthalmological examination, a significant decrease in twilight vision was detected. Dairy products, vegetables and fruits were practically never included in the diet of the student. Which of the mentioned diseases can be assumed?
МіопіяMyopia
A-гіповітамінозA-hypovitaminosis
C-гіповітамінозC-hypovitaminosis
АстигматизмAstigmatism
Діенцефальний синдромDiencephalic syndrome
Терапія
34/1500
Пацієнт віком 56 років звернувся до сімейного лікаря зі скаргами на загальну слабкість, підвищення температури тіла до 38,5^oС, кашель із виділенням гнійного мокротиння, незначну задишку. Хворіє протягом трьох днів. З першого дня клінічних проявів приймав амоксицилін-клавуланат, однак стан не покращився. З анамнезу відомо, що пацієнт не палить, протягом останніх шести місяців не приймав антибіотики, п’ять років хворіє на цукровий діабет 2 типу, приймає метформін. Об'єктивно спостерігається: АТ - 130/80 мм рт. ст., пульс - 85/хв. ЧД - 20/хв. Під час перкусії легень виявлено притуплення перкуторного звуку в нижніх відділах правої легені, аускультативно над цією ділянкою вислуховуються ослаблене везикулярне дихання та дрібнопухирчасті вологі хрипи. З боку інших органів та систем патології не виявлено. За результатами загального аналізу крові: нейтрофільний лейкоцитоз із зсувом формули вліво. Яка подальша тактика сімейного лікаря?A 56-year-old patient turned to a family doctor with complaints of general weakness, an increase in body temperature up to 38.5°C, a cough with purulent sputum, slight shortness of breath. He has been sick for three days. He has been taking amoxicillin-clavulanate since the first day of clinical symptoms, but his condition has not improved. It is known that the patient does not smoke, has not taken antibiotics for the past six months, and has been suffering from type 2 diabetes for five years. Objectively, blood pressure is 130/80 mmHg, heart rate is 20/min. breathing and small vesicular wheezes. No pathology was detected on the part of other organs and systems. According to the results of the general blood analysis: neutrophilic leukocytosis with a shift to the left. What is the next tactic of the family doctor?
Збільшити дозу амоксицилін-клавуланата та додати муколітичні засобиIncrease amoxicillin-clavulanate dose and add mucolytics
Призначити цефалоспорини І поколінняPrescribe 1st generation cephalosporins
Призначити фторхінолон ІІІ-IV поколінняPrescribe fluoroquinolone III-IV generation
Призначити доксициклінPrescribe doxycycline
Направити на консультацію до ендокринологаSend for a consultation with an endocrinologist
Терапія
35/1500
У пацієнта віком 29 років протягом трьох тижнів на шкірі червоної кайми нижньої губи спостерігається безболісна виразка, яка останній тиждень супроводжується появою значного набряку підлеглих та навколишніх тканин. Під час огляду виявлено: виразка діаметром до 2,5-3 см, чіткі краї, дно кольору «старого сала», хрящеподібний інфільтрат під нею. Яке лабораторне обстеження потрібно призначити в першу чергу?A 29-year-old patient has had a painless ulcer on the skin of the red border of the lower lip for three weeks, which in the last week has been accompanied by the appearance of significant swelling of the underlying and surrounding tissues. During the examination revealed: an ulcer with a diameter of up to 2.5-3 cm, clear edges, the bottom is the color of 'old lard', a cartilaginous infiltrate under it. What laboratory examination should be prescribed first?
Дослідження на еозинофілиResearch on eosinophils
Обстеження на акантолітичні клітиниExamination for acantholytic cells
Зіскоб на мікозиScraping for mycosis
Бактеріальний посівBacterial culture
Обстеження на бліду трепонемуExamination for pale treponema
Гігієна
36/1500
При проведенні поточного профілактичного медичного огляду робітника цеху з виробництва автомобільних акумуляторів встановлено: шкіра бліда з сірувато-землистим відтінком, видимі слизові бліді. На яснах, переважно біля передніх зубів, є зміна кольору слизової у бузковий колір у вигляді смуги. Є тремор пальців рук. При пальпації рук є біль по ходу нервів. Назвіть токсичну сполуку та основний шлях її надходження до організму людини в умовах виробництва:During the current preventive medical examination of a worker in the workshop for the production of automobile batteries, it was established: the skin is pale with a grayish-earthy hue, the mucous membranes are pale. On the gums, mainly near the front teeth , there is a change in the color of the mucous membrane in the form of a stripe. There is pain along the nerves when palpating the hands. Name the toxic compound and its main route of entry into the human body:
Нікель, перкутаннийNickel, percutaneous
Арсен, інгаляційнийArsenic, inhalation
Кадмій, перкутаннийCadmium, percutaneous
Свинець, інгаляційнийLead, inhalation
Ртуть, пероральнийMercury, oral
Хірургія
37/1500
Чоловік госпіталізований після тупої травми грудної клітки. АТ - 100/60 мм рт. ст., пульс - 98/хв. На рентгенограмі виявлено: ОГК без особливостей. Ехокардіографія встановила: вільна рідина в порожнині перикарду в кількості 100 мл. Через годину спостерігається зниження АТ до 85/50 мм рт. ст., набухання шийних вен, пульс - 106/хв., тони серця різко ослаблені. Які найперші заходи слід вжити?A man was hospitalized after a blunt chest injury. Blood pressure - 100/60 mm Hg, pulse - 98/min. X-ray showed: OGK without features. Echocardiography showed: free fluid in the pericardial cavity in the amount of 100 ml. A decrease in blood pressure to 85/50 mmHg is observed, the pulse is 106/min. What are the first measures to be taken?
В/в введення серцевих глікозидівIV administration of cardiac glycosides
Пункція перикардуPericard puncture
Інфузія симпатоміметиківInfusion of sympathomimetics
АнтибіотикотерапіяAntibiotic therapy
Ввести діуретики в/вEnter intravenous diuretics
Терапія
38/1500
Жінка 31 року протягом 3 років скаржиться на біль та підпухлість променево-зап'ясних та п'ясно-фалангових суглобів, ранкову скутість до 1,5 години. Два тижні тому з'явився біль, підпухлість та почервоніння колінних суглобів, підвищення температури тіла до 37,5^oС. Під час обстеження внутрішніх органів патологічних змін не виявлено. Поставлено діагноз: ревматоїдний артрит. Які зміни на рентгенограмах суглобів найвірогідніші?A 31-year-old woman has been complaining of pain and swelling of the radiocarpal and metatarsophalangeal joints for 3 years, morning stiffness up to 1.5 hours. Two pain, swelling and redness of the knee joints appeared weeks ago, an increase in body temperature up to 37.5 °C. No pathological changes were detected. A diagnosis was made: rheumatoid arthritis. What changes are most likely on the X-rays of the joints?
Остеоліз епіфізівOsteolysis of epiphyses
Звуження суглобової щілини, узуриNarrowing of the joint space, patterns
Кісти в субхондральній кістціCysts in the subchondral bone
Звуження суглобової щілини, субхондральний остеосклерозNarrowing of the joint space, subchondral osteosclerosis
Жінка 30-ти років скаржиться на загальну слабкість, утруднене ковтання їжі, сухість шкіри та ламкість волосся. Об'єктивно: температура тіла - 36,6^oC, ЧД- 16/хв., Ps- 92/хв., АТ- 110/70 мм рт.ст. Шкірні покриви та видимі слизові бліді. У крові: Hb- 65 г/л, еритроцити - 3,2·10^12/л, КП- 0,6, рет - 3%, лейкоцити - 6,7·10^9/л, е - 2%, п - 3%, с - 64%, л - 26%, м - 5%, ШОЕ- 17 мм/год. Сироваткове залізо - 7,4 мкмоль/л, загальний білок - 78 г/л. Дефіцит якого фактору зумовив виникнення захворювання?A 30-year-old woman complains of general weakness, difficulty swallowing food, dry skin and brittle hair. Objectively: body temperature - 36.6^oC, BH- 16/min., Ps- 92/min., BP- 110/70 mm Hg. Skin and mucous membranes are pale. Blood: Hb- 65 g/l, erythrocytes - 3.2·10^12 /l, CP - 0.6, ret - 3%, leukocytes - 6.7·10^9/l, e - 2%, p - 3%, c - 64%, l - 26%, m - 5% , ESR - 17 mm/h. Serum iron - 7.4 μmol/l, total protein - 78 g/l. What factor caused the disease?
У пацієнтки віком 63 роки протягом 5 тижнів прогресує безбольова жовтяниця, шкірний свербіж, схуднення на 10 кг, ахолія. Пальпується позитивний симптом Курвуазьє. Який найімовірніший діагноз?A 63-year-old female patient developed painless jaundice, skin itching, weight loss of 10 kg, acholia for 5 weeks. A positive Courvoisier sign is palpated. What is the most likely diagnosis?
МаляріяMalaria
Вірусний гепатитViral hepatitis
Рак печінкиLiver cancer
Рак підшлункової залозиPancreatic cancer
Рак жовчного міхураGall Bladder Cancer
Терапія
41/1500
Раніше здорова жінка 47-ми років звернулася до сімейного лікаря через підвищену температуру та кашель із жовтим мокротинням, що тривають вже 5 днів. Температура тіла - 37,4^oC , пульс - 82/хв., частота дихання - 15/хв., артеріальний тиск - 130/80 мм рт.ст. При аускультації вислуховуються двосторонні розсіяні сухі хрипи. На оглядовій рентгенограмі грудної клітки патологічних змін не виявлено. Яка тактика лікування буде найбільш доречною для цієї пацієнтки?A previously healthy 47-year-old woman consulted a family doctor because of a high temperature and a cough with yellow sputum that has been going on for 5 days. Body temperature is 37.4^ oC, pulse - 15/min, blood pressure - 130/80 mmHg. On auscultation, bilateral diffuse rales are heard. No pathological changes are detected on the chest X-ray most appropriate for this patient?
Аскорбінова кислота (вітамін C) перорально на 5 днівAscorbic acid (vitamin C) orally for 5 days
Вживання достатньої кількості рідиниDrinking enough fluids
Комбінований препарат із противірусним та антигістамінним компонентом перорально на 5 днівCombined drug with antiviral and antihistamine component orally for 5 days
Інтерферону альфа-2b у сиропі протягом тижняInterferon alfa-2b in syrup for a week
Ципрофлоксацин перорально на 7 днівCiprofloxacin orally for 7 days
Педіатрія
42/1500
У дитини 9-ти років з'явився біль у верхній третині правої гомілки, підвищилась температура тіла до 39^oC. Дитина не може стати на праву ногу. З анамнезу відомо, що дитина мала травму гомілки та перенесла ангіну. Яке захворювання можна припустити?A 9-year-old child developed pain in the upper third of the right shin, the body temperature rose to 39^oC. The child cannot stand on his right leg. With it is known in the anamnesis that the child had a leg injury and suffered angina. What disease can be assumed?
Перелом кісток правої гомілкиFracture of the bones of the right tibia
Туберкульозний артритTuberculous arthritis
Злоякісна пухлина кісткиMalignant bone tumor
Терапія
43/1500
Чоловік скаржиться на болючість, почервоніння шкіри на правій литці. Об'єктивно встановлено: температура тіла - 38,5^oС, збільшені і болючі лімфатичні пахвинні вузли з правого боку, шкіра червона, межі між почервонінням і здоровою шкірою чіткі, є набряк, пухирці з темною рідиною всередині, пальпація болюча. Який попередній діагноз?The man complains of soreness, redness of the skin on the right calf. Objectively established: body temperature - 38.5°C, enlarged and painful inguinal lymph nodes on the right side, the skin is red, the boundaries between redness and healthy skin are clear, there is swelling, blisters with dark liquid inside, palpation is painful. What is the previous diagnosis?
Вітряна віспаChicken Pox
Сибірка, шкіряна формаAnthrax, skin form
БешихаBeshikha
Герпетична інфекціяHerpes infection
Флегмона ногиPhlegmon of the leg
Терапія
44/1500
Чоловік 46-ти років звернувся до лікаря зі скаргами на появу висипань в ділянці кистей рук, що супроводжуються сильним свербежем і турбують протягом тижня. У анамнезі бронхіальна астма, сезонні алергії. Пацієнт відзначає, що має дуже суху шкіру. При фізикальному обстеженні на тильній стороні правої та лівої кисті виявлено множинні еритематозні папули та везикули з серозним вмістом, ділянки ліхеніфікації та екскоріації. Який діагноз є найбільш імовірним?A 46-year-old man turned to the doctor with complaints about the appearance of rashes in the area of the hands, accompanied by severe itching and bothering him for a week. He has a history of bronchial asthma, seasonal allergies. The patient notes that he has very dry skin. On physical examination, multiple erythematous papules and vesicles with serous contents, areas of lichenification and excoriation are found.
Себорейний дерматитSeborrheic dermatitis
КоростаScabies
ЕкземаEczema
ПсоріазPsoriasis
Контактний дерматитContact dermatitis
Хірургія
45/1500
Пацієнтка 37 років. Переведена до очного відділення із ЛОР відділення, у якому лікувалася з приводу гнійного пансинуситу. Об'єктивно встановлено: загальна слабість, температура тіла - 38,5^oC. Гострота зору правого ока - 0,6 (з анамнезу VIS OD=1,0). Повіки синюшні, набряклі, щільні. Хемоз кон'юнктиви. Екзофтальм. Нерухомість очного яблука. Репозиція ока затруднена. Поставте діагноз:The patient is 37 years old. She was transferred to the eye department from the ENT department, where she was treated for purulent pansinusitis. Objectively established: general weakness, body temperature - 38, 5^oC. Visual acuity - 0.6. Eyelids are swollen. Conjunctiva. Immobility of the eyeball. Make a diagnosis:
До сімейного лікаря звернувся па-цієнт з астено-вегетативним синдромом. Пацієнт пред'являв скарги на порушення cну, частий головний біль, запаморочення, дратівливість, підвищену втомлюваність, порушення пам'яті. Пацієнт протягом 2-х років не працював, проживає в житловому будинку, який розташований поруч з аеро-портом. З впливом якого чинника найімовірніше можливе виникнення даних симптомів?A patient with astheno-vegetative syndrome consulted a family doctor. The patient complained of cna disorders, frequent headache, dizziness, irritability, increased fatigue, memory impairment. The patient has not worked for 2 years, lives in a residential building located near the airport. Which factor is most likely to cause these symptoms?
Електромагнітні поляElectromagnetic fields
ВібраціяVibration
Хімічне забруднення атмосферного повітряChemical air pollution
УльтразвукUltrasound
ШумNoise
Педіатрія
47/1500
Батьки восьмирічного хлопчика звернулися до лікаря зі скаргами на відставання дитини у зрості. З анамнезу відомо: народжений доношеним, із вагою 3100 г, довжиною тіла 50 см. Батьки нормального зросту. На момент огляду хлопчика співвідношення зросту до віку = -3, вага - теж знаходиться у діапазоні < -2. <<Кістковий вік>> дорівнює 5 років. Патології з боку інших органів і систем не виявлено. Яке дослідження необхідно призначити дитині для уточнення діагнозу?The parents of an eight-year-old boy turned to the doctor with complaints about the child's growth retardation. It is known from the anamnesis: he was born full-term, with a weight of 3100 g, a body length of 50 cm. The parents of a normal At the time of the examination, the boy's height-to-age ratio is in the range of <-2. No pathology from other organs and systems has been found for the child clarifying the diagnosis?
Визначення рівня соматотропного гормонуDetermining the level of somatotropic hormone
Визначення рівня гонадотропного гормонуDetermining the level of gonadotropic hormone
Визначення рівня тиреотропного гормонуDetermining the level of thyroid-stimulating hormone
Визначення рівня адренокортикотропного гормонуDetermining the level of adrenocorticotropic hormone
Визначення рівня лютеїнізуючого гормонуDetermining the level of luteinizing hormone
Педіатрія
48/1500
Дитина народилась із масою 3800 г від перших фізіологічних пологів. На другий день життя у неї з’явилася везикульозна висипка на шкірі та волосяній частині голови, підвищилася температура тіла до 37,5^oС. Встановлено діагноз: вітряна віспа. До якого віку дитини вітряна віспа вважається вродженою?The child was born with a weight of 3800 g from the first physiological delivery. On the second day of life, she developed a vesicular rash on the skin and scalp, the body temperature rose to 37.5°C. Diagnosed: chicken pox. Until what age is chicken pox considered congenital?
До 10 дняUp to 10 days
До 30 дняUp to 30 days
До 6 місяцівUp to 6 months
До 14 дняUp to 14 days
До 21 дняUp to 21 days
Хірургія
49/1500
Пацієнта віком 62 роки турбують постійний біль в епігастрії та схуднення на 12 кг. За даними фізикального та інструментального обстежень (фіброгастроскопія з біопсією, УЗД органів черевної порожнини, рентгенографія органів грудної клітки) у пацієнта виявлено рак тіла шлунка без ознак віддаленого метастазування. Висновок гістологічного дослідження: помірно диференційована аденокарцинома. Який адекватний обсяг оперативного втручання?A 62-year-old patient is concerned about constant pain in the epigastrium and weight loss of 12 kg. According to physical and instrumental examinations (fibrogastroscopy with biopsy, ultrasound of the abdominal organs, radiography of the organs of the chest) the patient was found to have cancer of the body of the stomach without signs of distant metastasis. The conclusion of the histological examination: moderately differentiated adenocarcinoma. What is the adequate amount of surgical intervention?
Пацієнт віком 37 років неодноразово перебував на лікуванні в психіатричній лікарні. Поточна госпіталізація зумовлена порушеннями поведінки у вигляді замкненості, відмови від їжі, періодичного хаотичного збудження зі стереотипними рухами. Під час огляду виявлено: пацієнт перебуває в ліжку, загальмований, контакт відсутній, на запитання не відповідає, поза одноманітна, гіпомімічний, симптом «хоботка», воскова гнучкість м'язів, симптом «повітряної подушки». Такий стан триває близько тижня. Встановіть попередній діагноз.A 37-year-old patient was repeatedly treated in a psychiatric hospital. The current hospitalization is due to behavioral disorders in the form of withdrawal, refusal to eat, periodic chaotic excitement with stereotyped movements. Under the time of the examination revealed: the patient is in bed, inhibited, does not respond to questions, the posture is monotonous, the symptom is 'proboscis', the symptom is 'air cushion'. This condition lasts about a week. Establish a preliminary diagnosis .
Шизофренія, проста формаSchizophrenia, simple form
Шизофренія, гебефренічна формаSchizophrenia, hebephrenic form
Шизофренія, параноїдна формаSchizophrenia, paranoid form
Реактивний ступорReactive stupor
Шизофренія, кататонічна формаSchizophrenia, catatonic form
Хірургія
51/1500
Пацієнтка 74 років скаржиться на біль та здуття живота, нудоту. Страждає на ішемічну хворобу серця, постінфарктний та дифузний кардіосклероз. Об'єктивно спостерігається: стан важкий, живіт здутий, черевна стінка слабко бере участь в акті дихання. Під час лапароскопії виявлено: у черевній порожнині невелика кількість каламутного випоту, одна з петель тонкої кишки темно-синюшнього кольору. Поставте діагноз.A 74-year-old patient complains of pain and abdominal distension, nausea. She suffers from coronary heart disease, post-infarction and diffuse cardiosclerosis. Objectively observed: the condition is severe, the abdomen is distended , the abdominal wall is weakly involved in the act of breathing. During laparoscopy, a small amount of cloudy effusion was found in the abdominal cavity, one of the loops of the small intestine is dark blue. Make a diagnosis.
Тромбоз мезентеріальних судинThrombosis of mesenteric vessels
Лабораторією СЕС визначений вміст афлотоксинів у м'ясі корів, яким згодовували запліснявілий корм. Рівень афлотоксину перевищує гранично-допустимий у 5 разів. Назвіть можливі зрушення в здоров'ї внаслідок афлатоксикозу у населення, що тривалий час вживає таке м'ясо:The SES laboratory determined the content of aflatoxins in the meat of cows that were fed with moldy feed. The level of aflatoxin exceeds the permissible limit by 5 times. Name the possible changes in health as a result of aflatoxicosis in the population that consumes such meat for a long time:
АнеміяAnemia
АтеросклерозAtherosclerosis
Цироз і первинний рак печінкиCirrhosis and primary liver cancer
--
ГломерулонефритGlomerulonephritis
Терапія
53/1500
У чоловіка 22 років з'явились сильні болі в нижній частині спини, які ірра-діюють у п'яткові кістки, внутрішню поверхню стегон, спостерігається підвищення температури до 38,2^oС. За три тижні відчув важкорухомість у хребті, болі в ілеосакральних суглобах. Рентгенологічно виявлено: розширення щілини ілеосакрального з'єднання, склерозування спінальних зв'язків. Про яке захворювання слід думати?A 22-year-old man developed severe pain in the lower back, radiating to the heel bones, the inner surface of the thighs, an increase in temperature to 38 ,2^oC. In three weeks, I felt stiffness in the spine, pain in the ileosacral joints. X-ray revealed: enlargement of the ileosacral joint. What disease should I think about?
Міжреберна невралгіяIntercostal neuralgia
Анкілозуючий спондилоартритAnkylosing spondylitis
Киста сакрального відділу хребтаSacral spine cyst
Грижа спинального каналуHernia of the spinal canal
Оперізуючий лишайZingles
Терапія
54/1500
У чоловіка 33-х років з'явилися множинні висипання на тулубі, розгинальних поверхнях верхніх та нижніх кінцівок, які місцями зливаються з утворенням бляшок, супроводжуються свербінням. Поверхня елементів висипань вкрита сріблясто-білими дрібнопластинчастими лусочками, які при зішкрібанні легко знімаються. При проведенні граттажу спостерігаються три послідовні феномени: стеаринової плями, термінальної плівки та крапкової кровотечі. Який діагноз можна припустити?A 33-year-old man developed multiple rashes on the trunk, extensor surfaces of the upper and lower limbs, which in places merge with the formation of plaques, accompanied by itching. The surface of the elements the rash is covered with silvery-white scales, which are easily removed when scraping. Three consecutive phenomena are observed: stearin spot, terminal film, and dot bleeding?
Жінка 22 років із підозрою на системний червоний вовчак скаржиться на <<летючий>> біль у суглобах рук і ніг, підвищення температури до 38,5-39^oС упродовж 3 тижнів, задишку, серцебиття, слабкість. Об'єктивно встановлено: еритема на щоках та носі. У крові виявлено: Нb - 90 г/л, тромбоцити - 135·10^9/л, ШОЕ - 43 мм/год. У сечі виявлено: білок - 2,66 г/л, еритроцитів - 8-10 в п/з. Виявлення яких антитіл у пацієнтки найбільш значиме для постановки діагнозу?A 22-year-old woman with suspicion of systemic lupus erythematosus complains of <> pain in the joints of her hands and feet, an increase in temperature to 38.5-39^oС for 3 weeks, shortness of breath, weakness. Objectively established: erythema on the cheeks and nose. Blood - 90 g/l, platelets - 43 mm/h urine was found: protein - 2.66 g/l, erythrocytes - 8-10 in p/z. Which antibodies are most significant for the diagnosis in the patient?
До фосфоліпідівTo phospholipids
До 2-х спіральної нативної ДНКUp to 2-helix native DNA
Ревматоїдний факторRheumatoid factor
До тромбоцитівTo platelets
КріоглобуліниCryoglobulins
Терапія
56/1500
Хлопчик 11-ти років хворіє на бронхіа-льну астму. На тлі ГРВІ з'явилася ядуха, сухий кашель, свистяче дихання. Па-цієнт впродовж години тричі з інтервалом у 20 хвилин отримував інгаляції бета-2-агоніста короткої дії та антихолінергічного засобу, проте стан не покращився. Наступним кроком невідкладної допомоги є призначення:An 11-year-old boy suffers from bronchial asthma. On the background of SARS, dyspnea, dry cough, and wheezing appeared. The patient three times within an hour with received inhalations of a short-acting beta-2-agonist and an anticholinergic agent at 20-minute intervals, but the condition did not improve. The next step in emergency care is to prescribe:
Хворий 52-х років звернувся до лікаря зі скаргами на задишку, кашель із мокротинням з домішками крові, підвищену пітливість вночі. В сімейному анамнезі хворих на рак легень немає. За даними картки профілактичних щеплень вакцину БЦЖ отримав вчасно. При фізикальному обстеженні температура - 37,2^oC, артеріальний тиск - 110/70 мм рт.ст., пульс - 100/хв., частота дихання - 20/хв., SpO_2 - 94% при кімнатному повітрі. При аускультації грудної клітки справа дихальні шуми не прослуховуються. На рентгенограмі ОГК правосторонній гідроторакс. Який наступний крок лікаря буде найбільш доречним?A 52-year-old patient consulted a doctor with complaints of shortness of breath, cough with blood-tinged sputum, increased sweating at night. There is no family history of lung cancer patients. According to the preventive vaccination card, he received the BCG vaccine on time. During the physical examination, the temperature was 37.2°C, the blood pressure was 110/70 mm Hg, the pulse was 100/min., the respiratory rate was 20/min., SpO_2 - 94% with room air. When auscultating the chest, no breathing sounds are heard. Right-sided hydrothorax will be the most appropriate step for the doctor?
Тест вивільнення гамма-інтерферонуGamma interferon release test
Проба МантуMantoux Test
Терапія
58/1500
Чоловік віком 49 років скаржиться на напади стенокардії, що виникають під час ходьби на відстань до 500 м. З анамнезу відомо, що він багато років хворіє на хронічний бронхіт. Під час обстеження виявлено: в легенях невелика кількість сухих розсіяних хрипів, ЧД - 18/хв. Межі серця розширені вліво, тони приглушені, ЧСС=ПС=86/хв., АТ - 160/100 мм рт. ст. У клінічному аналізі крові виявлено: Hb - 160 г/л, лейкоцити - 6,4 тис., ШОЕ - 7 мм/год. На ЕКГ спостерігається гіпертрофія лівого шлуночка. Застосування якої групи ліків протипоказане з урахуванням супутньої патології у пацієнта?A 49-year-old man complains of angina attacks that occur while walking up to 500 m. From the anamnesis, it is known that he has been suffering from chronic bronchitis for many years. During the examination, a small number of scattered wheezes were detected, heart rate was 18/min, heart rate was muffled, HR=86/min revealed: Hb - 160 g/l, leukocytes - 6.4 thousand, ESR - 7 mm/h. On the ECG, the use of which group of drugs is contraindicated in the patient's accompanying pathology?
АнтиагрегантиAntiplatelets
АнгіопротекториAngioprotectors
Пролонговані нітратиProlonged Nitrates
Антагоністи кальціюCalcium antagonists
Бета-блокаториBeta blockers
Терапія
59/1500
Хворий 48-ми років скаржиться на випорожнення 2-3 рази на день, великий обсяг калових мас з неприємним запахом, супроводжується болем навколо пупка, випадінням волосся, парестезіями. При огляді: блідість шкіри, знижена маса тіла, набряки на нижніх кінцівках. Болючість при пальпації навколо пупка та кишечника. В аналізах: в крові - анемія, в аналізі калу - стеаторея, креаторея, амілорея. Який синдром спостерігається у хворого?A 48-year-old patient complains of defecation 2-3 times a day, a large volume of feces with an unpleasant smell, accompanied by pain around the navel, hair loss, paresthesias. On examination: paleness of the body, swelling on the lower limbs. In the tests: in the blood - anemia, in the analysis of the stool - what syndrome is observed in the patient?
Синдром гіперкатаболічної ексудативної ентеропатіїSyndrome of hypercatabolic exudative enteropathy
В операційній під час проведення контролю за дотриманням санітарно-гігієнічного режиму відібрано проби повітря для лабораторного аналізу. Отримано такі результати: загальна кількість мікроорганізмів в 1 м^3 повітря - 490, зокрема гемолітичний стрептокок - не виявлено. Встановіть ступінь чистоти повітря в операційній.Air samples were taken for laboratory analysis in the operating room during monitoring of compliance with the sanitary and hygienic regime. The following results were obtained: the total number of microorganisms in 1 m^3 of air - 490, in particular, hemolytic streptococcus - not detected. Set the degree of air cleanliness in the operating room.
ЧистеNet
Помірно забрудненеModerately polluted
Сильно забрудненеHeavily polluted
Дуже чистеVery clean
ЗабрудненеContaminated
Терапія
61/1500
Хвора 35-ти років захворіла гостро з лихоманки до 39,0^oC та кашлю. Через 3 дні задишка спокою зросла до 35/хв. Вниз від кута правої лопатки перкуторно визначається тупий звук, відсутнє голосове тремтіння, дихальні шуми не вислуховую-ться. Яка тактика лікування?A 35-year-old patient became acutely ill with fever up to 39.0^oC and cough. After 3 days, rest dyspnea increased to 35/min. Down from the corner of the right a dull sound is determined by percussion of the scapula, there is no voice tremor, no breathing sounds are heard. What is the treatment strategy?
Жінка 31 року доставлена до лікарні в непритомному стані. Зі слів супроводжувачів в анамнезі у хворої зазначена двобічна адреналектомія з приводу хвороби Іценко-Кушинга. Об'єктивно встановлено: ціаноз слизових, гіперпігментація, клонічні судоми, загальмованість, АТ - 80/60 мм рт. ст., температура тіла - 34,5^oС, тони серця глухі. Під час пальпації живота спостерігається напруження передньої черевної стінки. Які невідкладні засоби лікарняної допомоги слід використати насамперед?A 31-year-old woman was brought to the hospital in an unconscious state. According to the attendants, the patient had bilateral adrenalectomy due to Itsenko-Cushing's disease. Objectively established: cyanosis mucous membranes, clonic convulsions, blood pressure - 80/60 mm Hg, heart sounds dull. During palpation of the abdomen, what emergency measures should be used first of all?
Уведення 40%-го розчину глюкозиIntroduction of 40% glucose solution
Внутрішньовенне введення електролітівIntravenous administration of electrolytes
Уведення інсулінуInsulin administration
Призначення панангінуPanangin appointment
Терапія
63/1500
Жінка 35 років скаржиться на задишку, відчуття стискання в правій половині грудної клітки, кашель із виділенням невеликої кількості слизово-гнійного харкотиння. Хворіє тиждень. Скарги пов'язує з переохолодженням. Об'єктивно виявлено: температура тіла - 38,7^oС, легкий акроціаноз губ, Рs - 90/хв., АТ - 140/85 мм рт. ст. Права половина грудної клітки відстає в акті дихання. Перкуторно встановлено: справа нижче кута лопатки тупість з межею до верху, дихання не вислуховується. Який найімовірніший діагноз?A 35-year-old woman complains of shortness of breath, a feeling of tightness in the right half of the chest, cough with the release of a small amount of mucous-purulent sputum. She has been ill for a week. She associates the complaints with Objectively revealed: body temperature - 38.7°C, Рs - 140/85 mmHg below the angle of the scapula, dullness with a border to the top, breathing is not heard. What is the most likely diagnosis?
Під час огляду студента лікарем було виявлено сухість та зроговіння шкіри на ліктях та колінах. На шкірі сідниць, стегон та литок зроговіння епітелію, якщо провести рукою, відчувається шорсткуватість шкіри цих ділянок, яке нагадувало <<гусячу шкіру>>. Студент також скаржився на погіршення зору у сутінках. Яке із наведених захворювань можна припустити?During the examination of the student, the doctor found dryness and keratinization of the skin on the elbows and knees. On the skin of the buttocks, thighs, and calves, keratinization of the epithelium, if you run your hand over it, you can feel the roughness of the skin of these areas, which resembled 'goosebumps'. The student also complained about the deterioration of vision at dusk. Which of the following diseases can be assumed?
Гіповітаміноз DHypovitaminosis D
Гіповітаміноз В_6Hypovitaminosis B_6
Гіповітаміноз КHypovitaminosis K
Гіповітаміноз АHypovitaminosis A
Гіповітаміноз СHypovitaminosis C
Гігієна
65/1500
Добовий раціон харчування дівчинки 10 років містить: вітамін В_1 - 1,2 мг, вітамін В_2 - 0,6 мг, вітамін В_6 - 1,4 мг, вітамін РР - 15 мг, вітамін С - 65 мг. під час оцінювання харчового статусу у дівчинки виявлено хейлоз, глосит, ангулярний стоматит, кон'юктивіт. У дівчинки, найімовірніше:The daily diet of a 10-year-old girl contains: vitamin B_1 - 1.2 mg, vitamin B_2 - 0.6 mg, vitamin B_6 - 1.4 mg, vitamin RR - 15 mg, vitamin C - 65 mg. Cheilosis, glossitis, angular stomatitis, conjunctivitis were detected in the girl, most likely:
С - гіповітамінозC - hypovitaminosis
В_1 - гіповітамінозB_1 - hypovitaminosis
В_6 - гіповітамінозB_6 - hypovitaminosis
РР - гіповітамінозRR - hypovitaminosis
В_2 - гіповітамінозB_2 - hypovitaminosis
Терапія
66/1500
Хвора 17-ти років при зрості 172 см і масі тіла 40 кг вважає себе дуже повною. Останні 2 роки дотримується суворої низькокалорійної дієти, одночасно до знемагання займається фізичними вправами, стягує талію шнурами. Часто викликає штучну блювоту. Скаржиться на неприєм-ні відчуття в епігастрії, аменорею. Визначте найбільш імовірний діагноз:A 17-year-old patient with a height of 172 cm and a body weight of 40 kg considers herself very fat. For the past 2 years, she has been following a strict low-calorie diet, and at the same time she is doing physical exercises until exhaustion , tightens the waist with cords. Often causes artificial vomiting. Complains of unpleasant sensations in the epigastrium. Determine the most likely diagnosis:
ДепресіяDepression
Реакція на стресReaction to stress
АнорексіяAnorexia
Розлад особистостіPersonality Disorder
БуліміяBulimia
Акушерство і гінекологія
67/1500
Жінка 48 років звернулась із приводу того, що протягом останніх 8-9 місяців менструації дуже рясні, призводять до анемізації, порушена працездатність. Протягом 2 років спостерігається гінекологом з приводу фіброміоми матки. Об'єктивно встановлено: шийка матки циліндрична, чиста, вічко закрите. Тіло матки збільшене до 9-10 тижнів вагітності, цупке, рухоме, безболісне. Придатки з обох сторін не визначаються. Параметрії вільні. Склепіння глибокі. Виділення слизові. Який найбільш вірогідний діагноз?A 48-year-old woman complained about the fact that during the last 8-9 months, menstruation is very abundant, leads to anemia, impaired working capacity. For 2 years, she has been observed by a gynecologist for Objectively, the uterus is cylindrical, the eye is closed, the uterus is tender, the appendages are not defined. The discharge is deep the most likely diagnosis?
Фіброміома матки та вагітністьUterine fibroids and pregnancy
Рак ендометріюEndometrial cancer
Ендометріоз із переважним ураженням тіла маткиEndometriosis with predominant damage to the body of the uterus
Терапія
68/1500
У чоловіка віком 25 років після переохолодження з'явилась гематурія. Об'єктивно спостерігається: АТ - 160/110 мм рт. ст. У аналізі сечі визначається: протеїнурія 3,5 г/л, еритроцити покривають усе поле зору, циліндри гіалінові 5-6 у полі зору. Який метод діагностики є найінформативнішним у цьому разі?A 25-year-old man developed hematuria after hypothermia. Objectively observed: blood pressure - 160/110 mm Hg. Urinalysis revealed: proteinuria 3.5 g/l, erythrocytes cover the entire field of vision, hyaline cylinders 5-6 in the field of vision. What diagnostic method is the most informative in this case?
УЗД нирокKidney ultrasound
ЦистоскопіяCystoscopy
Біопсія ниркиKidney biopsy
ТомографіяTomography
Екскреторна урографіяExcretory urography
Хірургія
69/1500
У потерпілого опікова травма 15 % поверхні тіла II-III ступеня. На двадцяту добу після травми у пацієнта спостерігаються різке підвищення температури тіла, загальна слабкість, часте поверхневе дихання, артеріальний тиск - 90/50 мм рт. ст., пульс - 112/хв. Яке ускладнення можна припустити?The victim has a burn injury of 15% of the body surface II-III degree. On the twentieth day after the injury, the patient has a sharp rise in body temperature, general weakness, frequent shallow breathing, blood pressure - 90/50 mm Hg, pulse - 112/min. What complications can be assumed?
Гнійний бронхітPurulent bronchitis
СепсисSepsis
ПневмоніяPneumonia
Гостра інтоксикаціяAcute intoxication
Анаеробна інфекціяAnaerobic infection
Терапія
70/1500
На прийом до лікаря-венеролога звернувся пацієнт, якому на підставі клінічних проявів і даних лабораторного обстеження був встановлений діагноз «урогенітальний хламідіоз». Під час призначення лікування цьому пацієнту основним препаратом потрібно вважати:A patient who, on the basis of clinical manifestations and laboratory examination data, was diagnosed with 'urogenital chlamydia'. During the appointment of treatment for this patient, the main the drug should be considered:
Сульфаніламідні препаратиSulfonamide drugs
Антибіотики пеніцилінового рядуAntibiotics of the penicillin series
Циклоферон або інші імуномодуляториCycloferon or other immunomodulators
Препарати групи тинідазолуDrugs of the tinidazole group
Антибіотики макролідиMacrolide antibiotics
Гігієна
71/1500
Який недостатньо оброблений продукт харчування може викликати захворювання - дифілоботріоз?Which insufficiently processed food product can cause the disease - diphyllobotriosis?
ЯловичинаBeef
СвининаPork
РибаFish
М'ясо птиціPoultry
БаранинаLamb
Терапія
72/1500
Хлопчик 6 років поступив із скаргами на набряки обличчя і нижніх кінцівок. Об'єктивно встановлено: у нижніх відділах легень притуплення, ослаблене дихання. Набряклість передньої черевної стінки. Асцит. АТ - 90/50 мм рт. ст. Загальний аналіз сечі виявив: білок - 4,2 г/л, лейкоцити - 5-6 у п/з, еритроцити - 2-3 у п/з. Загальний білок крові - 48,6 г/л, холестерин - 8,2 ммоль/л. Попередній діагноз: гострий гломерулонефрит. Укажіть клінічний варіант захворювання:A 6-year-old boy was admitted with complaints of swelling of the face and lower limbs. Objectively established: dullness in the lower parts of the lungs, weakened breathing. Swelling of the anterior abdominal wall. Ascites Blood pressure - 90/50 mmHg. Urinalysis - 4.2 g/l, erythrocytes - 2-3 g/l .6 g/l, cholesterol - 8.2 mmol/l. Preliminary diagnosis: acute glomerulonephritis. Specify the clinical variant of the disease:
Сечовий синдромUrinary syndrome
Нефротичний синдром з гематурією і гіпертензієюNephrotic syndrome with hematuria and hypertension
Нефритичний синдромNephritic syndrome
Нефротичний синдромNephrotic syndrome
Нефротичний синдром з гематурієюNephrotic syndrome with hematuria
Гігієна
73/1500
Проведено дослідження з метою визначення зв'язку між споживанням мікронут-рієнтів та захворюванням на катаракту серед людей похилого віку. Участь у дослідженні взяли 2900 людей у віці від 49 до 97 років. Кожний учасник заповнював опитувальник частоти споживання певних харчових продуктів та проходив офтальмоскопію з фотографуванням кришталика. У результаті дослідження, було виявлено, що достатня кількість у раціоні вітаміну A, рибофлавіну, ніацину та тіаміну поперед-жують виникнення ядерної катаракти. Який з перерахованих варіантів найбільш точно описує вид проведеного клінічного дослідження?A study was conducted to determine the relationship between micronutrient intake and cataracts in the elderly. The study involved 2,900 people aged 49 to 97 years old. Each participant filled out a questionnaire on the frequency of consumption of certain foods and underwent ophthalmoscopy. As a result of the study, it was found that sufficient amounts of vitamin A, niacin and thiamine in the diet prevent the occurrence of nuclear cataracts most accurately describes the type of clinical research conducted?
Крос-секціональне поперечне дослідженняCross-sectional cross-sectional study
Проспективне когортне дослідженняProspective cohort study
Дослідження ''випадок-контроль''Case-control study
Опис серії клінічних випадківDescription of a series of clinical cases
Хлопчика 5-ти років привели на прийом до лікаря зі скаргами на стрибкоподібне підвищення температури до 39,3^oC протягом 12-ти днів. При фізикальному обстеженні виявлені: ін'єкція бульбарних кон'юн-ктив з обох сторін, потріскані червоні губи та ''малиновий'' язик, шийна лімфаденопатія та еритематозний папульозний висип промежини, набряклість рук та ніг. Що з перерахованого лікар має призначити першочергово?A 5-year-old boy was brought to a doctor's appointment with complaints of a sudden rise in temperature up to 39.3^oC for 12 days. Physical examination revealed: injection of the bulbar conjunctiva on both sides, cracked red lips and 'raspberry' tongue, cervical lymphadenopathy and erythematous papular rash of the perineum, swelling of the hands and feet. Which of the following should the doctor prescribe first?
Фуросемід пероральноFurosemide Oral
Преднізолон внутрішньовенноPrednisone IV
Валацикловір пероральноValacyclovir Oral
Пеніцилін внутрішньовенноPenicillin IV
Імуноглобулін людини нормальний внутрішньовенноHuman immunoglobulin normal IV
Акушерство і гінекологія
75/1500
Жінка 32-х років звернулась до лікаря зі скаргами на відсутність вагітності впродовж 4-х років. У анамнезі: 5 років тому перша вагітність закінчилась штучним абортом. За даними вагінального дослідження та ультразвукового дослідження (УЗД) встановлено діагноз: ендометріоїдна кіста правого яєчника. Який оптимальний метод лікування?A 32-year-old woman turned to the doctor with complaints about the absence of pregnancy for 4 years. History: 5 years ago, the first pregnancy ended in an artificial abortion. According to a diagnosis of endometrioid cyst of the right ovary was established by vaginal examination and ultrasound examination. What is the optimal method of treatment?
Консервативна терапія естроген-геста-генними препаратамиConservative therapy with estrogen-progestogen drugs
Протизапальна терапіяAnti-inflammatory therapy
Акушерство і гінекологія
76/1500
Пацієнтка 40 років доставлена зі скаргами на переймоподібні болі внизу живота та надмірні кров'яні виділення зі статевих шляхів. Останні два роки менструації тривають до 16 днів, надмірні, із згустками, болісні. Під час бімануального дослідження встановлена наявність фіброматозного вузла, що народжується. Оберіть правильну тактику:A 40-year-old female patient was brought in with complaints of spasm-like pains in the lower abdomen and excessive bleeding from the genital tract. For the last two years, menstruation has lasted up to 16 days, excessive, with clots, painful. During the bimanual examination, the presence of a nascent fibromatous node is established. Choose the correct tactics:
Видалення фіброматозного вузла вагінальноRemoval of a fibromatous node vaginally
Екстирпація матки без додатківExtirpation of the uterus without attachments
Гормональний гемостазHormonal hemostasis
Надпіхвова ампутація матки без додатківSupravaginal amputation of the uterus without attachments
ВітамінотерапіяVitamin therapy
Терапія
77/1500
Пацієнт віком 35 років звернувся до лікарні зі скаргами на напади болю в правій поперековій ділянці, часте сечовипускання. В загальному дослідженні сечі білок 0,066 г/л; свіжі еритроцити 6-8 в полі зору. За даними УЗД помірний уростаз справа. Яке дослідження необхідно виконати пацієнту для встановлення діагнозу?A 35-year-old patient came to the hospital with complaints of pain attacks in the right lumbar region, frequent urination. In the general examination of urine, protein 0.066 g/l; fresh red blood cells 6 -8 in the field of vision. According to the ultrasound, moderate urostasis is needed for the patient to establish the diagnosis?
Пацієнт віком 20 років звернувся до лікаря зі скаргами на появу висипу на шкірі. Об'єктивно спостерігається: на шкірі тулуба, верхніх кінцівок, задньої поверхні шиї плямистий висип розміром 1-3 см, світло-коричневого кольору, що місцями зливається. Позитивна проба Бальцера. Який найімовірніший діагноз?A 20-year-old patient turned to the doctor with complaints about the appearance of a rash on the skin. Objectively observed: on the skin of the trunk, upper limbs, back of the neck, a spotted rash 1-3 cm, light brown, confluent in places. Positive Balzer's test. What is the most likely diagnosis?
Різнокольоровий лишайMulticolored lichen
Вторинний сифілісSecondary syphilis
Мікробна екземаMicrobial Eczema
Рожевий лишайPink lichen
ПарапсоріазParapsoriasis
Гігієна
79/1500
Робітник 40 років звернувся в поліклініку зі скаргами на швидку втомлюваність, епізодичні болі в животі, порушення сну. Півтора роки працює на приватному підприємстві у цеху зарядки акумуляторів та вулканізації шин. Об'єктивно встановлено: тони серця приглушені, АТ - 140/70 мм рт. ст., пульс - 84/хв., дихання везикулярне. У крові виявлено: Нb - 120 г/л, еритроцити - 4,0·10^12/л, еритроцити з базофільною зернистістю, загальна кількість ретикулоцитів - 0,3%. Аналіз сечі: питома вага - 1,018 г/см^3, лейкоцити - 5 в полі зору, креатинін сечі - 300 мкг/г. Визначте професійну патологію та вкажіть, які лабораторні дослідження підтвердять її наявність:A 40-year-old worker came to the clinic with complaints of rapid fatigue, episodic abdominal pain, sleep disturbances. He has been working for a year and a half at a private enterprise in a battery charging and tire vulcanizing shop Objectively, heart sounds are muffled, blood pressure - 140/70 mm Hg, vesicular respiration is found: Нb - 120 g/l, erythrocytes - 4.0·10 12/l, erythrocytes with basophilic granularity, total number of reticulocytes - 0.3%. Urine analysis: specific gravity - 1.018 g/cm^3, leukocytes - 5 in the field of vision, urine creatinine - 300 μg/g indicate which laboratory tests will confirm its presence:
Хронічна свинцева інтоксикація (сатурнізм), скарги хворогоChronic lead intoxication (saturnism), complaints of the patient
Хронічна свинцева інтоксикація (сатурнізм); базофільна зернистість еритроцитів та креатинін сечіChronic lead intoxication (saturnism); basophilic granularity of erythrocytes and urine creatinine
Карбокситоксикоз; рівень гемоглобіну в кровіCarboxytoxicosis; hemoglobin level in blood
Цинкова інтоксикація; загальна кількість ретикулоцитівZinc intoxication; total reticulocyte count
Терапія
80/1500
Пацієнтка віком 17 років скаржиться на біль та набряк ІІ пальця правої кисті. З анамнезу відомо що 3 доби по тому зробила манікюр. Біль з'явився на другий день. Об'єктивно спостерігається: навколонігтьовий валик набрякший, гіперемований, нависає над нігтьовою пластинкою, болючий під час пальпації. Який найімовірніший діагноз?A 17-year-old patient complains of pain and swelling of the II finger of the right hand. From the anamnesis, it is known that she had a manicure 3 days later. The pain appeared on the second day. Objectively observed: the nail plate is swollen, hyperemic, hanging over the nail plate, painful during palpation. What is the most likely diagnosis?
Шкірний панариційSkin panaritium
Піднігтьовий панариційSubungual panaritium
Підшкірний панариційSubcutaneous panaritium
ЕризипелоїдErysipeloid
ПароніхіяParonychia
Педіатрія
81/1500
У шестимісячної дитини гостро підвищилася температура тіла до 39^oС, з'явилося блювання, відмова від їжі, короткочасні тоніко-клонічні судоми. Об'єктивно спостерігається: дитина в'яла, сонлива, ригідність м'язів потилиці, симптоми Керніга, вибухання та напруження тім'ячка. Яке лабораторно-інструментальне обстеження необхідно провести для підтвердження діагнозу?A six-month-old child's body temperature rose sharply to 39^oС, vomiting, refusal to eat, short-term tonic-clonic convulsions appeared. Objectively observed: the child flabby, sleepy, stiffness of the occipital muscles, Kernig's symptoms, bulging and tension of the crown of the head. What laboratory-instrumental examination should be performed to confirm the diagnosis?
Люмбальна пункціяLumbar puncture
Біохімічне дослідження сироватки кровіBiochemical examination of blood serum
Рентгенографія черепаX-ray skull
МРТ головного мозкуMRI brain
НейросонографіяNeurosonography
Педіатрія
82/1500
У хворого 2 років із рецидивуючим бронхо-легеневим захворюванням, гіпотрофією, установлений діагноз: муковісцидоз. Яке дослідження повинно підтвердити клінічний діагноз?A 2-year-old patient with recurrent broncho-pulmonary disease, hypotrophy, diagnosed with cystic fibrosis. What research should confirm the clinical diagnosis?
Кількість хлоридів у потіNumber of chlorides in sweat
Рівень лужної фосфатози в сироватціSerum alkaline phosphatase level
Наявність білку в сечіPresence of protein in urine
Рівень кальцію в кровіCalcium level in blood
Вміст 2-фракції гамма-глобулінівContent 2-fraction of gamma-globulins
Терапія
83/1500
Пацієнтка віком 32 роки скаржиться на загальну слабкість, головний біль, надлишкову масу тіла, біль у кістках, порушення менструального циклу. Об'єктивно спостерігається: шкіра суха, ціанотична. У ділянці живота, плечей, стегон багряно-ціанотичні смуги розтягу. Відкладення жиру переважно на обличчі, шиї, тулубі. АТ - 165/100 мм рт. ст. У аналізі крові виявлено: глюкоза - 7,2 ммоль/л, рівень АКТГ - підвищений. Під час рентгенологічного дослідження кісток виявлено: ознаки остеопорозу. Який найімовірніший діагноз?A 32-year-old patient complains of general weakness, headache, excess body weight, bone pain, irregular menstrual cycle. Objectively observed: dry, cyanotic skin Cyanotic stretch marks on the abdomen, neck, and trunk. Blood pressure is 7.2 mmol/l - increased. During the X-ray examination of the bones, signs of osteoporosis were found. What is the most likely diagnosis?
Патологічний клімаксPathological climax
Синдром Іценко-КушингаItsenko-Cushing syndrome
Хвороба Іценко-КушингаItsenko-Cushing disease
Ожиріння аліментарнеAlimentary obesity
Гіпертонічна хворобаHypertensive disease
Терапія
84/1500
У чоловіка о 5-й годині ранку стався напад сильного болю за грудиною, що супроводжувався кволістю та холодним потом. На ЕКГ виявлено: куполоподібний підйом сегмента ST у відведеннях ІІ, ІІІ, avF, V5-V6. Після вщухання болю показники ЕКГ повернулися до норми. Який імовірний діагноз?At 5 o'clock in the morning, a man had an attack of severe pain behind the sternum, accompanied by weakness and cold sweat. The ECG revealed: dome-shaped elevation of the ST segment in leads II , ІІІ, avF, V5-V6. After the pain subsided, the ECG parameters returned to normal. What is the probable diagnosis?
Спонтанна стенокардія типу PrinzmetalSpontaneous angina of the Prinzmetal type
Гострий інфаркт міокарда в задньобоковій ділянці лівого шлуночкаAcute myocardial infarction in the posterolateral area of the left ventricle
Дилятаційна кардіоміопатіяDilated cardiomyopathy
МіокардитMyocarditis
ПерикардитPericarditis
Акушерство і гінекологія
85/1500
Вагітна жінка віком 25 років скаржиться на підвищення температури до 38,5^oC впродовж двох днів, кашель, задишку. Ці скарги з'явилися після переохолодження. Під час аускультації виявлено: крепітація та локалізовані вологі хрипи в нижній частині правої легені, там же спостерігається притуплення перкуторного звуку. У загальному аналізі крові спостерігається: лейкоцити - 11,0·10^9/л, ШОЕ - 22 мм/год. Який антибактеріальний засіб потрібно призначити?A 25-year-old pregnant woman complains of an increase in temperature to 38.5^oC for two days, cough, shortness of breath. These complaints appeared after hypothermia. During auscultation revealed: crepitation and localized moist rales in the lower part of the right lung, there is a dullness of the percussion sound. In the general blood analysis, leukocytes are observed - 11.0·10^9/l, ESR - 22 mm/h appoint?
АмоксицилінAmoxicillin
КарбеніцилінCarbenicillin
АмікацинAmikacin
ДоксициклінDoxycycline
ЛевофлоксацинLevofloxacin
Терапія
86/1500
Раніше здоровий хлопець 22-х років прийшов на прийом до лікаря зі скаргами на підвищену температуру та кашель з жовтим мокротинням, що продовжуються протягом 7-ми днів. При фізикальному обстеженні температура - 38,3^oC, артеріальний тиск - 130/70 мм рт.ст., частота серцевих скорочень - 79/хв., частота дихання - 17/хв., SpO_2 - 95% при кімнатному повітрі. При аускультації звучні вологі хрипи зліва. На оглядовій рентгенограмі консолідація у нижній частці лівої легені. Який з перерахованих препаратів буде найбільш доцільно призначити цьому пацієнту?A previously healthy 22-year-old boy came to the doctor with complaints of high temperature and cough with yellow sputum, which continued for 7 days. With physical examination temperature - 38.3°C, blood pressure - 130/70 mmHg, heart rate - 79/min, respiratory rate - 17/min, SpO_2 - 95% in room air wheezing on the left. Consolidation in the lower lobe of the left lung. Which of the following drugs would be most appropriate for this patient?
У шестимісячної дитини різко підвищилася температура тіла до 39^oС, з'явилися блювання, відмова від їжі, короткочасні тоніко-клонічні судоми. Дитина в'яла, сонлива. Виявлені ригідність м'язів потилиці, симптоми Керніга, вибухання та напруження тім'ячка. Яке лабораторно-інструментальне обстеження треба провести для верифікації діагнозу?A six-month-old child's body temperature rose sharply to 39^oС, vomiting, refusal to eat, and short-term tonic-clonic convulsions appeared. The child was lethargic, sleepy . The stiffness of the occiput muscles, Kernig's symptoms, bulging and tension of the crown of the head were detected. What laboratory-instrumental examination should be carried out to verify the diagnosis?
МРТ головного мозкуMRI brain
Люмбальну пункціюLumbar puncture
Біохімічне дослідження сироватки кровіBiochemical examination of blood serum
НейросонографіюNeurosonography
Рентгенографію черепаX-ray skull
Педіатрія
88/1500
У трирічної дитини спостерігаються напади, що супроводжуються ціанозом, різким занепокоєнням, присіданням навпочіпки. Об'єктивно виявлено: деформація фаланг пальців у вигляді 'барабанних паличок', нігті у формі 'скелець годинника'. Межі серцевої тупості зсунуті вліво та вправо, у ІІ міжребер'ї біля лівого краю грудини визначається систолічне дрижання, вислухується грубий систолічний шум з р.max. у ІІ міжребер'ї, ІІ тон над основою серця ослаблений. Під час рентгенологічного дослідження виявлено: серце у вигляді 'дерев'яного черевика', легеневий малюнок виражений слабо. Який найімовірніший діагноз?A three-year-old child has seizures accompanied by cyanosis, sharp anxiety, squatting. Objectively revealed: deformation of the phalanges of the fingers in the form of 'drumsticks', nails in The borders of the heart are shifted to the left and to the right, a systolic murmur is heard near the left edge of the sternum, and the second sound over the base of the heart is weakened X-ray examination revealed: the heart in the form of a 'wooden shoe', the pulmonary pattern is weak. What is the most likely diagnosis?
Роділля, І вагітність, 38 тижнів, скаржиться на нерегулярний переймоподібний біль внизу живота та в попереку протягом 2 діб, вночі спала погано. При зовнішньому акушерському дослідженні - тонус матки підвищений, поздовжнє положення плода І позиція, передній вид, голівка плода рухома над входом в малий таз. Піхвове дослідження - шийка матки до 2,5 см, відхилена дозаду, розм’якшена нерівномірно, зовнішнє вічко відкрите до 0,5 см, внутрішнє вічко закрите. Ваш діагноз:Mother in labor, 1st pregnancy, 38 weeks, complains of irregular spasm-like pain in the lower abdomen and lower back for 2 days, slept poorly at night. During external obstetric examination - uterine tone elevated, longitudinal position of the fetus I position, front view, the fetal head is mobile above the entrance to the pelvis. Vaginal examination - the cervix is up to 2.5 cm, deviated backward, unevenly softened, the external eye is open to 0.5 cm, the internal eye close. Your diagnosis:
Передвісники пологівForerunners of childbirth
Первинна слабкість пологової діяльностіPrimary weakness of labor activity
Вторинна слабкість пологової діяльностіSecondary weakness of labor activity
Патологічний прелімінарний періодPathological preliminary period
Терапія
90/1500
У дитини 11 років з цукровим діабетом 1-го типу на фоні гострої респіраторної вірусної інфекції посилилася спрага, з'явилася нудота та блювота. У видихуваному повітрі різкий запах ацетону. Очі запалі. Дихання шумне, аритмічне, типу Кусмауля. ЧСС - 128/хв., глухість серцевих тонів. Діагностована кетоацидотична кома. Укажіть провідний патогенетичний механізм розвитку невідкладного стану у дитини:In an 11-year-old child with type 1 diabetes, against the background of an acute respiratory viral infection, thirst increased, nausea and vomiting appeared. The exhaled air has a sharp smell of acetone Eyes inflamed. Breathing is noisy, heart rate is 128/min. Diagnosed ketoacidotic coma:
Недостатність функції кори наднирниківAdrenal cortex insufficiency
Гіперактивність центральної нервової системиHyperactivity of the central nervous system
Надлишкова продукція інсулінуExcess insulin production
Активація симпато-адреналової системиActivation of sympatho-adrenal system
Терапія
91/1500
Жінка 32 років звернулась з приводу збільшення щитоподібної залози. Під час аварії на ЧАЕС перебувала у зоні підвищених радіоактивних опадів. Об'єктивно встановлено: щитоподібна залоза збільшена до ІІ ст., у ній пальпується щільний вузол, малорухомий, неболючий. Підщелепні лімфатичні вузли збільшені, неболючі. Під час УЗД у щитоподібній залозі виявлено гіпоехогенний утвір без чітких меж з кальцинатами. АТ - 120/70 мм рт. ст. Пульс - 78/хв. На основі приведених даних у хворої можна запідозрити:A 32-year-old woman applied for an enlarged thyroid gland. During the accident at the Chernobyl nuclear power plant, she was in the zone of increased radioactive fallout. It was objectively established: the thyroid gland was enlarged to the II century ., a dense, non-moving node is palpable. During ultrasound, a hypoechoic mass with calcifications was detected. Pulse - 78 mmHg. Based on the given data, the patient may be suspected of:
Підгострий тиреоїдитSubacute thyroiditis
Рак щитоподібної залозиThyroid cancer
Дифузний токсичний зобDiffuse toxic goiter
Аденому щитоподібної залозиAdenoma of the thyroid gland
Дифузний нетоксичний зобDiffuse non-toxic goiter
Акушерство і гінекологія
92/1500
Пацієнтка віком 22 роки скаржиться на ниючий біль у правій здухвинній ділянці, що турбує протягом тижня, нудоту зранку, зміну відчуття смаку. З анамнезу відомо: затримка менструації - 3 тижні. Об'єктивно спостерігається: АТ - 110/70 мм рт. ст., Рs - 78/хв, температура тіла - 37,0^oС. Під час бімануального дослідження виявлено: матка дещо збільшена, пом'якшена, рухома, неболюча. Додатки: справа пальпується болюче утворення 3·4 см, щільно-еластичної консистенції, помірно рухоме. Який попередній діагноз?A 22-year-old patient complains of aching pain in the right pubic region, which has been bothering her for a week, morning sickness, a change in the sense of taste. From the anamnesis, it is known: delayed menstruation - 3 Objectively observed: blood pressure - 110/70 mm Hg, body temperature - 37.0 °C. During bimanual examination, the uterus was slightly enlarged, mobile, painless Appendices: a painful mass of 3·4 cm is palpated on the right, of a dense and elastic consistency. What is the preliminary diagnosis?
Трубна вагітність, що перерваласьAborted tubal pregnancy
Гігієна
93/1500
Унаслідок неправильного зберігання проросла або позеленіла картопля має гіркуватий смак. Яка отруйна речовина, що міститься в такій картоплі, може спричинити харчове отруєння?Due to improper storage, sprouted or green potatoes have a bitter taste. What poisonous substance contained in such potatoes can cause food poisoning?
Гельвелова кислотаHelvelic acid
МускаридинMuscaridine
СоланінSolanine
МускаринMuscarine
ФазинFazin
Терапія
94/1500
Чоловік віком 34 роки захворів 3 дні тому після переохолодження. Скаржиться на підвищення температури до 39,2^oС, виражену загальну слабкість, пітливість, кашель. Кашель був спочатку сухий, протягом останньої доби з'явилася невелика кількість «іржавого» мокротиння. Під час об'єктивного обстеження спостерігається: герпес на губах, перкуторно над легенями справа в нижніх відділах спостерігається притуплений звук. Аускультативно виявлені бронхіальне дихання, тахікардія. В органах черевної порожнини змін не виявлено. Який діагноз є найімовірнішим?A 34-year-old man fell ill 3 days ago after hypothermia. He complains of an increase in temperature to 39.2^oС, severe general weakness, sweating, cough. The cough was initially dry, during the last day a small amount of 'rusty' sputum appeared. During the objective examination, a dull sound was observed over the lungs on the right. Bronchial breathing, tachycardia were detected in the abdominal organs not detected. What is the most likely diagnosis?
Під час операції у жінки 67 років із приводу гострого деструктивного холециститу визначена наявність гнійного холангіта. Під час ревізії холедоха не знайдені конкременти та ознаки стенозу дуоденального соска. Виконано холецистектомію. Як слід закінчити операцію?During the operation of a 67-year-old woman for acute destructive cholecystitis, the presence of purulent cholangitis was determined. No calculi or signs of stenosis of the duodenal nipple were found during the choledochal revision. A cholecystectomy was performed. How should the operation be completed?
Пацієнт 27 років близько року спостерігає втому, пітливість, важкість у лівому підребер'ї, особливо після прийому їжі. Об'єктивно спостерігається: збільшена селезінка, печінка. В крові виявлено: ермтроцити - 3,2·10^12/л, Hb - 100 г/л, КП - 0,87, лейкоцити - 100·10^9/л, базофіли - 7%, еозинофіли - 5%, моноцити - 15%, юні - 16%, паличкоядерні - 10%, сегментоядерні - 45%, лімфоцити - 2%, моноцити - 0%, ретикулоцити - 0,3%, тромбоцити - 400·10^9/л, ШOЕ - 25 мм/год. Поставте діагноз.A 27-year-old patient has been experiencing fatigue, sweating, and heaviness in the left hypochondrium for about a year, especially after eating. Objectively observed: enlarged spleen, liver. In blood revealed: erythrocytes - 3.2·10^12/l, Hb - 100 g/l, CP - 0.87, leukocytes - 100·10^9/l, basophils - 7%, eosinophils - 5%, monocytes - 15%, young - 16%, rod-nuclear - 10%, segmentonuclear - 45%, lymphocytes - 2%, monocytes - 0%, reticulocytes - 0.3%, platelets - 400·10^9/l, SOE - 25 mm/ Make a diagnosis.
Хронічний лімфолейкозChronic lymphocytic leukemia
ЕритреміяErythremia
Хронічний мієлолейкозChronic myelogenous leukemia
Гострий лейкозAcute leukemia
Цироз печінкиLiver cirrhosis
Хірургія
97/1500
Чоловік 50-ти років звернувся до лікаря зі скаргами на біль в стегні після падіння на сходах. Піднятись та ходити не може. Права нога коротша за ліву на 5 см, ступня ротована назовні. Пульс на правій тильній артерії стопи задовільний. Стегно збільшене в об'ємі, деформоване, в середній третині болюча пальпація. Виявляється патологічна рухомість, крепітація. Який діагноз є найбільш імовірним?A 50-year-old man went to the doctor complaining of hip pain after falling down the stairs. He can't get up and walk. The right leg is 5 cm shorter than the left , the foot is rotated outwards. The pulse on the right back artery is satisfactory. The thigh is enlarged, painful palpation. What is the most likely diagnosis?
Перелом діафіза стегнової кісткиFracture of the diaphysis of the femur
Дайте оцінку фізичного розвитку десятирічної дівчинки за шкалою регресії, якщо показники ваги тіла та окружності грудної клітки знаходяться у межах pm 1 сигми.Give an estimate of the physical development of a ten-year-old girl on a regression scale, if the indicators of body weight and chest circumference are within pm 1 sigma.
ВисокийHigh
ДисгармонійнийDisharmonious
СереднійAverage
ГармонійнийHarmonic
НизькийLow
Гігієна
99/1500
У комп'ютерному залі науково-дослідного сектору політехнічного інституту робочі місця лаборантів-операторів розташовані біля передніх панелей моніторів. Упродовж усього робочого дня оператори перебувають під впливом електромагнітних хвиль надвисокої частоти. Вплив електромагнітних хвиль великої інтенсивності у вказаному діапазоні частот є особливо небезпечним для:In the computer hall of the research sector of the polytechnic institute, the workplaces of laboratory assistants-operators are located near the front panels of the monitors. Throughout the working day, the operators are under the influence of ultra-high electromagnetic waves frequencies. The influence of electromagnetic waves of high intensity in the indicated frequency range is especially dangerous for:
Зорового аналізатораVisual Analyzer
Слухового аналізатораHearing Analyzer
Больової чутливостіPain sensitivity
Сенсорної чутливостіTouch sensitivity
Тактильної чутливостіTactile sensitivity
Акушерство і гінекологія
100/1500
Встановлено, що на 100 пологів у жінок, які мають фактори ризику, було 30 передчасних пологів, а у жінок, які не мають факторів ризику, було 5 передчасних пологів. Який метод статистичної обробки даних буде оптимальним для використання лікарем, щоб оцінити вірогідність розбіжностей у порівнюваних групах?For every 100 births, women with risk factors had 30 premature births, while women without risk factors had 5 premature births . What statistical data processing method would be optimal for a physician to assess the likelihood of differences in the compared groups?
Розрахунок середніх величинCalculation of average values
Метод стандартизаціїStandardization method
Розрахунок відносних величинCalculation of relative values
Жінка 28 років звернулась до гінеколога зі скаргами на неплідність протягом 3 років. Менструальна функція не порушена. В анамнезі вказано: один штучний аборт, хронічний сальпінгоофорит. Контрацептивні засоби не використовує. Спермограмма чоловіка без патології. З якого методу слід почати обстеження для виявлення причин безпліддя?A 28-year-old woman turned to a gynecologist with complaints of infertility for 3 years. Menstrual function is not disturbed. The anamnesis indicates: one artificial abortion, chronic salpingo-oophoritis. Contraceptives are not uses. A man's spermogram without pathology. What method should be used to identify the causes of infertility?
ЛапароскопіяLaparoscopy
Діагностичне вишкрібання порожнини маткиDiagnostic scraping of the uterine cavity
ГістероскопіяHysteroscopy
Гормональне дослідженняHormonal study
ГістеросальпінгографіяHysterosalpingography
Терапія
102/1500
Пацієнтка віком 44 роки скаржиться на загальну слабкість, сонливість, серцебиття, сухість шкіри, зниження працездатності. Об'єктивно спостерігається: шкіра звичайного кольору, пульс - 72/хв, АТ - 125/80 мм рт. ст., щитоподібна залоза дифузно збільшена до ІІ ступеня, щільна, рухлива, болюча. Під час УЗД виявлено: зниження ехогенності, неоднорідність ехоструктури, потовщення капсули залози. У аналізі крові спостерігається: рівень Т4 - знижений, ТТГ - підвищений, антитіла до тиреопероксидази та антимікросомальні антитіла значно підвищені. Який найімовірніший діагноз?A 44-year-old female patient complains of general weakness, drowsiness, palpitations, dry skin, reduced work capacity. Objectively observed: normal-colored skin, pulse - 72/min , blood pressure - 125/80 mmHg, the thyroid gland is diffusely enlarged to the II degree, during ultrasound, it is found: a decrease in echogenicity, a thickening of the gland capsule. In the blood analysis, the level of T4 is reduced , TSH is increased, antibodies to thyroperoxidase and antimicrosomal antibodies are significantly increased. What is the most likely diagnosis?
Аутоімунний тиреоїдитAutoimmune thyroiditis
Підгострий тиреоїдитSubacute thyroiditis
Дифузний токсичний зоб ІІ ступеняDiffuse toxic goiter II degree
Рак щитоподібної залозиThyroid cancer
Дифузний еутиреоїдний зоб ІІ ступеняDiffuse euthyroid goiter II degree
Терапія
103/1500
Дівчина 22-х років прийшла до лікаря на планове обстеження. Вона викурює 1 пач-ку сигарет у день протягом 5-ти років. Має одного постійного статевого партнера, користуються презервативами. Дідусь по татовій лінії помер від серцевого нападу у 60 років. При фізикальному обстеженні пульс - 78/хв., частота дихання - 14/хв., артеріальний тиск - 110/70 мм рт.ст. При аус-культації серця вислуховується голосистолічний шум у ІІ міжребер'ї зліва від грудини. Яка рекомендація лікаря цій пацієнтці буде найбільш доречною?A 22-year-old girl came to the doctor for a routine examination. She has smoked 1 pack of cigarettes a day for 5 years. She has one permanent sexual partner, paternal grandfather died of a heart attack at the age of 60. Heart rate is 78/min., blood pressure is 110/70 mm Hg Holosystolic murmur in the 2nd intercostal space to the left of the sternum. What would be the most appropriate recommendation for this patient?
Навчитися самообстеженню молочних залозLearn breast self-examination
Скринінг гіперліпідеміїScreening for hyperlipidemia
Кольпоскопія з біопсієюColposcopy with biopsy
Пройти ПАП-тестPass the Pap test
Пройти ПАП-тест та здати аналіз на ВПЛPass a Pap test and pass an analysis for HPV
Гігієна
104/1500
Під час профілактичного огляду населення міста у 25% людей виявлено ознаки флюорозу: плямиста емаль зубів, у частини обстежених - генералізований остеосклероз із кальцифікацією міжхребетних зв'язок. Яка найімовірніша причина виникнення флюорозу?During a preventive examination of the city's population, 25% of people were found to have signs of fluorosis: mottled tooth enamel, some of the examined had generalized osteosclerosis with calcification of the intervertebral ligaments. What is the most likely the cause of fluorosis?
Надмірний вміст фтору у грунті та водіExcessive fluoride content in soil and water
Надмірний вміст фтору в рослинних продуктахExcessive content of fluorine in vegetable products
Недостатнє надходження фтору в організм із чаємInsufficient intake of fluoride in the body with tea
Недостатній вміст фтору в тваринних продуктахInsufficient fluoride content in animal products
Недостатній вміст фтору у грунті та водіInsufficient fluoride content in soil and water
Педіатрія
105/1500
Восьмирічний пацієнт спостерігається через затримку зросту. Народився в асфіксії з масою тіла 2800 г. В школі навчається добре. Батьки середнього зросту. Об'єктивно спостерігається: зріст - 107 см, маса тіла - 23 кг, недорозвинення лицьового скелета на кшталт лялькового обличчя. Волосся тонке, шкіра суха з іктеричним відтінком. Підшкірно-жирова клітковина розвинута добре на шиї, грудях, животі. М'язи розвинуті недостатньо. Який найімовірніший діагноз?An eight-year-old patient is observed due to growth retardation. He was born asphyxiated with a body weight of 2800 g. He studies well at school. Parents are of average height. Objectively observed: height - 107 cm, body weight - 23 kg, underdevelopment of the facial skeleton like a doll's face. The skin is dry with a icteric shade. The subcutaneous fat is well developed on the neck, chest, abdomen. What is the most likely diagnosis?
КраніофарінгіомаCraniopharyngioma
ХондродистрофіяChondrodystrophy
Синдром ФанконіFanconi Syndrome
Гіпофізарний нанізмPituitary dwarfism
Синдром ДаунаDown Syndrome
Хірургія
106/1500
Чоловік віком 38 років скаржиться на переймоподібні болі в животі, нудоту, блювоту, вздуття живота, невідходження газів, відсутність стільця. Із анамнезу відомо, що нещодавно пацієнта проперовано з приводу апендициту. Пальпаторно живіт болісний у всіх відділах, напружений. Визначається шум плескоту, нечіткі симптоми подразнення очеревини, гіперперистальтика. Який попередній діагноз?A 38-year-old man complains of spasm-like abdominal pain, nausea, vomiting, abdominal distension, non-emission of gases, absence of stool. It is known from the anamnesis that the patient was recently operated on of appendicitis. The abdomen is painful on palpation, tense. There is a rumbling sound, vague symptoms of peritoneum irritation. What is the preliminary diagnosis?
Роділля віком 29 років, пологи треті, у терміні 40 тижнів народила хлопчика масою 1900 г, зростом 48 см, із оцінкою за шкалою Апгар 7-8 балів. У ІІІ періоді пологів народилася плацента розмірами 17х16х1,5см, масою 340 г, із множинними петрифікатами, оболонки зеленуватого кольору. Яка патологія найімовірніше спричинила затримку внутрішньоутробного росту плода?A 29-year-old woman in labor, her third delivery, gave birth to a boy weighing 1900 g, height 48 cm, with an Apgar score of 7-8 points at 40 weeks. In the third period of labor, a placenta was born measuring 17x16x1.5 cm, weighing 340 g, with multiple petrifications, a greenish membrane. What pathology most likely caused intrauterine growth retardation?
Передчасне відшарування нормально розташованої плацентиPremature detachment of a normally located placenta
Гіпотрофія плода І стуненяFetal hypotrophy and moaning
Передчасні пологиPremature birth
Плацентарна недостатністьPlacental insufficiency
Терапія
108/1500
Чоловік 56-ти років, звернувся до лікаря з носовою кровотечею, що розпочалася з лівої ніздрі 30 хвилин тому. Будь-які травми носа пацієнт заперечує. В анамнезі: частих носових кровотеч немає, подібний епізод вперше. Хворіє на фібриляцію передсердь (приймає варфарин) та артеріальну гіпертензію (лікується гідрохлортіазидом, атенололом). Температура тіла - 37,2^oC, пульс - 86/хв., артеріальний тиск - 120/70 мм рт.ст. Перед тим, як визначити необхідність тампонади носа, який першочерговий крок лікаря буде найбільш доречним?A 56-year-old man consulted a doctor with a nosebleed that started from the left nostril 30 minutes ago. The patient denies any injuries to the nose. In the anamnesis: there are no frequent nosebleeds, this is the first time he has atrial fibrillation (he takes warfarin) and hypertension (he is treated with hydrochlorothiazide, atenolol). Body temperature - 37.2°C, pulse - 86/min. Before determining the need for nasal tamponade, what is the most appropriate first step of the doctor?
Прикласти холодний компрес на переніссяApply a cold compress to the bridge of the nose
Ввести вітамін K внутрішньом'язовоInject vitamin K intramuscularly
Оксиметазоліну інтраназально, попросити хворого затиснути крило носа та нахилитися впередOxymetazoline intranasally, ask the patient to pinch the wing of the nose and lean forward
Акушерство і гінекологія
109/1500
Жінка 27-ми років на 8-му тижні вагітності скаржиться лікарю, що за останні 8 днів вона відмічає тривалу нудоту та блювання майже після усіх прийомів їжі. За останній тиждень пацієнтка втратила 3 кг ваги. Зараз при рості 160 см жінка важить 46 кг. Пульс - 100/хв., артеріальний тиск - 90/50 мм рт.ст. При огляді відмічаються сухість слизових оболонок, зниження тургору шкіри та астенічна статура. Гінекологічний огляд виявив розмір матки, що відповідає 8-му тижню вагітності, без патологічних змін. На УЗД виявлена вагітність одним плодом. Концентрація гемоглобіну - 150 г/л. У загальному аналізі сечі виявлені кетонові тіла (+++). Що із перерахованого є найбільш доречним наступним кроком у веденні пацієнтки?A 27-year-old woman in the 8th week of pregnancy complains to the doctor that for the past 8 days she has been experiencing prolonged nausea and vomiting after almost every meal. In the last The patient has lost 3 kg of weight. Now she is 160 cm tall. The pulse is 100/min. During the examination, the mucous membranes are dry and the body is asthenic the examination revealed the size of the uterus, which corresponds to the 8th week of pregnancy, and the pregnancy was detected by one fetus. In the general analysis of the urine, ketone bodies were found (+++). appropriate next step in the management of the patient?
Постільний режим та часте годування малими порціямиBed rest and frequent feeding in small portions
Внутрішньовенна інфузійна терапія та призначення антиеметиківIntravenous infusion therapy and prescription of antiemetics
Пероральний прийом антиеметиків та антихолінергічних препаратівOral intake of antiemetics and anticholinergic drugs
Ендоскопічне дослідження та промивання шлункаEndoscopy and gastric lavage
Внутрішньовенне введення beta-адрено-блокаторів та парентеральне харчуванняIntravenous administration of beta-blockers and parenteral nutrition
Терапія
110/1500
Жінку віком 24 років шпиталізовано зі скаргами на блювання за типом <<кавової гущі>>, запаморочення, загальну слабкість. Під час об'єктивного обстеження встановлено: пацієнтка у свідомості, шкіра бліда, суха, над легенями аускультативно - жорстке дихання. ЧД - 20/хв, АТ - 100/80 мм рт. ст, пульс - 100/хв. Живіт м'який, безболісний. Перитонеальні симптоми негативні. Стілець рідкий, чорний. Діурез збережений. У клінічному аналізі крові виявлено: Нb - 95 г/л, еритроцити - 3,1 г/л, лейкоцити - 9,8 г/л. Який показник шокового індекса Альговера у цієї пацієнтки?A 24-year-old woman was hospitalized with complaints of vomiting of the type <>, dizziness, general weakness. During the objective examination, it was found: the patient in consciousness, skin is dry, respiratory rate is 20/min, pulse is 100/min. Stool is liquid. Diuresis is preserved. Clinical blood analysis revealed: Hb - 3.1 g/l, leukocytes - 9.8 g/l. What is this patient's shock index?
22
11
0,850.85
1,51.5
1,251.25
Терапія
111/1500
Жінка 46 років, яка рік тому мала резекцію щитоподібної залози з приводу багатовузлового зобу, скаржиться на загальну слабкість, сонливість, постійне відчуття втоми, зниження працездатності, закрепи, набряк обличчя та кінцівок. Об'єктивно спостерігається: температура тіла - 36^oC. Шкірні покриви сухі, зморшкуваті, почало випадати волосся. Аменорея. Яке захворювання виникло у пацієнтки? A 46-year-old woman, who a year ago had a resection of the thyroid gland due to a multinodular goiter, complains of general weakness, drowsiness, a constant feeling of fatigue, reduced work capacity, constipation, edema face and limbs. Body temperature is 36°C. The skin is wrinkled. What disease did the patient have?
60-річна жінка доставлена у відділення невідкладної допомоги зі скаргами на раптовий інтенсивний головний біль та нудоту. Останні півроку спостерігалася легка диплопія. Протягом багатьох років хворіє на артеріальну гіпертензію та цукровий діа-бет II типу. Артеріальний тиск - 160/90 мм рт.ст., частота серцевих скорочень - 82/хв. Фізикальне обстеження виявило правосторонній птоз, легку анізокорію та ригідність потиличних м'язів. Атаксія не спостерігається. Який діагноз є найбільш імовірним?A 60-year-old woman was brought to the emergency department with complaints of sudden intense headache and nausea. Mild diplopia has been observed for the past six months. She has been suffering from arterial hypertension for many years and Type II diabetes. Blood pressure - 160/90 mm Hg. Physical examination revealed right-sided ptosis and stiffness of the occipital muscles. What is the most common diagnosis probable?
Чоловік 28 років після піднімання ваги відчув сильний біль у попереку, який іррадіював у праву ногу. Звернувся до лікаря. Після огляду лікар поставив діагноз: гострий дискогенний попереково-крижовий радикуліт. Яке обстеження треба пройти хворому для підтвердження діагнозу?A 28-year-old man, after lifting weights, felt severe pain in the lower back, which radiated to the right leg. He consulted a doctor. After an examination, the doctor made a diagnosis: acute discogenic lumbosacral sciatica. What examination should the patient undergo to confirm the diagnosis?
Загальний аналіз сечіGeneral urinalysis
МРТ поперекового відділу хребтаMRI of the lumbar spine
Електроміографія м'язів нігElectromyography of leg muscles
Люмбальна пункціяLumbar puncture
Рентгенографія нирокKidney X-ray
Терапія
114/1500
Працівник заводу з виробницва фарб скаржиться на неприємний смак у роті, переймоподібні болі в животі, закрепи, які не знімаються послаблюючими засобами. Об'єктивно встановлено: шкіра обличчя блідо-сіруватого кольору, по краю ясен біля передніх зубів - смужка сіровато-бузкового кольору. Під час лабораторного дослідження виявлено: у крові анемію, ретикулоцитоз, базофільну зернистість еритроцитів, педвищений вміст білірубіну; у сечі - підвищений вміст порфірину. Яке захворювання у робітника?An employee of a paint production plant complains of an unpleasant taste in the mouth, spasm-like pains in the stomach, constipation that is not removed by laxatives. Objectively established: the skin of the face is pale - grayish color, along the edge of the gums - a grayish-lilac color. During the laboratory examination, anemia, reticulocytosis, increased bilirubin content in the urine - increased porphyrin content were found. ;
Пацієнтка 20-ти років проходить лікування з приводу анемії (гемоглобін - 72 г/л). Півтора роки тому після мимовільного викидня у терміні 16 тижнів та крововтрати, відмічає зниження пам'яті, втомлюваність, втрату апетиту, сухість шкіри, ламкість нігтів, набряклість, порушення менструальної функції. Об'єктивно: артеріальний тиск - 80/55 мм рт.ст., пульс - 54/хв., зріст - 168 см, вага - 48 кг, гіпоплазія статевих органів. Призначення якого з перерахованих препаратів буде найбільш доречним цій пацієнтці?A 20-year-old female patient is being treated for anemia (hemoglobin - 72 g/l). One and a half years ago, after an involuntary miscarriage at 16 weeks and blood loss, she notes memory loss, fatigue, loss of appetite, dry skin, brittle nails, swelling, menstrual dysfunction Objectively: blood pressure - 80/55 mm Hg, pulse - 54/min, height - 168 cm, weight - 48 kg, genital hypoplasia. Which of the listed drugs would be most appropriate for this patient?
Імуноглобулін людини нормальнийHuman immunoglobulin is normal
МетотрексатMethotrexate
ІнфліксимабInfliximab
ГідроксихлорохінHydroxychloroquine
ГідрокортизонHydrocortisone
Терапія
116/1500
Чоловіка 63 років госпіталізовано зі скаргами на біль у поперековій ділянці ліворуч, погіршення апетиту, слабкість, періодичну появу крові в сечі протягом місяця. Шкіра бліда. Анемія: еритроцити - 3,1·10^12/л, Нb - 101 г/л, ШОЕ - 37 мм/год., протеїнурія - 0,37 г/л, гематурія на все поле зору, креатінемія - 0,270 ммоль/л. Яке діагностичне припущення?A 63-year-old man was hospitalized with complaints of pain in the lumbar region on the left, loss of appetite, weakness, periodic appearance of blood in the urine for a month. The skin is pale. Anemia: erythrocytes - 3.1·10^12/l, Hb - 101 g/l, ESR - 37 mm/h, proteinuria - 0.37 g/l, hematuria in the entire field of vision, creatininemia - 0.270 mmol/l. What is the diagnostic assumption ?
Чоловік 40 років протягом 10 років страждає на хронічний гломерулонефрит. Скаржиться на блювоту, судоми м'язів нижніх кінцівок. АТ - 180/120 мм рт. ст., креатинін у сироватці крові - 770 мкмоль/л, швидкість клубочкової фільтрації - 5 мл/хв. Протягом останніх двох днів діурез знизився до 400 мл на добу. Яка лікувальна тактика показана для цього хворого?A 40-year-old man has been suffering from chronic glomerulonephritis for 10 years. He complains of vomiting, muscle spasms of the lower limbs. BP - 180/120 mm Hg, creatinine in blood serum - 770 μmol/l, glomerular filtration rate - 5 ml/min. During the last two days, diuresis has decreased to 400 ml per day. What therapeutic tactics are indicated for this patient?
ГемосорбціяHemosorption
СорбентиSorbents
ГемодіалізHemodialysis
ГемофільтраціяHemofiltration
ПлазмаферезPlasmapheresis
Терапія
118/1500
Пацієнта віком 55 років шпиталізовано у непритомному стані до відділення реанімації. Об'єктивно спостерігається: стан важкий, зіниці розширені, реагують на світло, дихання шумне, ЧД - 10/ хв, пульс на променевих і сонних артеріях не визначається, АТ - 40/20 мм рт. ст. На ЕКГ виявлено: тріпотіння шлуночків з ЧСС - 210/хв. Які реанімаційні заходи потрібно провести насамперед?A 55-year-old patient was hospitalized in an unconscious state to the intensive care unit. Objectively observed: the condition is severe, the pupils are dilated, react to light, breathing is noisy, BH - 10 / min, the pulse on the radial and carotid arteries is not determined, BP - 40/20 mmHg. Ventricular flutter with heart rate - 210/min. What resuscitation measures should be carried out first?
Внутрішньовенне введення норадреналінуIntravenous administration of norepinephrine
Електроімпульсну терапіюElectropulse therapy
Зовнішній масаж серцяExternal heart massage
Внутрішньовенне введення лідокаїнуIntravenous administration of lidocaine
Внутрішньовенне введення адреналіну з атропіномIntravenous administration of epinephrine with atropine
Педіатрія
119/1500
Хворий 10-ти років надійшов до клініки зі скаргами на експіраторну задишку, частота дихання - 30/хв. Погіршення стану пов'язує з зміною метеорологічних умов. Хлопчик стоїть на диспансерному обліку впродовж 4-х років з діагнозом бронхіальна астма, 3 ступінь, персистуюча. З чого слід розпочати надання невідкладної допомоги?A 10-year-old patient came to the clinic with complaints of expiratory shortness of breath, respiratory rate - 30/min. The deterioration of the condition is associated with a change in meteorological conditions. The boy is standing at the dispensary for 4 years with a diagnosis of bronchial asthma, grade 3, persistent. Where should emergency care be started?
Другі пологи у повторнороділлі в терміні 36-37 тижнів. Навколоплідні води вилились 8 годин тому, пологова діяльність триває 4 години, регулярна, перейми через 3-4 хвилини по 35 секунд. Передлегла голівка притиснута до входу в малий таз. Роділля поскаржилась на раптовий різкий біль в животі. Пульс - 100/хв., АТ- 110/70-100/70 мм рт.ст. Матка напружена, не розслабляється поза переймами. Серцебиття плода приглушене - 100/хв. Підтікають навколоплідні води, забарвлені кров'ю. Вкажіть найбільш імовірний діагноз:Second birth in repeated labor at 36-37 weeks. Amniotic fluid spilled 8 hours ago, labor lasts 4 hours, regular, takes after 3-4 minutes 35 seconds. The head is pressed against the entrance to the pelvis. The woman in labor complained of sudden pain in the abdomen. Pulse - 110/70/70 mm Hg. The uterus does not relax. The fetal heartbeat is muffled - 100/min. Amniotic fluid is flowing, stained with blood. Specify the most likely diagnosis:
Передчасне відшарування нормально розташованої плацентиPremature detachment of a normally located placenta
Розрив судин пуповиниRupture of umbilical cord vessels
Акушерство і гінекологія
121/1500
Вагітна віком 25 років направлена до стаціонарного відділення для лікування. В анамнезі - 2 самовільних викидня. Під час обстеження виявлено вагітність 14-15 тижнів. Вагінально спостерігається: шийка матки вкорочена, зовнішнє вічко пропускає кінчик пальця. Встановлено діагноз: істміко-цервікальна недостатність. Яка тактика лікаря у цьому разі?A 25-year-old pregnant woman was sent to the inpatient department for treatment. She had 2 spontaneous miscarriages in her history. During the examination, a pregnancy of 14-15 weeks was detected. Vaginal observation: cervix shortened, the outer eye passes the tip of the finger. The diagnosis is established: isthmic-cervical insufficiency. What is the doctor's tactic in this case?
Провести гормональне лікуванняConduct hormonal treatment
Провести амніоцентез та перервати вагітністьPerform amniocentesis and terminate pregnancy
Накласти циркулярний шов на шийку маткиPut a circular suture on the cervix
Ліжковий режим із призначенням седативних препаратівBed bed with the appointment of sedatives
Не чекати виникнення самовільного аборту, увести утеротонічні препаратиDo not wait for spontaneous abortion to occur, introduce uterotonic drugs
Хірургія
122/1500
У хворої 65-ти рокiв при пальпацiї живота в ділянцi пупка i вище пухлина розмiром 13х8 см, помiрно болюча, не змiщується, пульсує. Аускультативно: систолічний шум. Який найбільш імовірний дiагноз?A 65-year-old patient has a 13x8 cm tumor on palpation of the abdomen in the area of the navel and above, moderately painful, does not move, pulsates. Auscultation: systolic murmur. What the most likely diagnosis?
Водопостачання районного центру здійснюється з міжпластового водоносного горизонту з високим вмістом солей кальцію та магнію. Вкажіть найоптимальніший метод обробки води перед подачею в розподільну мережу.The water supply of the district center is carried out from an interlayer aquifer with a high content of calcium and magnesium salts. Specify the most optimal method of water treatment before feeding into the distribution network.
ОпрісненняDesalination
ЗнезараженняDisinfection
ВідстоюванняAdvocacy
ОзонуванняOzonation
Пом'якшенняMitigating
Терапія
124/1500
Жінка 29-ти рокiв звернулася до лікаря зі скаргами на остуду, бiль у горлi, утруднене ковтання та підвищення температури до 38^oC. При фізикальному обстеженні лiкарем на обох мигдаликах виявлено нальоти у виглядi сiрих щiльних плiвок, збiльшення i болючiсть пiдщелепних лiмфатичних вузлiв. Використання якого діагностичного методу буде найбільш доречним наступним кроком?A 29-year-old woman consulted a doctor with complaints of a cold, sore throat, difficulty swallowing, and an increase in temperature up to 38°C. During a physical examination by a doctor on both plaques in the form of gray dense films, enlargement and tenderness of the submandibular lymph nodes were found. The use of which diagnostic method would be the most appropriate next step?
Рентгенографія м'яких тканин шиїX-ray of soft tissues of the neck
Пункцiя підщелепних лiмфовузлiвPuncture of submandibular lymph nodes
Аналіз на гетерофільні антитілаHeterophilic antibody analysis
Експрес-тест для виявлення антигенів стрептококу групи АExpress test for detection of group A streptococcus antigens
Бактерiологiчне дослідження мазку із порожнини носа та горлаBacteriological examination of a smear from the cavity of the nose and throat
Хірургія
125/1500
У хворого 26-ти років має місце зовнішня кровотеча з рваної рани гомілки. З рани постійним потоком витікає кров темно-червоного кольору, загальний обся-г крововтрати приблизно 400 мл. Який метод зупинки кровотечі потрібно використати на догоспітальному етапі?A 26-year-old patient has external bleeding from a torn leg wound. Dark red blood is flowing out of the wound in a constant stream, the total amount of blood loss is approximately 400 Jr. What method of stopping bleeding should be used at the pre-hospital stage?
Пальцеве притиснення стегнової артеріїFinger pressure of the femoral artery
Накладання затискача на сосуд, що кровитьClamping a bleeding vessel
Стискальна пов'язка на рануWound compression bandage
Накладання артеріального джгута на стегноApplying an arterial tourniquet to the thigh
Накладання джгута дистальніше місця кровотечіApplying a tourniquet distal to the bleeding site
Терапія
126/1500
Після перенесеного ішемічного інсу-льту, зумовленого кардіоемболією, хворому з фібриляцією передсердь у якості засобу вторинної профілактики призначають:After an ischemic stroke caused by cardioembolism, a patient with atrial fibrillation is prescribed as a means of secondary prevention:
Ноотропні препаратиNootropic drugs
Оральні антикоагулянтиOral anticoagulants
Аспірин чи клопідогрельAspirin or clopidogrel
beta-адреноблокаториbeta blockers
Антагоністи кальціюCalcium antagonists
Хірургія
127/1500
Жінка 45 років звернулась до лікарні за 12 діб після початку захворювання зі скаргами на біль та припухлість І пальця лівої кисті. Під час обстеження виявлено підвищення температури тіла до 38,9^oС. Нігтьова фаланга І пальця колбоподібно потовщена, синьо-багряного кольору. Під нігтьовою пластинкою та місцями під епідермісом видно гній. Під час пальпації відчуває різкий біль. На Ro-грамі пальця видні деструктивні зміни в кістці нігтьової фаланги. Який вид панарицію у хворої?A 45-year-old woman went to the hospital 12 days after the onset of the disease with complaints of pain and swelling of the I finger of the left hand. During the examination, an increase in body temperature up to 38, 9^C. The nail phalanx is thickened, purple in color. During palpation, there is a sharp pain in the bone of the nail phalanx sick?
ПідшкірянийSubcutaneous
КістковийBone
СуглобовийArticulated
СухожильнийTendino
ШкірянийLeather
Педіатрія
128/1500
Дитина 5-ти років хворіє 2 тижні. Спочатку з'явилися напади кашлю, потім - репризи. Під час кашлю обличчя хворого червоніє, вени шиї набухають. Напади кашлю закінчуються блюванням. На рентгенограмі: підсилення бронхіального малюнка. Аналіз крові: лейкоцити - 16·10^9/л, лімф.- 72%, швидкість осідання еритроцитів - 4 мм/год. Який найбільш імовірний діагноз?A 5-year-old child has been ill for 2 weeks. At first, coughing attacks appeared, then repeated attacks. During coughing, the patient's face turns red, neck veins swell. Coughing attacks end with vomiting. On X-ray: increase in bronchial pattern. Blood count - 16·10^9/l, erythrocyte sedimentation rate - 4 mm/h. What is the most likely diagnosis?
Стороннє тілоForeign body
ПневмоніяPneumonia
Аденовірусна інфекціяAdenovirus infection
КашлюкWhooping cough
Обструктивний бронхітObstructive bronchitis
Терапія
129/1500
Хворий 52-х років скаржився на раптову слабкість і оніміння в лівих кінцівках, утруднення при ходьбі. Об'єктивно відзначалися лівобічні гемігіпестезія і легкий геміпарез. Через 4 години стан хворого нормалізувався, вогнищева симптоматика регресувала, хворий зміг нормально ходити. АТ- 120/80 мм рт.ст. Який найбільш імовірний діагноз?A 52-year-old patient complained of sudden weakness and numbness in the left limbs, difficulty walking. Objectively, left-sided hemipaesthesia and mild hemiparesis were noted. After 4 hours, the condition the patient normalized, the focal symptoms regressed, the patient was able to walk normally. Blood pressure - 120/80 mm Hg. What is the most likely diagnosis?
Хвора 46-ти років надійшла до клініки з приводу відкритого перелому лівого стегна в середній третині, при надходженні оперована - накістковий остеосинтез пластиною. На 4-й день після операції скаржиться на біль в рані, підвищення температури вище 39^oC. Яких заходів нео-бхідно вжити в цьому випадку?A 46-year-old patient came to the clinic with an open fracture of the left hip in the middle third, upon admission she underwent surgery - osseous osteosynthesis with a plate. On the 4th day after the operation complains of pain in the wound, an increase in temperature above 39°C. What measures should be taken in this case?
Призначити антибіотики широкого спектра дії та гормональні препаратиPrescribe broad-spectrum antibiotics and hormonal drugs
Видалити металевий фіксатор, призначити сульфаніламідиRemove metal retainer, prescribe sulfonamides
Внутрішньокістково ввести антибіотики, місцево гіпотерміюIntraosseous inject antibiotics, local hypothermia
Розпустити шви, дренувати рану та призначити антибіотикиDissolve sutures, drain the wound and prescribe antibiotics
Обколоти рану антибіотиками, призначити спазмолітики й анальгетикиSurround the wound with antibiotics, prescribe antispasmodics and analgesics
Терапія
131/1500
У пацієнта віком 35 років хвороба розпочалась бурхливо: з ознобу, підвищення температури тіла до 39^oС, блювання, болю в епігастрії, проносу з водянистими випорожненнями з домішками слизу і зеленкуватим відтінком. За 6 годин до захворювання з'їв сире яйце, смажену картоплю з тушкованим м`ясом, випив сік. Який збудник найімовірніше викликав подібний стан?In a 35-year-old patient, the disease began violently: with chills, an increase in body temperature to 39°C, vomiting, pain in the epigastrium, diarrhea with watery stools mixed with mucus and a greenish hue. 6 hours before the illness, he ate a raw egg, fried potatoes with stewed meat, and drank juice. What pathogen most likely caused this condition?
Кишкова паличкаEscherichia coli
Холерний вібріонVibrio cholerae
ШигелаShigela
СальмонелаSalmonella
КампілобактерCampylobacter
Акушерство і гінекологія
132/1500
Першовагітна з'явилась в жіночу консультацію в 37 тижнів вагітності. Скарг немає. Протягом останніх 2 тижнів набрала 2 кг ваги. Визначаються набряки ніг. АТ - 120/70 мм рт. ст. Білок у сечі - 0,8 г/л. Поставлено діагноз: пре-екламсія легкого ступеня. Яка лікувальна тактика?A first-time pregnant woman came to the antenatal clinic at 37 weeks of pregnancy. There are no complaints. She has gained 2 kg of weight over the past 2 weeks. Swelling of the legs is detected. BP - 120/70 mm Hg. Protein in urine - 0.8 g/l. Diagnosed: mild pre-eclampsia?
Стаціонарне лікуванняInpatient treatment
Кесарський розтинCaesarean section
Пролонгування вагітностіProlongation of pregnancy
Термінове розродженняUrgent delivery
Амбулаторне лікуванняOutpatient treatment
Терапія
133/1500
Шахтар із 15-річним стажем роботи звернувся до лікаря зі скаргами на погіршення стану здоров'я, задишку, кашель, біль у ділянці легень. Про яке захворювання слід думати?A miner with 15 years of work experience went to the doctor with complaints of deteriorating health, shortness of breath, cough, pain in the lung area. What disease should you think about ?
БронхітBronchitis
ГрипFlu
Бронхіальна астмаBronchial asthma
ТуберкульозTuberculosis
ПневмоконіозPneumoconiosis
Педіатрія
134/1500
На 8-му добу життя у новонародженого хлопчика з масою тіла 3500 г підвищується температура тіла та з'являється висип. Напередодні дитина була дуже дратівлива та мала субфебрильну температуру, що поступово підвищувалася. Мати повідомила, що до зазначеного епізоду дитина була здорова та ніяких ліків не приймала. Температура тіла - 38,9^oC, артеріальний тиск - 90/50 мм рт.ст., пульс - 160/хв., частота дихання - 17/хв., SpO_2 - 98% при кімнатному повітрі. При огляді виявлено висип у вигляді пухирів, що легко лопаються. Висип займає 60% поверхні тіла, навкруги рота, але не виявлено на слизових оболонках. Позитивний симптом Нікольського. Який діагноз є найбільш імовірним?On the 8th day of life, a newborn boy with a body weight of 3500 g has a fever and a rash appears. The day before, the child was very irritable and had a low-grade fever, which gradually increased. The mother reported that the child was healthy and did not take any medications. Body temperature - 38.9°C, blood pressure - 90/50 mmHg, pulse - 160/min. - 17/min., 98% in room air. Rash in the form of blisters, around 60% of the body surface, but not found in the mucous membranes. What is the diagnosis most likely?
Проведено дослідження щодо встановлення зв'язку між ожирінням і інфарктом міокарду у медичних сестер. Протягом декількох років реєстрували і порівнювали частоту появи нових випадків інфаркту міокарда в групах та зміни ІМТ. Вкажіть тип епідеміологічного дослідження:A study was conducted to establish the relationship between obesity and myocardial infarction in nurses. For several years, the frequency of new cases of myocardial infarction in groups and changes in BMI were recorded and compared Specify the type of epidemiological study:
Експериментальне дослідженняExperimental study
Когортне дослідженняCohort Study
Поперечне дослідженняCross-sectional study
Дослідження опис серії випадківStudy description of case series
Дослідження випадок-контрольCase-control study
Педіатрія
136/1500
Дівчинка, 7 років, надійшла у відділення зі скаргами на біль у горлі, підвищення температури до 39^oC. Під час огляду стан дитини важкий, шийні лімфатичні вузли до 1,5 см. При пальпації печінка на 3 см, селезінка на 2 см виступає з-під краю реберної дуги. При лабораторному дослідженні крові: еритроцити - 4,0·10^12/л, Hb- 121 г/л, кольоровий показник - 0,9, тромбоцити - 190·10^9/л, лейкоцити - 19·10^9/л, е- 0, п/я- 1, с/я- 0, л- 87, м- 2, швидкість зсідання еритроцитів - 36 мм/год. Що з перерахованого буде найбільш доречним наступним кроком у веденні пацієнта?A 7-year-old girl came to the department with complaints of a sore throat, temperature rise to 39^oC. During the examination, the child's condition is serious, cervical lymph nodes to 1.5 cm. During palpation, the liver is 3 cm, the spleen protrudes 2 cm from the edge of the costal arch. During the laboratory blood test: erythrocytes - 4.0·10^12/l, Hb - 121 g/l, color index - 0.9, platelets - 190·10^9/l, leukocytes - 19·10^9/l, e- 0, p/ya- 1, s/ya- 0, l- 87, m- 2, speed erythrocyte sedimentation rate - 36 mm/h. Which of the following would be the most appropriate next step in the patient's management?
Аналіз крові на виявлення гетерофільних антитілBlood analysis for detection of heterophilic antibodies
Дослідження кісткового мозкуBone marrow research
--
Спостереження протягом 2 тижнівObservation for 2 weeks
Повторний загальний аналіз крові через 1 тижденьRepeat general blood test in 1 week
Терапія
137/1500
Жінка 45 років скаржиться на загальну кволість, задишку, запаморочення. Упродовж року посивіло волосся, почали розшаровуватися нігті, змінився смак. 5 років перебуває на <<Д>> обліку у гінеколога з приводу фіброміоми матки. У крові виявлено: еритроцити - 3,0·10^12/л, Нb - 76 г/л, КП - 0,7, ретикулоцити - 0,7%, тромбоцити - 160·10^9/л , лейкоцити - 5,0 ·10^9/л, еозинофіли - 2%, паличкоядерні - 3%, сегментоядерні - 63%, лімфоцити - 28%, моноцити - 4%, анізо-, мікроцитоз, ШОЕ - 30 мм/год. Яку форму анемії можна припустити?A 45-year-old woman complains of general weakness, shortness of breath, dizziness. Over the course of a year, her hair turned gray, her nails began to peel, her taste changed. She has been on <> for 5 years examination by a gynecologist for uterine fibroids. In the blood, erythrocytes - 3.0·10^12/l, Hb - 76 g/l, CP - 0.7, reticulocytes - 0.7%, platelets - 160·10^ 9/l, leukocytes - 5.0 ·10^9/l, eosinophils - 2%, rod-nuclear - 3%, segmentonuclear - 63%, lymphocytes - 28%, monocytes - 4%, aniso-, microcytosis, ESR - 30 mm /h. What form of anemia can be assumed?
Мінковського-ШоффараMinkowski-Shofar
Аутоімунну гемолітичнуAutoimmune hemolytic
ЗалізодефіцитнуIron deficiency
В_12-дефіцитнуB_12-deficient
ГіпопластичнуHypoplastic
Терапія
138/1500
Чоловік 64-х років надійшов у відділення невідкладної допомоги зі скаргами на набряки нижніх кінцівок, попереку та передньої черевної стінки. В анамнезі хронічний бронхіт з бронхоектазами. При фізикальному обстеженні температура тіла - 37,2^oC, артеріальний тиск - 110/75 мм рт.ст., пульс - 82/хв., частота дихання - 19/хв. При лабораторному дослідженні у загальному аналізі сечі (ЗАС): питома вага - 1025, білок - 9,9 г/л, лейкоцити - 2-3 у полі зору, еритроцити - 1-2 у полі зору, циліндри - немає. Добова протеїнурія - 11,4 г/добу. У біохімічному аналізі крові загальний білок - 52 г/л, альбуміни - 30 г/л, холестерин - 9,6 ммоль/л. Який тип ураження нирок у цього хворого є найбільш імовірним?A 64-year-old man was admitted to the emergency department with complaints of swelling of the lower extremities, lower back, and anterior abdominal wall. He has a history of chronic bronchitis with bronchiectasis. During physical examination body temperature - 37.2°C, blood pressure - 110/75 mmHg, heart rate - 82/min, respiratory rate - 19/min. In the laboratory examination in the general analysis of urine (ZAS): specific gravity - 1025 , protein - 9.9 g/l, leukocytes - 2-3 in the field of vision, erythrocytes - 1-2 in the field of vision. Daily proteinuria - 11.4 g/day. In the biochemical analysis of blood - 52 g/l, albumin - 30 g/l, cholesterol - 9.6 mmol/l. What type of kidney damage is most likely in this patient?
У пацієнтки віком 32 роки, що народжує вперше, почалися інтенсивні потуги з інтервалом 1-2 хв, тривалістю 55-60 с. Об'єктивно спостерігається: настає прорізування голівки плода. Промежина, висота якої 4 см, надмірно випинається. Шкіра промежини бліда, напружена, після припинення потуги із статевої щілини з'явився тоненький струмок крові. Вкажіть подальшу тактику ведення пологів.In a 32-year-old female patient, who is giving birth for the first time, intense efforts began with an interval of 1-2 minutes, lasting 55-60 seconds. Objectively observed: teething occurs the head of the fetus, the height of which is 4 cm. The skin of the perineum is pale, tense, a thin stream of blood appeared from the genital opening.
Чоловік віком 45 років скаржиться на інтенсивний біль за грудиною, що іррадіює у нижню щелепу, виникає у спокої, вночі, кілька разів по 10-15 хвилин. Під час больового нападу на ЕКГ реєструється елевація сегменту SТ у відведеннях V 3-4. Який попередній діагноз?A 45-year-old man complains of intense pain behind the sternum, radiating to the lower jaw, occurring at rest, at night, several times for 10-15 minutes. During of a pain attack, ST segment elevation is recorded in leads V 3-4. What is the preliminary diagnosis?
Інфаркт міокардаMyocardial infarction
Стенокардія ПринцметалаPrinzmetal Angina
Стабільна стенокардія IV функціонального класуStable angina of functional class IV
Стенокардія, що прогресуєProgressive Angina
Стабільна стенокардія II функціонального класуStable angina of functional class II
Акушерство і гінекологія
141/1500
32-річна вагітна у терміні 5-6 тижнів була вакцинована проти грипу неживою вакциною разом з усією родиною. На той момент про вагітність вона не знала. Вагітність бажана. Пацієнтка звернулась до сімейного лікаря для отримання консультації щодо можливого впливу вакцини на розвиток і перебіг вагітності, виникнення вад розвитку у плода. Яку пораду слід надати вагітній?A 32-year-old pregnant woman was vaccinated against influenza with a non-live vaccine in 5-6 weeks along with the whole family. She did not know about the pregnancy at that time. The pregnancy is desirable. The patient turned to the family doctor for advice on the possible impact of the vaccine on the development and course of pregnancy, the occurrence of developmental defects in the fetus. What advice should be given to the pregnant woman?
Запропонувати медикаментозний абортOffer medical abortion
Вакцинація проти грипу є безпечною протягом вагітностіFlu vaccination is safe during pregnancy
Тест на антитіла до вірусу грипуTest for antibodies to influenza virus
УЗД для виявлення вад розвитку плодаUltrasound to detect fetal malformations
Обов'язкова консультація інфекціоніста і генетикаMandatory consultation of an infectious disease specialist and geneticist
Педіатрія
142/1500
У хворого 10 років діагностовано геморагічний васкуліт, шкіряна форма. Одним з основних лікувальних заходів є тривала преднізолонотерапія. З якою метою застосовується ця терапія?A 10-year-old patient was diagnosed with hemorrhagic vasculitis, cutaneous form. One of the main treatment measures is long-term prednisolone therapy. What is the purpose of this therapy?
Підвищення синтезу простагландинівIncreased synthesis of prostaglandins
Зменшення синтезу патологічних імунних комплексівDecreased synthesis of pathological immune complexes
Підвищення синтезу антитілIncreasing synthesis of antibodies
Зменшення синтезу простагландинівDecreased synthesis of prostaglandins
Як замістна терапіяAs replacement therapy
Гігієна
143/1500
У заводській їдальні виникло харчове отруєння, клініка якого вказувала на стафілококову етіологію. Захворіло 15 чоловік. Які матеріали від хворого необхідно надіслати на дослідження у лабораторію аби підтвердити харчове отруєння?Food poisoning occurred in the factory canteen, the clinic of which indicated a staphylococcal etiology. 15 people fell ill. What materials from the patient should be sent to the laboratory for research to confirm food poisoning?
СечуUrine
Кров на гемокультуриBlood for blood cultures
Блювотні маси хворогоVomiting masses of the patient
Кров (клінічний аналіз)Blood (clinical analysis)
СлинуSaliva
Гігієна
144/1500
Працівник оформлюється на роботу, пройшов медичний профілактичний огляд. Визнаний придатним для роботи в умовах даного виробництва. Який вид медичного профілактичного огляду прой-шов працівник?The employee is registered for work, passed a preventive medical examination. Recognized as suitable for work in the conditions of this production. What type of preventive medical examination did the employee undergo?
ПеріодичнийPeriodic
ПлановийPlanned
ПопереднійPrevious
КомплекснийComprehensive
ЦільовийTarget
Терапія
145/1500
Жінка віком 59 років скаржиться на біль та набряклість дрібних суглобів кистей, задуху, слабкість. Хворіє 8 років. Об'єктивно спостерігається: t^o тіла - 37,8^oC, дрібноточкові крововиливи на тулубі та кінцівках, ульнарна девіація кистей. Межі серця зміщені вліво, систолічний шум над верхівкою. Пульс - 96/хв. АТ - 170/100 мм рт. ст. У загальному аналізі крові виявлено: еритроцити - 3,2·10^12/л, Hb - 108 г/л, лейкоцити - 6,8·10^9/л, тромбоцити - 220·10^9, ШОЕ - 48 мм/год., С-реактивний білок +++. У загальному аналізі сечі виявлено: щільність - 1016, білок - 2,8 г/л, лейкоцити - 10-12 в п/з, еритроцити - 2-4 в п/з. Який діагноз найімовірніший?A 59-year-old woman complains of pain and swelling of the small joints of the hands, shortness of breath, weakness. She has been ill for 8 years. Objectively observed: body t^o - 37, 8^oC, small point hemorrhages on the trunk and extremities, ulnar deviation of the hands, systolic murmur above the apex - 170/100 mm Hg ,2·10^12/l, Hb - 108 g/l, leukocytes - 6.8·10^9/l, platelets - 220·10^9, ESR - 48 mm/h, C-reactive protein ++ +. The general analysis of urine revealed: density - 1016, protein - 2.8 g/l, leukocytes - 10-12 in p/z, erythrocytes - 2-4 in p/z. What is the most likely diagnosis?
Після прогулянки на вулиці в ясний сонячний день у дитини 8 місяців з проя-вами рахіту ІІ ступеню, підгострого перебігу, періоду розпалу, з'явилися судоми у вигляді карпопедального спазму. Вони свідчать про наявність у дитини:After a walk on the street on a clear sunny day, an 8-month-old child with manifestations of rickets of the II degree, subacute course, period of exacerbation, developed convulsions in the form of carpopedal spasm. They indicate that the child has:
ГРВІARV
МенінгітуMeningitis
СпазмофіліїSpasmophilia
Черепно-мозкової травмиTraumatic brain injury
ЕпілепсіїEpileptics
Терапія
147/1500
Пацієнт скаржиться на різкий біль у горлі праворуч, що іррадіює в ліве вухо, неможливість відкрити рот, підвищення температури тіла до 40^oС. Об'єктивно спостерігається: виражений тризм жувальних м'язів, асиметрія праворуч, лівий піднебінний мигдалик гіперемований, зміщений до середини ротоглотки, язичок зміщений ліворуч. Неприємний запах із рота. Гіперсалівація. Защелепні лімфатичні вузли праворуч збільшені, болісні під час пальпації. Риноскопічна і отоскопічна картини в нормі. Встановіть діагноз.The patient complains of a sharp pain in the right throat radiating to the left ear, inability to open the mouth, an increase in body temperature up to 40°C. Objectively observed: pronounced masticatory muscles, asymmetry of the left palate, shifted to the middle of the oropharynx, unpleasant smell from the mouth, enlarged lymph nodes on the right, rhinoscopic and otoscopic findings are normal .
Підщелепний лімфаденіт праворучSubmandibular lymphadenitis on the right
Пухлина правого піднебінного мигдаликаTumor of the right palatine tonsil
Терапія
148/1500
Пацієнтка віком 35 років звернулась до лікаря зі скаргами на задишку, погіршення зору ('туман в очах'), подвоєння зображень, косоокість, ністагм, виражену сухість в роті, гугнявість голосу, утруднене ковтання, м'язеву слабкість та здуття живота. З харчового анамнезу відомо, що 20 годин тому споживала м’ясні консерви, рибу домашнього приготування. Який найімовірніший діагноз?A 35-year-old patient turned to the doctor with complaints of shortness of breath, impaired vision ('fog in the eyes'), doubling of images, strabismus, nystagmus, severe dry mouth , hoarseness of voice, difficulty swallowing, muscle weakness and bloating. It is known from the food history that she consumed canned meat and home-cooked fish 20 hours ago. What is the most likely diagnosis?
Харчове отруєння хімічного походженняFood poisoning of chemical origin
Харчовий мікотоксикозFood mycotoxicosis
Харчова токсикоінфекціяFood poisoning
Терапія
149/1500
Чоловік 37 років скаржиться на біль у поперековому і грудному відділах хребта, обмеження рухливості в ньому протягом 5 років. Під час обстеження встановлено діагноз: анкілозуючий спондилоартрит, центральна форма. Носієм якого антигену HLA, най-імовірніше, є пацієнт?A 37-year-old man complains of pain in the lumbar and thoracic regions of the spine, limited mobility in it for 5 years. During the examination, the diagnosis was established: ankylosing spondylitis, central form. Which HLA antigen carrier is the patient most likely?
HLA-DR20HLA-DR20
HLA-B5HLA-B5
HLA-DR4HLA-DR4
HLA-DR8HLA-DR8
HLA-B27HLA-B27
Акушерство і гінекологія
150/1500
Жінка 42-х років прийшла до лікаря на профілактичний прийом. Скарг не має. Загальний стан задовільний. Зріст 162 см, маса тіла 87 кг, ІМТ = 33 кг/м^2. Артеріальний тиск справа - 140/90 мм рт.ст., зліва - 145/85 мм рт.ст., пульс - 72/хв. Аускультативно тони серця приглушені, шуми не прослуховуються. В легенях дихання везикулярне. Нижній край печінки виступає на 1,5-2 см з-під нижнього краю. Набряків не виявлено. Яку оцінку конституціональній будові тіла цієї пацієнтки має надати лікар (за рекомендаціями ВООЗ)?A 42-year-old woman came to the doctor for a preventive appointment. She has no complaints. The general condition is satisfactory. Height 162 cm, body weight 87 kg, BMI = 33 kg /m^2. Arterial pressure - 140/90 mmHg, left - 72/min. Heart sounds are muffled. In the lungs, breathing is vesicular the edge of the liver protrudes 1.5-2 cm from the lower edge. No swelling was detected. What assessment should the doctor give to the constitutional structure of this patient (according to WHO recommendations)?
Нормальна вагаNormal weight
Ожиріння III класуObesity class III
Надлишкова вагаOverweight
Ожиріння I класуObesity class I
Ожиріння II класуObesity class II
Терапія
151/1500
До дерматолога звернулася жінка 38 років зі скаргами на сухість та лущення шкіри. Під час огляду на розгинальних поверхнях ліктьових і колінних суглобів виявлено папульозний висип і дрібне лущення, у ділянці волосяних фолікулів є вузлики воскоподібного кольору, що виступають над поверхнею шкіри. Ці клінічні прояви, найімовірніше, пов'язані з недостатнім надходженням з їжею в організм:A 38-year-old woman consulted a dermatologist with complaints of dryness and peeling of the skin. During the examination, a papular rash and small peeling were found on the extensor surfaces of the elbow and knee joints, in the area of hair follicles are nodules of a waxy color protruding above the surface of the skin. These clinical manifestations are most likely associated with insufficient intake of food into the body:
РибофлавінуRiboflavin
Аскорбінової кислотиAscorbic acid
РетинолуRetinol
ТіамінуThiamine
ПіридоксинуPyridoxine
Терапія
152/1500
Хворий 54-х років звернувся до сімейного лікаря зі скаргами на утруднення дихання, слабкість. Останні два тижні турбував біль та набряк правої нижньої кінцівки. Дані симптоми вперше в житті, раніше на обліку у лікаря не перебував. АТ- 110/70 мм рт.ст., ЧСС- 96/хв. Який діаг-ностичний метод має вирішальне значення?A 54-year-old patient turned to his family doctor with complaints of difficulty breathing, weakness. For the past two weeks, he has been troubled by pain and swelling of the right lower limb. These symptoms are the first time in his life , was not previously registered with a doctor. Blood pressure - 110/70 mmHg, heart rate - 96/min. What diagnostic method is crucial?
Функція зовнішнього диханняFunction of external breathing
Рентгенографія органів грудної кліткиX-ray of chest organs
Пацієнт 66 років скаржиться на значну слабкість. З'явився озноб, коли відчув підвищення температури тіла, біль у суглобах та за ходом м'язів ніг. Об'єктивно спостерігається: фіолетово-синюшна еритема навколо очей та над колінними суглобами. ЧСС - 120/хв., тони серця ослаблені. В крові виявлено: лейкоцити - 12·10^9/л, ШОЕ - 40 мм/год. Поставте діагноз.A 66-year-old patient complains of significant weakness. Chills appeared when he felt an increase in body temperature, pain in the joints and along the course of the muscles of the legs. Objectively observed: purple-blue erythema around the eyes and over the knee joints. Heart rate - 120/min. In the blood: leukocytes - 12·10^9/l. Make a diagnosis.
32-річна жінка скаржиться на нерегулярні менструації з затримками до 2-3 місяців, значну прибавку маси тіла, гірсу-тизм. Заміжня 5 років. Вагітностей не було. При піхвовому дослідженні матка незначно зменшена, з обох сторін визначаю-ться щільні, рухомі яєчники до 4-5 см в діаметрі. Яку патологію можна припустити в даному випадку?A 32-year-old woman complains of irregular menstruation with delays of up to 2-3 months, a significant increase in body weight, hirsutism. She has been married for 5 years. There have been no pregnancies. During the vaginal examination, the uterus is slightly reduced, dense, mobile ovaries up to 4-5 cm in diameter are detected on both sides. What pathology can be assumed in this case?
Туберкульоз придатків маткиTuberculosis of uterine appendages
Гіпоменструальний синдромHypomenstrual syndrome
Гігієна
155/1500
Амортизаційні видатки на відновлення томографу закладено у вартість томографії у розмірі 10% річних від його первісної вартості. Через який термін стане можливим оновлення томографу?Amortization expenses for the restoration of the tomograph are included in the cost of the tomography in the amount of 10% per annum of its initial cost. After what period will it be possible to update the tomograph?
Через 15 роківAfter 15 years
Через 20 роківAfter 20 years
Через 10 роківAfter 10 years
Через 5 роківAfter 5 years
Через 7 роківAfter 7 years
Акушерство і гінекологія
156/1500
На сільській лікарській дільниці зростає захворюваність на рак шийки матки. Прийнято рішення провести обстеження жінок. Який це вид медичних оглядів?The incidence of cervical cancer is increasing at the rural medical district. It was decided to examine women. What kind of medical examinations are these?
ЦільовийTarget
ПопереднійPrevious
СкринінговийScreening
ПоточнийCurrent
КомплекснийComprehensive
Педіатрія
157/1500
Лікар-неонатолог оглядає доношену дитину від ІІ вагітності, ІІ термінових пологів, з масою тіла - 3980 г. З анамнезу відомо, що в пологах виникла первинна слабкість пологової діяльності, використовувалась акушерська допомога. Об'єктивно спостерігається: права рука приведена до тулуба, ротована, відсутні рухи в плечовому та ліктьовому суглобах, спостерігається симптом 'лялькової ручки', кисть в положенні долонного згинання. Годується грудьми, смокче активно. Який найімовірніший діагноз?A neonatologist examines a full-term child from the 2nd pregnancy, the 2nd emergency delivery, with a body weight of 3980 g. From the anamnesis, it is known that during childbirth there was a primary weakness of labor activity , obstetric care was used. Objectively observed: the right arm is brought to the body, there is no movement in the shoulder and elbow joints, the hand is in a palmar flexion position. What is the most likely diagnosis?
Тотальний парез плечового сплетенняTotal paresis of the brachial plexus
Вивих плеча справаRight shoulder dislocation
Парез Дежерін-КлюмпкеParez Dezherin-Klumpke
Парез Дюшена-ЕрбаDuchen-Erb paresis
Перелом плечової кістки справаFracture of humerus on the right
Гігієна
158/1500
В організм людини з атмосферного повітря надходять декілька хімічних речовин. Як називається тип комбінованої дії, при якому сумісний ефект менший від суми ефектів кожної з речовин, що входить у комбінацію, при їх ізольованій дії на організм?Several chemical substances enter the human body from atmospheric air. What is the type of combined action called, in which the combined effect is less than the sum of the effects of each of the substances included in the combination , with their isolated action on the body?
Ізольована діяIsolated Action
ПотенціюванняPotentiation
Поєднана діяCombined Action
АнтагонізмAntagonism
Комплексна діяComplex Action
Педіатрія
159/1500
Огляд у педіатра проходить дівчинка віком 14 років. Об'єктивно виявлено: дівчинка високого зросту, астенічної статури, на шкірі черевної порожнини стрії, голубі склери. Діагностовані сколіотична постава, деформація з боку грудної клітки. Відзначаються гіперрухливість суглобів, довгі пальці та руки. На УЗД серця візуалізується пролапс мітрального клапана. Що з перерахованого є найімовірнішою причиною її високого зросту?A 14-year-old girl is being examined by a pediatrician. Objectively revealed: a girl of tall stature, asthenic physique, stretch marks on the skin of the abdominal cavity, blue sclerae. Scoliotic posture was diagnosed , deformation on the side of the chest. Hypermobility of the joints, long fingers and hands are noted. Mitral valve prolapse is visualized on the ultrasound. Which of the following is the most likely cause of its high growth?
Синдром КлайнфельтераKlinefelter syndrome
Синдром НунанNoonan Syndrome
Синдром Елерса-ДанлосаEhlers-Danlos Syndrome
Синдром МарфанаMarfan syndrome
Синдром ВільямсаWilliams Syndrome
Педіатрія
160/1500
Восьмирічна дитина скаржиться на тривалий вологий кашель з виділенням великої кількості харкотиння гнійного характеру з неприємним запахом, інколи з домішкою крові. Загальний стан тяжкий, температура тіла - 38,7^oC, шкіра бліда, периоральний ціаноз, пальці у вигляді ''барабанних паличок''. Над легенями: ослаблене дихання, різнокаліберні вологі хрипи. Рентгенологічно: ателектатичні ділянки. Яке з обстежень для уточнення діагнозу буде найбільш доцільним?An eight-year-old child complains of a long wet cough with the release of a large amount of sputum of a purulent nature with an unpleasant odor, sometimes with an admixture of blood. The general condition is serious, the body temperature is 38.7 ^oC, perioral cyanosis, fingers in the form of 'drumsticks.
БронхоскопіяBronchoscopy
ЕхокардіографіяEchocardiography
Комп'ютерна томографія з високою розподільною здатністюComputed tomography with high resolution
Рентгенографія ОГКRoentgenography of OGK
--
Акушерство і гінекологія
161/1500
Вагітній у терміні 32 тижні із загрозою передчасних пологів проводиться профілактика респіраторного дистрес-синдрому плода. Який препарат призначено?A 32-week pregnant woman with a risk of premature birth is being treated for fetal respiratory distress syndrome. What drug is prescribed?
ПрогестеронProgesterone
МізопростолMisoprostol
ОкситоцинOxytocin
ДексаметазонDexamethasone
ГініпралGinipral
Терапія
162/1500
У 38-річної жінки після сварки виникло головокружіння, слабкість. В анамнезі вказані: анорексія, втрата ваги, нудота, діарея. Гіпотензія - 60/30 мм рт. ст. у вертикальному положенні. Пульс - 110/хв., малий, ритмічний. Глюкоза крові - 3,3 мМ/л. Гіпонатріємія. Гіперкаліємія. Гіперпігментація шкіри. Екскреція 17-КС та 17-ОКС із сечею знижені. Який попередній діагноз?A 38-year-old woman developed dizziness and weakness after a quarrel. The anamnesis indicated: anorexia, weight loss, nausea, diarrhea. Hypotension - 60/30 mm Hg. Art. Pulse - 110/min. Hyponatremia. 17-KS excretion in urine?
Цукровий діабет, гіпоглікемічний станDiabetes, hypoglycemic state
Вагітність, гіпотонічний станPregnancy, hypotonic state
--
Терапія
163/1500
У чоловіка 20-ти років невдовзі після екстракції зуба розпочалася сильна кровотеча. З анамнезу відомо, що пацієнт хворіє на гемофілію А. При лабораторному дослідженні у загальному аналізі крові: еритроцити - 3,2·10^12/л, Hb- 98 г/л, кольоровий показник - 0,92, лейкоцити - 7,4·10^9/л, тромбоцити - 240·10^9/л, швидкість осідання еритроцитів - 11 мм/год. Час кровотечі за Дюке - 3 хв., час зсідання крові за Лі-Уайтом - 20 хв. Яка фармакотерапія є найбільш ефективною для цього пацієнта?A 20-year-old man started bleeding heavily shortly after tooth extraction. From the anamnesis, it is known that the patient suffers from hemophilia A. During the laboratory examination in the general blood test: erythrocytes - 3.2·10^12/l, Hb - 98 g/l, color indicator - 0.92, leukocytes - 7.4·10^9/l, platelets - 240·10^9/l, sedimentation rate erythrocytes - 11 mm/h. Bleeding time according to Duque - 20 minutes. Which pharmacotherapy is the most effective for this patient?
Рекомбінантний X факторRecombinant X Factor
Рекомбінантний XІ факторRecombinant XI factor
Рекомбінантний VIII факторRecombinant factor VIII
Тромбоцитарна масаPlatelet mass
Еритроцитарна масаErythrocyte mass
Акушерство і гінекологія
164/1500
25-річна жінка пів року тому перенесла аборт, скаржиться на зникнення апетиту, слабкість, артралгію. Через два тижні з'явилися темний колір сечі та жовтяниця, на тлі якої загальний стан продовжує погіршуватися. Припущено вірусний гепатит. Який з маркерів вірусного гепатиту ймовірно буде позитивним у пацієнтки?A 25-year-old woman had an abortion half a year ago, complains of loss of appetite, weakness, arthralgia. Two weeks later, dark urine and jaundice appeared, against the background whose general condition continues to deteriorate. Viral hepatitis is suspected. Which of the markers for viral hepatitis is likely to be positive in the patient?
Anti-HBc IgMAnti-HBc IgM
Anti-CMV IgMAnti-CMV IgM
Anti-HAV IgMAnti-HAV IgM
Anti-HEV IgMAnti-HEV IgM
Anti-HBsAnti-HBs
Терапія
165/1500
Жінка 68 років була госпіталізована зі скаргами на головний біль у потиличній ділянці, виражену задишку в спокої, яка посилюється у положенні лежачи, та сухий кашель. Об'єктивно встановлено: положення ортопное, акроціаноз. Над легенями вислуховується ослаблене везикулярне дихання, середньо- і дрібнопухірчасті вологі хрипи в нижніх відділах. Тони серця ослаблені, ритм галопу. Пульс - 102/хв., АТ - 210/110 мм рт. ст. Печінка виступає на 2 см з-під краю реберної дуги, чутлива під час пальпації. Набряки на гомілках. Які препарати потрібно призначити насамперед?A 68-year-old woman was hospitalized with complaints of a headache in the occipital region, pronounced shortness of breath at rest, which worsens when lying down, and a dry cough. Objectively established : orthopneic position, weak vesicular breathing is heard in the lower parts, heart rate is 102/min 2 cm from the edge of the costal arch, sensitive during palpation. Swelling on the lower legs. What drugs should be prescribed first?
Левофлоксацин в/в, амброксол пероральноLevofloxacin IV, ambroxol oral
Нітрогліцерин, фуросемід в/вNitroglycerin, IV furosemide
Цефтріаксон, лазолван в/вCeftriaxone, Lazolvan IV
Лабеталол в/в, фуросемід в/мLabetalol IV, Furosemide IV
Сульфат магнію в/в, фуросемід в/мMagnesium sulfate IV, furosemide IV
Терапія
166/1500
Пацієнт віком 47 років, захворів 3 дні тому. Скаржиться на підвищення температури до 39^oС, продуктивний кашель з мокротинням жовто-зеленого кольору, задишку, біль у лівій половині грудної клітини. Під час обстеження виявлено: ЧД - 26/хв, укорочення перкуторного звуку та крепітація нижче кута лопатки зліва. SpO2 в межах норми (96%). Який з наведених методів досліджень найбільш інформативний для встановлення остаточного діагнозу?A 47-year-old patient became ill 3 days ago. He complains of an increase in temperature up to 39°C, a productive cough with yellow-green sputum, shortness of breath, pain in the left half of the chest. During the examination, HR - 26/min, and crepitation below the angle of the scapula were found. Which of the following research methods is the most informative for establishing the final diagnosis?
БронхоскопіяBronchoscopy
Клінічний аналіз кровіClinical blood test
Мікробіологічне дослідження мокротинняMicrobiological examination of sputum
Рентгенографія органів грудної кліткиX-ray of chest organs
СпірографіяSpirography
Терапія
167/1500
Пацієнту віком 65 років з приводу гіпертонічної хвороби з супутньою ІХС була призначена фармакотерапія (раміприл, аторвастатин, амлодипін, ацетилсаліцилова кислота, бісопролол). Через 2 тижні він звернувся до лікаря з приводу сухого кашлю. Під час обстеження не було виявлено ознак ГРВІ, ураження бронхо-легеневого апарата. Припущено побічну дію раміприлу. Яким з наведених препаратів можна замінити раміприл у схемі лікування?A 65-year-old patient was prescribed pharmacotherapy (ramipril, atorvastatin, amlodipine, acetylsalicylic acid, bisoprolol) for hypertension with concomitant coronary artery disease. After 2 weeks, he turned to doctor about a dry cough. During the examination, no signs of ARVI were detected. A side effect of ramipril was assumed. Which of the following drugs can be replaced with ramipril in the treatment regimen?
ТорасемідTorasemide
ВалсартанValsartan
НіфедипінNifedipine
НебівололNebivolol
ЕналаприлEnalapril
Терапія
168/1500
Жінка 60-ти років надійшла до відділення невідкладної допомоги зі скаргами на сильний головний біль у потиличній ділянці, нудоту, блювання, запаморочення. При огляді хвора трохи загальмована, обличчя гіперемоване, артеріальний тиск - 220/130 мм рт.ст., пульс - 78/хв. При неврологічному обстеженні парезів не виявлено, рефлекси рівномірно жваві, м'язовий тонус не змінений, чутливість збережена. Менінгеальні симптоми відсутні. Який стан найбільш імовірно розвинувся у хворої?A 60-year-old woman came to the emergency department with complaints of a severe headache in the back of the head, nausea, vomiting, dizziness. On examination, the patient is slightly depressed, her face hyperemic, blood pressure - 220/130 mmHg, pulse - 78/min. No paresis was detected, muscle tone was not changed. Meningeal symptoms are most likely to have developed in the patient?
Транзиторне порушення мозкового кровообігуTransient cerebral circulation disorder
Під час огляду трупа людини, яка померла внаслідок повішення, виявлено: трупні плями під час натискування зникають, відновлюються за 50 сек, трупне заклякання помірно виражене тільки в жувальних м'язах, у м'язах шиї та пальців кисті. Температура тіла - 31,0^oC. Визначте час настання смерті?During examination of the corpse of a person who died as a result of hanging, it was found: corpse spots disappear when pressed, recover in 50 seconds, corpse incantation is moderately expressed only in chewing m in the muscles of the neck and fingers. Body temperature - 31.0°C. Determine the time of death?
1-2 години тому1-2 hours ago
10-18 годин тому10-18 hours ago
16-24 години тому16-24 hours ago
6-7 годин тому6-7 hours ago
8-10 годин тому8-10 hours ago
Гігієна
170/1500
На території із підвищеною ендемічною захворюваністю на зоб сімейний лікар для вторинної профілактики запропонував вживати багаті на йод харчові продукти. Які із наведених продуктів варто вживати за цих умов?In an area with a high endemic incidence of goiter, the family doctor suggested consuming iodine-rich food products for secondary prevention. Which of the following products should be consumed under these conditions?
МорепродуктиSeafood
Молочні продуктиDairy products
Вироби з борошнаFlour products
Овочі та фруктиVegetables and fruits
М'ясні продуктиMeat products
Педіатрія
171/1500
Жінка 28 років скаржиться на загальну слабкість, головний біль, підвищення температури тіла до 37-38^oС, незначний біль у горлі. Хворіє 3-й день. Об'єктивно встановлено: шкіра бліда, губи ціанотичні. Гіперемія ротоглотки з ціанотичним відтінком, набряклі язичок, піднебінні дужки, мигдалики. На поверхні мигдаликів - суцільні щільні білуваті з перламутровим відтінком нальоти, які знімаються шпателем з великим зусиллям, після їх видалення підлегла слизова оболонка кровоточить. Збільшені підщелепні лімфатичні вузли. Набряк шиї. Тахікардія. АТ - 105/65 мм рт. ст. Який найбільш імовірний діагноз?A 28-year-old woman complains of general weakness, a headache, an increase in body temperature to 37-38°C, a slight sore throat. She has been ill for the 3rd day. About objectively established: pale skin, cyanotic lips. Hyperemia of the oropharynx with a cyanotic tinge, swollen tongue, palatal brackets, tonsils - continuous whitish plaques with a pearly tint, which are removed with a spatula with great effort, the underlying mucous membrane bleeds Enlarged lymph nodes. Tachycardia - 105/65 mmHg. What is the most likely diagnosis?
Аденовірусна інфекціяAdenovirus infection
Інфекційний мононуклеозInfectious mononucleosis
Дифтерія ротоглоткиOropharyngeal diphtheria
Гострий лейкозAcute leukemia
АнгінаAngina
Терапія
172/1500
Жінку на четвертий день після приїзду з Індії шпиталізовано до інфекційного відділення зі скаргами на біль у животі, випорожнення рідкої консистенції в невеликій кількості, 7-8 разів за добу, що містять велику кількість склоподібного слизу і кров. Загальний стан задовільний, шкіра бліда, чиста. Живіт м’який, чутливий у ділянці сліпої та висхідної кишок. Який лікарський засіб потрібно призначити жінці?On the fourth day after arriving from India, a woman was hospitalized in the infectious disease department with complaints of abdominal pain, small amounts of liquid stools, 7-8 times a day, containing a large amount of vitreous mucus and blood. The general condition is satisfactory, the skin is soft, sensitive in the area of the cecum and the ascending colon. What medicine should be prescribed to the woman?
ЛоперамідLoperamide
ЦефтріаксонCeftriaxone
ЕритроміцинErythromycin
НіфуроксазидNifuroxazide
МетронідазолMetronidazole
Терапія
173/1500
Пацієнт віком 47 років отримав опік полум’ям верхніх кінцівок та тулуба. Під час шпиталізації спостерігається: стан важкий, свідомість затьмарена, лихоманка, АТ - 80/50 мм рт. ст., пульс - 118/хв. Місцево констатовано опіки III Б ступеня площею 20%. Які лікувальні заходи необхідно провести насамперед?A 47-year-old patient received flame burns on his upper limbs and trunk. During hospitalization, the following conditions were observed: his condition was severe, his consciousness was clouded, fever, blood pressure - 80/50 mm rt., pulse - 118/min. Locally, 20% degree of burns were diagnosed. What treatment measures should be taken first?
Призначити антибактеріальну та дезінтоксикаційну терапіюPrescribe antibacterial and detoxification therapy
Провести первинну хірургічну обробку опікової раниPerform primary surgical treatment of a burn wound
Терапія
174/1500
Чоловікові 38 років. З анамнезу відомо, що він багато років страждає на епілепсію. 3 доби тому переніс стан, який розпочався раптово і супроводжувався злістю. Перебуваючи в цьому стані, пацієнт говорив щось дивне і незрозуміле, поламав меблі, ударив дружину. Тривав цей стан близько години, після чого пацієнт заснув. Потім казав, що не пам’ятає подій цього періоду. Найімовірнішим визначенням цього стану є:The man is 38 years old. It is known from the anamnesis that he has suffered from epilepsy for many years. 3 days ago he suffered a condition that began suddenly and was accompanied by anger. Being in this condition , the patient said something strange and incomprehensible, hit his wife. This state lasted for about an hour, after which the patient said that he did not remember the events of this period. The most likely definition of this state is:
ФугаFugue
ТрансTrans
Амбулаторний автоматизмAmbulatory automatism
Сутінковий розлад свідомостіTwilight disorder of consciousness
Аментивний розлад свідомостіAmentative disorder of consciousness
Хірургія
175/1500
До приймального відділення звернулася хвора 28-ми років зі скаргами на біль в ділянці лівого плечового суглобу, який посилюється при рухах. При огляді: в ділянці ключично-акроміального з'єднання виявляється набряк і сходинкоподібна деформація. При пальпації - болючість, позитивний симптом ''клавіші''. Який попередній діагноз?A 28-year-old female patient came to the emergency department with complaints of pain in the area of the left shoulder joint, which worsens with movement. On examination: in the area of the clavicular-acromial with 'the joint is swollen and stair-like deformation. On palpation - soreness, a positive symptom of 'keys'. What is the preliminary diagnosis?
Перелом акроміального відростка лопаткиFracture of the acromial process of the scapula
Перелом акроміального кінця ключиціFracture of the acromial end of the clavicle
Вивих акроміального кінця ключиціDislocation of the acromial end of the clavicle
Передньо-верхній вивих плечової кісткиAnterior-superior dislocation of the humerus
Перелом головки плечової кісткиFracture of humeral head
Терапія
176/1500
Пацієнтка віком 39 років скаржиться на періодичний головний біль, пульсуючий за характером, завжди зліва, частіше перед менструаціями. Які групи препаратів доцільно призначити насамперед?A 39-year-old patient complains of periodic headache, throbbing in nature, always on the left side, more often before menstruation. What groups of drugs should be prescribed first?
АнтидепресантиAntidepressants
ВалеріануValeriana
ТриптаниTriptans
ДексаметазонDexamethasone
ПарацетамолParacetamol
Акушерство і гінекологія
177/1500
До лікарні шпиталізовано вагітну у терміні гестації 38 тижнів, вагітність друга. Перша вагітність завершилася природніми пологами, народився хлопчик вагою 3600 гр. У жінки було виявлено ВІЛ-позитивний статус під час другої вагітності на 12 тижні гестації. Жінка отримує ААРТ із 22 тижня вагітності. Положення плода поздовжнє, головне передлежання. Передбачувана вага 3200pm200 гр. Серцебиття плода звучне, ритмічне, 122-138/хв. Вірусне навантаження становить 40 вірусних копій/мл. Яка тактика ведення пологів у цьому разі?A pregnant woman was hospitalized at 38 weeks' gestation, second pregnancy. The first pregnancy ended in natural childbirth, a boy weighing 3600 g was born. The woman was found to be HIV-positive During the 12th week of pregnancy, the woman receives AART from the 22nd week of pregnancy. What are the delivery tactics in this case?
Вести пологи консервативноGive birth conservatively
Кесарів розтин на 40 тижнів вагітностіCaesarean section at 40 weeks of pregnancy
Виключити ІІ період пологівExclude II period of childbirth
Провести індукцію пологів на 38 тижні вагітностіInduce labor at 38 weeks of pregnancy
Кесаревів розтин на 38 тижнів вагітностіCaesarean section at 38 weeks of pregnancy
Педіатрія
178/1500
Для вивчення фізичного розвитку дітей та підлітків широко використовують антропометричні дослідження. Виберіть із наведеного фізіометричний метод досліджень:Anthropometric studies are widely used to study the physical development of children and adolescents. Choose a physiometric research method from the following:
Визначення маси тілаDetermination of body weight
Вимірювання зростуHeight measurement
Визначення форми хребтаDefinition of spine shape
Визначення життєвої ємності легеньDetermining the vital capacity of the lungs
Визначення форми грудної кліткиDetermining the shape of the chest
Гігієна
179/1500
Під час медичного огляду робітників металургійного заводу після вимірювання індексу маси тіла (ІМТ) у 20% осіб було вия-влено надлишкову вагу. Вміст яких продуктів необхідно зменшити в раціоні в першу чергу для нормалізації маси тіла у цієї групи осіб?During the medical examination of workers at a metallurgical plant, after measuring the body mass index (BMI), 20% of them were found to be overweight. Which foods should be reduced in the diet primarily to normalize body weight in this group of people?
М'ясних та рибних продуктівMeat and fish products
Хлібобулочних виробівBakery
ОвочівVegetables
Молока та молочних продуктівMilk and milk products
ФруктівFruits
Терапія
180/1500
У хворого 37-ми років, ВІЛ-позитивного, впродовж 2-х тижнів наростала задишка, температура підвищилась до 37,7^oC. Діагностована пневмоцистна пневмонія. Який з препаратів етіотропної терапії показаний у цьому випадку?A 37-year-old HIV-positive patient developed shortness of breath over the course of 2 weeks, and the temperature rose to 37.7°C. Pneumocystis pneumonia was diagnosed. What from drugs of etiotropic therapy indicated in this case?
Ко-тримоксазолCo-trimoxazole
ФлуконазолFluconazole
МетронідазолMetronidazole
АцикловірAcyclovir
ЦефтріаксонCeftriaxone
Гігієна
181/1500
Під час профілактичного огляду населення одного з міст України у 25 % мешканців виявлені наступні симптоми: плямиста емаль зубів, генералізований остеосклероз із кальцифікацією міжхребцевих зв'язок. Яка найімовірніша причина виникнення цих симптомів?During a preventive examination of the population of one of the cities of Ukraine, 25% of the residents were found to have the following symptoms: spotted tooth enamel, generalized osteosclerosis with calcification of the intervertebral ligaments. What is the most likely cause occurrence of these symptoms?
Недостатнє надходження фтору в організм із чаємInsufficient intake of fluoride in the body with tea
Надмірний вміст фтору у водіExcessive fluoride content in water
Недостатній вміст фтору у ґрунті та водіInsufficient fluoride content in soil and water
Надмірний вміст фтору в рослинних продуктахExcessive fluoride content in vegetable products
Недостатній вміст фтору в тваринних продуктахInsufficient fluoride content in animal products
Терапія
182/1500
Хлопчик 10-ти років звернувся в поліклініку зі скаргами на закладеність носу. Відомо, що подібні симптоми виникають періодично (весна, осінь). З анамнезу життя: атопічний дерматит. Батько дитини хворіє на бронхіальну астму. Об'єктивно: обличчя бліде, трохи набрякле. ЧД- 22/хв. Аускультативно: над легенями везикулярне дихання. Риноскопічно: слизова носової порожнини набрякла, бліда. Яке захворювання можна припустити?A 10-year-old boy came to the polyclinic with complaints of nasal congestion. It is known that similar symptoms occur periodically (spring, autumn). Life history: atopic dermatitis The child's father is suffering from bronchial asthma. Objectively, the face is slightly swollen. Auscultation: vesicular breathing. Nasal mucosa is pale.
У хворого 35-ти років, який у зв'язку із хронічним гломерулонефритом 3 останні роки перебуває на гемодіалізі, з'явилися перебої в діяльності серця, гіпотонія, наростаюча слабкість, задишка. На ЕКГ: брадикардія, атріовентрикулярна блокада І ступеня, високі загострені зубці Т. Напередодні було грубе порушення питного та дієтичного режимів. Які біо-хімічні зміни є найбільш імовірною причиною вищевказаної клінічної картини?A 35-year-old patient, who has been on hemodialysis for the last 3 years due to chronic glomerulonephritis, had heart failure, hypotension, increasing weakness, shortness of breath. On the ECG: bradycardia, atrioventricular blockade of the first degree, high sharp T waves. The day before, there was a gross violation of the drinking and dietary regimes. What bio-chemical changes are the most likely cause of the above-mentioned clinical picture?
ГіпокальцієміяHypocalcemia
ГіперкалієміяHyperkalemia
ГіпокалієміяHypokalemia
ГіпернатрієміяHypernatremia
ГіпергідратаціяHyperhydration
Педіатрія
184/1500
У 10-річної дівчинки на тлі підвищеної до 37,5^oC температури тіла спостерігаються помірне збільшення лімфовузлів шиї, сірувато-білі нашарування на мигдаликах у вигляді щільної плівки. Плівка важко знімається, поверхня під нею кровоточить. Набряку шиї немає. Попередній діагноз:A 10-year-old girl, against the background of a body temperature raised to 37.5^oC, has a moderate increase in neck lymph nodes, grayish-white layering on the tonsils in the form of a dense film. The film is difficult to remove, the surface is bleeding. There is no swelling of the neck. Preliminary diagnosis:
Пацієнтка віком 26 років скаржиться на переймоподібний біль в животі, проноси зі значною кількістю слизу і крові, підвищення температури тіла до 37,5-38,0^oС. Об'єктивно спостерігається: блідість шкіри та слизових оболонок, астенічна будова тіла. Під час пальпації виявлена болючість за ходом товстої кишки. В ході колонофіброскопії виявлено: стінка прямої та сигмоподібної кишки набрякла, ерозії, дрібні виразки, в просвіті слиз із кров'ю. Який імовірний діагноз?A 26-year-old patient complains of spasm-like abdominal pain, diarrhea with a significant amount of mucus and blood, an increase in body temperature to 37.5-38.0°C. Objectively observed: pallor of the skin and mucous membranes, tenderness of the colon during palpation, swelling of the rectal and sigmoid wall, small ulcers, mucus in the lumen. What is the probable diagnosis?
У воді річки, нижче спуску стічних вод гірничо-металургійного комбінату в місті водозабору, вміст кадмію перевищує гранично-допустиму концентрацію в 8-10 раз. Які захворювання серед населення, пов'язані з даною речовиною, будуть спостерігатися?In the water of the river, below the discharge of wastewater of the mining and metallurgical plant in the city of water intake, the cadmium content exceeds the maximum permissible concentration by 8-10 times. What are the diseases among the population , related to this substance, will be observed?
Хвороба Ітай-ІтайItai-Itai disease
Хвороба МинаматаMinamata disease
Уровська хворобаUrov's disease
Хвороба ПрасадаPrasad Disease
Хвороба ЮшоYusho's disease
Педіатрія
187/1500
Пацієнтка віком 12 років скаржиться на високу температуру тіла, болі в м'язах, утруднене ковтання їжі. Об'єктивно спостерігається: параорбітальний набряк з рожево-фіолетовим відтінком, під час пальпації біль та зниження тонусу м'язів, капілярити в ділянках подушечок пальців та долонь, розширення меж та приглушеність тонів серця, гепато-спленомегалію. Лабораторно виявлено: збільшення рівня креатиніну в крові та сечі. Який найімовірніший діагноз?A 12-year-old patient complains of high body temperature, muscle pain, difficulty swallowing food. Objectively observed: paraorbital swelling with a pink-purple hue, during palpation, pain and decreased muscle tone, capillaritis in the areas of the pads of the fingers and palms, expansion of the borders and muffled tones of the heart. Laboratory findings: an increase in the level of creatinine in the blood and urine. What is the most likely diagnosis?
Жінка 29 років звернулась до центру планування сім'ї та репродуктивного здоров'я зі скаргами на безпліддя протягом 6 років подружнього життя. Місячні з 14 років по 3-4 дні, кількість виділень незначна, нерегулярні. Об'єктивно встановлено: гірсутизм, галакторея. Базальна температура - монофазна. Яка найбільш імовірна причина жіночого безпліддя?A 29-year-old woman applied to the center of family planning and reproductive health with complaints of infertility during 6 years of married life. Monthly from 14 years to 3-4 days, the amount of discharge is insignificant, objectively established: hirsutism, basal temperature - monophasic. What is the most likely cause of female infertility?
26-річна вагітна жінка доставлена до відділення невідкладної терапії на 36-му тижні вагітності зі скаргами на інтенсивний головний біль у лобній ділянці. При фізикальному обстеженні: артеріальний тиск - 170/90 мм рт.ст., пульс - 85/хв., частота дихання - 15/хв., температура - 36,9^oC, набряки кінцівок. Серцебиття плоду - 159/хв. Під час огляду у жінки розвивається напад генералізованих тоніко-клонічних судом. Який препарат лікар має ввести першочергово?A 26-year-old pregnant woman was brought to the emergency department at the 36th week of pregnancy with complaints of an intense headache in the frontal area. On physical examination: blood pressure - 170/90 mm Hg, pulse - 85/min, temperature - 36.9°C, fetal heart rate - 159/min During the examination, the woman developed a generalized seizure tonic-clonic convulsions. What drug should the doctor administer first?
Магнію сульфатMagnesium sulfate
ДіазепамDiazepam
ЛамотриджинLamotrigine
ФенітоїнPhenytoin
Натрію вальпроатSodium valproate
Терапія
190/1500
У пацієнтки віком 30 років під час обстеження виявлено добову протеїнурію - 2,2 г. Під час біопсії виявлено: зміни стінок капілярів клубочків у вигляді дротяних петель. Про яку морфологічну форму ураження нирок свідчать результати біопсії?During the examination, a 30-year-old patient was found to have daily proteinuria - 2.2 g. During the biopsy, it was found: changes in the walls of the glomerular capillaries in the form of wire loops. About which the morphological form of kidney damage is evidenced by biopsy results?
Пацієнт 18 років хворіє з раннього дитинства. Під час обстеження встановлено: Нb - 110 г/л, еритроцити - 3,9·10^12/л, КП - 0,8, лейкоцити - 6,0·10^9/л; ШOЕ - 30 мм/год. Коагулограма показала: протромбіновий індекс - 95%, ретракція кров'яного згустка - 50%, час згортання крові - за 40 хвилин не відбулось, тривалість кровотечі - 3 хв. Які механізми складають основу патогенезу цього захворювання?An 18-year-old patient has been sick since early childhood. During the examination, the following was found: Hb - 110 g/l, erythrocytes - 3.9·10^12/l, CP - 0.8, leukocytes - 6.0·10^9/l; ESR - 30 mm/h. The coagulogram showed: prothrombin index - 50%, blood clotting time - 40 minutes. occurred, duration of bleeding - 3 minutes. What mechanisms form the basis of the pathogenesis of this disease?
Екзогенний дефіцит залізаExogenous iron deficiency
Наявність специфічних антитіл до ендотеліальних стінок судинPresence of specific antibodies to the endothelial walls of vessels
Недостатність у крові антигемофільного глобуліну АAntihemophilic globulin A deficiency in the blood
Терапія
192/1500
Серед пацієнтів загальної лікарської практики за останні роки відбувається неухильне збільшення кількості осіб похилого та старечого віку. Переважання якого типу патології необхідно чекати в структурі захворюваності населення, яке обслуговується?Among the patients of general medical practice, there has been a steady increase in the number of elderly and senile people in recent years. What type of pathology should be expected to predominate in the morbidity structure of the population being served?
Гострої патологіїAcute pathologies
Неепідемічної патологіїNon-epidemic pathology
Виробничої патологіїProduction pathology
Хронічної патологіїChronic pathology
Інфекційної патологіїInfectious pathology
Терапія
193/1500
Жінка віком 36 років скаржиться на біль у суглобах та м'язах, втрату апетиту, закрепи, швидку втомлюваність та субфебрильну температуру. Об'єктивно виявлено: дисфагія, симетричний артрит та потовщення шкіри на кистях та стопах, атрофія та дрібні виразки на пучках пальців, синдром Рейно, телеангіектазія. Яке захворювання зумовлює таку картину?A 36-year-old woman complains of joint and muscle pain, loss of appetite, constipation, rapid fatigue, and low-grade fever. Objectively found: dysphagia, symmetrical arthritis and thickening of the skin on the hands and feet, atrophy and small ulcers on the bundles of the fingers, Raynaud's syndrome, telangiectasia. What disease causes such a picture?
Жінка 40-ка років звернулася до лікаря зі скаргами на тупий ниючий біль в лівій поперековій ділянці, пітливість, зниження працездатності, субфебрильну температуру. У дитинстві перенесла туберкульоз шийних хребців. Лікувалася, знята з диспансерного обліку. При фізикальному обстеженні нирки не пальпуються, болючість в проекції лівої нирки. При лабораторному дослідженні в загальному аналізі сечі спостерігаються протеїнурія, піурія. Реакція сечі кисла. Яке дослідження найбільш імовірно дозволить уточнити діагноз цієї хворої?A 40-year-old woman consulted a doctor with complaints of dull, aching pain in the left lumbar region, sweating, reduced work capacity, low-grade fever. She suffered from tuberculosis of the cervical vertebrae as a child She was treated, removed from the medical records. The kidneys are not palpable. In the laboratory examination, proteinuria, pyuria are most likely to clarify the diagnosis of this patient.
Ультразвукове дослідження нирокUltrasound study of kidneys
Полімеразна ланцюгова реакція (ПЛР) сечіPolymerase chain reaction (PCR) of urine
Екскреторна урографіяExcretory urography
ЦистографіяCystography
Проба Манту та повторний візит через 2 дніMantoux test and return visit in 2 days
Гігієна
195/1500
Під час виробництва коксу концентрація пилу в повітрі робочої зони впродовж багатьох років перевищує ГДК у 4-8 разів. Розвиток якого захворювання є найімовірнішим у робітників цього виробництва?During the production of coke, the concentration of dust in the air of the working area for many years exceeds the MPC by 4-8 times. The development of which disease is most likely among the workers of this production?
АнтракозAnthracosis
СидерозSiderosis
БісинозBisinosis
АзбестозAsbestosis
СилікозSilicosis
Терапія
196/1500
Дівчина 18 років скаржиться на напади чхання, переважно вранці, 15-20 разів поспіль, свербіння в носі, глотці, численні водянисті виділення з носа, нежить, біль в очах. Який із тестів є найінформативнішим для постановки діагнозу?An 18-year-old girl complains of sneezing attacks, mostly in the morning, 15-20 times in a row, itching in the nose, throat, numerous watery discharge from the nose, runny nose, pain in eyes. Which of the tests is the most informative for making a diagnosis?
Прик-тестPrick test
Внутрішньошкірна пробаIntradermal test
Крапельна пробаDrop sample
Провокаційна пробаProvocative test
Загальний IgETotal IgE
Акушерство і гінекологія
197/1500
Пацієнтка віком 30 років звернулася до лікаря зі скаргами на відсутність місячних впродовж двох років після других пологів. З анамнезу відомо, що пологи ускладнилися масивною кровотечею. Після пологів пацієнтка почала відзначати випадіння волосся, втрату ваги. Об'єктивно спостерігається: жінка астенічна, зовнішні статеві органи гіпопластичні, шийка матки циліндричної форми, тіло матки маленьке, безболісне, додатки матки не визначаються. Який діагноз найімовірніший?A 30-year-old patient turned to the doctor complaining of the absence of menstruation for two years after the second childbirth. From the anamnesis, it is known that childbirth was complicated by massive bleeding. After childbirth, the patient began note hair loss, weight loss. Objectively observed: the woman is asthenic, the external genitalia are cylindrical, the uterine body is small, painless, the uterine appendages are not identified. What is the most likely diagnosis?
Пацієнтка віком 26 років, яка хворіє на ревматоїдний артрит впродовж 8 місяців, має набряки ліктьових, променево-зап'ясткових, колінних та гомілковостопних суглобів, ревматоїдні вузлики в ділянці ліктьових суглобів. У загальному аналізі крові спостерігається: ШОЕ - 57 мм/год., С-реактивний білок (+++). На рентгенограмах суглобів визначається виражений остеопороз. Що є базисною терапією в такій ситуації?A 26-year-old female patient, who has been suffering from rheumatoid arthritis for 8 months, has swelling of the elbow, wrist, knee and ankle joints, rheumatoid nodules in the area elbow joints. In the general blood test: ESR - 57 mm/h, C-reactive protein is determined on X-rays of the joints. What is the basic therapy in such a situation?
Диклофенак натріюDiclofenac sodium
МетилпреднізолонMethylprednisolone
МетотрексатMethotrexate
МелоксикамMeloxicam
ІнфліксимабInfliximab
Педіатрія
199/1500
До лікаря звернулися діти школи-інтернату за 4-5 годин після обіду із скаргами на нудоту, блювоту, холодний піт і невеликий біль в епігастрії. На обід діти їли картопляний суп на м'ясному бульйоні, млинці з м'ясом, компот із сухофрук-тів. Млинці начинялися вареним м'ясом, яке подрібнювалось на м'ясорубці працівником їдальні, хворим панарицієм, і термічній обробці не піддавались. Який найімовірніший діагноз у цьому разі?The children of the boarding school turned to the doctor 4-5 hours after lunch with complaints of nausea, vomiting, cold sweat and a slight pain in the epigastrium. For lunch, the children ate potato soup on meat broth, pancakes with dried fruits. The pancakes were filled with boiled meat, which was ground on a meat grinder by a canteen worker, and were not subjected to heat treatment. What is the most likely diagnosis in this if so?
Пацієнт скаржиться на запаморочення, блювання. Блювотні маси темного кольору. З анамнезу відомо, що пацієнт часто вживає алкоголь. Під час ЕФГС виявлено: вміст шлунку схожий на 'кавову гущу', в ділянці кардії чотири продольні тріщини слизової оболонки, з яких потрапляє кров у незначній кількості. Який найімовірніший діагноз?The patient complains of dizziness, vomiting. Vomiting masses are dark in color. It is known from the anamnesis that the patient often drinks alcohol. During EFGS it was found: the contents of the stomach are similar to 'coffee thick', in the area of the cardia there are four longitudinal cracks of the mucous membrane, from which blood enters in small quantities. What is the most likely diagnosis?
Кровотеча з варикозно розширених вен шлункаBleeding from varicose veins of the stomach
Терапія
201/1500
Чоловік 61 року звернувся до дільничного лікаря зі скаргами на напади стиснення за грудиною, які виникають під час ходьби до 200 м та зникають, якщо зупинитися. Уважає себе хворим близько року. Межі серця в нормі, тони помірно приглушені, ЧСС=Ps=76/хв., АТ - 130/80 мм рт. ст. Який найбільш імовириний діагноз у хворого?A 61-year-old man turned to the district doctor with complaints of chest compression attacks that occur while walking up to 200 m and disappear when he stops. He considers himself ill year. Heart rates are normal, heart rate=76/min., BP - 130/80 mmHg. What is the most likely diagnosis for the patient?
ІХС: Інфаркт міокарда без елевації сегменту STCHD: Myocardial infarction without ST segment elevation
Тривожний розладAnxiety disorder
ІХС: Нестабільна стенокардіяIHD: Unstable angina
Хронічне обструктивне захворювання легеньChronic obstructive pulmonary disease
ІХС: Стабільна стенокардіяCHD: Stable angina
Терапія
202/1500
У вогнищах облисіння на голові волосся обламане на рівні 4-6 мм від поверхні шкіри, відзначається гіперемія, мукоподібне лущення. Імовірний попередній діагноз?In foci of baldness on the head, the hair is broken off at the level of 4-6 mm from the surface of the skin, there is hyperemia, muco-like desquamation. Possible preliminary diagnosis?
Гніздова плішивістьNest baldness
Себорейне облисінняSeborrheic alopecia
Вторинний сифілісSecondary syphilis
Мікроспорія волосистої частини головиMicrosporia of scalp
Головний лікар ЦПМСД має надати звіт щодо забезпеченості населення лікарями. За допомогою якого коефіціє-нту він може розрахувати даний показник?The chief physician of the CPMSD must provide a report on the provision of doctors to the population. By which coefficient can he calculate this indicator?
Спеціальної інтенсивностіSpecial intensity
ІнтенсивностіIntensities
СпіввідношенняRatio
ЕкстенсивностіExtensives
НаочностіVisibility
Гігієна
204/1500
Під час проведення комплексної оцінки мікроклімату приміщень необхідно визначити радіаційну температуру. Який прилад потрібно застосовувати?During the comprehensive assessment of the microclimate of the premises, it is necessary to determine the radiation temperature. What device should be used?
Спиртовий термометрAlcohol thermometer
Кульовий термометрBall thermometer
Ртутний термометрMercury thermometer
ТермографThermograph
КататермометрCatathermometer
Педіатрія
205/1500
Під час профілактичного медичного огляду у одного з учнів технічного ліцею виявлені ознаки хейлозу, який проявляється в мацерації епітелію в місці змикання губ. Губи яскраво-червоного кольору з одиничними вертикально розташованими тріщинами, які вкриті кірочками червоно-бурого кольору. Дані клінічні прояви, найімовірніше, пов'язані з недостатнім надходженням з їжею в організм:During a preventive medical examination, one of the students of the technical lyceum showed signs of cheilosis, which manifests itself in the maceration of the epithelium in the place where the lips close. The lips are bright red with single vertical located cracks, which are covered with red-brown crusts. These clinical manifestations are most likely associated with insufficient intake of food into the body:
КальциферолівCalciferol
РибофлавінуRiboflavin
ТіамінуThiamine
РетинолуRetinol
Аскорбінової кислотиAscorbic acid
Педіатрія
206/1500
У шестирічної дитини, яка хворіє на екзантемну інфекцію, на 8-й день захворювання повторно підвищилася температура тіла до 39,6^oС, з'явився головний біль, блювання, спостерігалася хитка хода, скандована мова. Під час огляду виявлено залишкові елементи висипу (кірочки), атаксію, дискоординацію рухів, тремор кінцівок, горизонтальний ністагм, нестійкість у позі Ромберга. Вкажіть препарат вибору для лікування ускладення, що виникло у дитини.A six-year-old child suffering from an exanthem infection, on the 8th day of the disease, the body temperature rose again to 39.6^oС, a headache appeared, vomiting, wobbly gait, slurred speech were observed. During the examination, residual elements of the rash (crusts), ataxia, limb tremor, instability in Romberg's position were found. Specify the drug of choice for the treatment of the child's condition.
Чоловік 23 років звернувся до поліклініки зі скаргами на головний біль, погіршення зору, задишку, одутлість обличчя, загальну слабкість, зменшення кількості виділеної за добу сечі та забарвлення її типу <<м'ясних зливів>>. Три тижні тому переніс ангіну. Об'єктивно встановлено: помірна блідість, обличчя одутле, повіки набряклі; серцева діяльність ритмічна, І тон на верхівці ослаблений, систолічний шум. Пульс - 66/хв. АТ - 175/105 мм рт. ст. Симптом Пастернацького слабко позитивний з обох боків. У сечі виявлено: білок - 1,48 г/л, еритроцити - 35-40 в п/з, змінені; лейкоцити - 8-10 в п/з. Який діагноз є найімовірнішим?A 23-year-old man came to the clinic with complaints of headache, visual impairment, shortness of breath, puffiness of the face, general weakness, decrease in the amount of urine released per day and the color of its type Three weeks ago, he suffered from angina: moderate pallor, swollen eyelids, rhythmic heart rate, systolic heart rate - 66/min. 175/105 mm Hg, weakly positive on both sides: protein - 1.48 g/l, erythrocytes - 8-10 g/l . What is the most likely diagnosis?
Хворий скаржиться на біль в горлі зліва, біль у лівому вусі, підвищення t^o до 39^oC, гугнявість голосу. Хворіє 5 днів. Виражений тризм, підвищена салівація. Голова нахилена до лівого плеча. Припухлість, гіперемія, інфільтрація лівої половини м'якого піднебіння. Защелепні лімфовузли зліва різко болючі при пальпації. Отоскопічна картина в нормі. Який найбільш імовірний діагноз?The patient complains of a sore throat on the left, pain in the left ear, an increase in t^o to 39^oC, hoarseness of the voice. He has been sick for 5 days. Pronounced trismus, increased salivation. The head is tilted to the left shoulder. Swelling, infiltration of the left half of the soft palate. The otoscopic picture is normal?
Паратонзиліт зліваLeft paratonsillitis
Заглотковий абсцесPharyngeal abscess
Парафарингеальна флегмонаParapharyngeal phlegmon
Флегмона шиї зліваPhlegmon of the neck on the left
Жінка 42-х років звернулася до лікаря зі скаргами на м'язову слабкість у верхніх і нижніх кінцівках. Об'єктивно: у періорбітальних ділянках еритема з бузковим відтінком, макульозна еритема на розгинальних поверхнях пальців. В аналізі крові вия-влено суттєве підвищення рівня креатинфосфокінази. Яким є імовірний діа-гноз?A 42-year-old woman consulted a doctor with complaints of muscle weakness in the upper and lower limbs. Objectively: in the periorbital areas erythema with a lilac tint, macular erythema on the extensor surfaces of the fingers. A significant increase in the level of creatine phosphokinase was detected in the blood test. What is the probable diagnosis?
Аналіз захворюваності жителів села, розташованого поблизу хімічного підприємства, показав, що протягом останніх років помітно зросла кількість захворювань на подагру і рак стравоходу. З яким забруднювачем навколишнього середовища можна пов'язати таку динаміку?An analysis of the morbidity of residents of a village located near a chemical plant showed that in recent years the number of diseases such as gout and esophageal cancer has increased significantly. What environmental pollutant can be associated with 'do such a dynamic?
НікелемNickel
РтуттюMercury
МарганцемManganese
МолібденомMolybdenum
СтронціємStrontium
Терапія
211/1500
Пацієнт віком 30 років звернувся із скаргами на сильний нападоподібний біль у попереку справа, що віддає у пахову ділянку і внутрішню поверхню правого стегна, нудоту, здуття живота, блювання, порушення сечовиділення, що супроводжується різями і домішками крові у сечі. Об'єктивно спостерігається: пацієнт неспокійний, змінює положення в ліжку. З боку органів дихання і серцево-судинної системи патології не виявлено. Під час пальпації живота відмічається здуття, напруження м'язів і болючість у правому боці відповідно проекції правої нирки і за ходом правого сечоводу. Печінка і селезінка не збільшені. Симптоми запалення жовчного міхура негативні. Позитивний симптом Пастернацького справа. Який найімовірніший діагноз?A 30-year-old patient complained of severe attack-like pain in the lower back on the right, radiating to the inguinal area and the inner surface of the right thigh, nausea, abdominal distension, vomiting, violation of urination, accompanied by blood in the urine. Objectively observed: the patient changes the position in bed. On the part of the respiratory system and the cardiovascular system, bloating and muscle tension are noted pain in the right side of the right kidney and behind the right ureter. Symptoms of gallbladder inflammation are negative. What is the most likely diagnosis?
ГломерулонефритGlomerulonephritis
Туберкульоз ниркиKidney tuberculosis
Сечокам'яна хворобаUrolithiasis
Жовчокам'яна хворобаCholelithiasis
Пухлина ниркиKidney tumor
Педіатрія
212/1500
У дівчинки 8 місяців під час огляду виявлений гучний інтенсивний систолічний шум над усією серцевою ділянкою з епіцентром у ІІІ-ІV міжребер'ї зліва від груднини та систолічне тремтіння; ІІ тон над легеневою артерією посиленої гучності. АТ - 110/70 мм рт. ст. Скарг немає. Під час рентгенологічного дослідження розміри і форма серцевої тіні не змінені. Який найімовірніший діагноз?In an 8-month-old girl, during the examination, a loud intense systolic murmur was detected over the entire cardiac region with an epicenter in the III-IV intercostal space to the left of the sternum and systolic tremor; II the tone over the pulmonary artery is 110/70 mmHg. No changes in the size and shape of the cardiac shadow?
Відкрита артеріальна протокаOpen ductus arteriosus
Дефект міжшлуночкової перетинкиDefect of interventricular membrane
Коарктація аортиCoarctation of the aorta
Стеноз гирла легеневої артеріїStenosis of the mouth of the pulmonary artery
Терапія
213/1500
Чоловік 50 років скаржиться на наявність висипу на шкірі тулуба без суб'єктивних відчуттів, хворіє 3 місяці, не лікувався, висип прогресує. Об'єктивно встановлено: на шкірі тулуба дисемінована висипка у вигляді червоних негострозапальних лентикулярних та нумулярних папул із чіткими границями, укритих сріблястими лусочками, які легко знімаються при пошкрябуванні предметним склом, інтенсивніше пошкрябування призводить до появи крапкової кровотечі. Який імовірний діагноз?A 50-year-old man complains about the presence of a rash on the skin of the trunk without subjective sensations, he has been sick for 3 months, was not treated, the rash is progressing. Objectively established: on the skin body, a disseminated rash in the form of red non-acute inflammatory lenticular and nummular papules with clear borders, covered with silvery scales, which are easily removed when scraped with a glass slide, more intensive scraping leads to the appearance of spot bleeding. What is the probable diagnosis?
ДерматитDermatitis
ЕкземаEczema
Рожевий пітиріазPityriasis rosea
Псоріаз звичайнийPsoriasis usual
Червоний плоский лишайRed lichen planus
Акушерство і гінекологія
214/1500
Жінці з терміном вагітності 34 тижні проведена доплерометрія пуповинного кровообігу - встановлений реверсний діа-столічний компонент. Об'єктивно: висота дна матки над лоном - 27 см, голівка плода рухома над входом в малий таз. Серцебиття плода - 136/хв. Піхвове дослідження: шийка матки закрита, довжиною 3 см. Якою має бути тактика акушера?A woman with a gestation period of 34 weeks underwent dopplerometry of the umbilical cord blood circulation - a reverse diastolic component was established. Objectively: the height of the bottom of the uterus above the womb is 27 cm, the head of the fetus moving over the entrance to the pelvis. Fetal heartbeat - 136/min. Vaginal examination: the cervix is closed, 3 cm long. What should be the obstetrician's tactics?
Біофізичний профіль плодаBiophysical profile of the fetus
Повторна доплерометрія наступної добиRepeated dopplerometry next day
Негайний кесарів розтинImmediate C-section
Гігієна
215/1500
На підприємстві з виробництва синтетичних миючих речовин збільшилися випадки захворювання ринітами, вазоринітами, фарингітами, дерматитами. У повітрі робочої зони присутній пил миючих речовин. Фахівці вважають, що клінічні прояви й зростання захворюваності зумовлене впливом пилу. Які особливості дії пилу полягають в основі патогенезу?At the enterprise for the production of synthetic detergents, cases of rhinitis, vasorinitis, pharyngitis, and dermatitis have increased. There is dust from detergents in the air of the working area. Experts believe that the clinical manifestations and the increase in morbidity is due to the influence of dust. What features of the action of dust are the basis of pathogenesis?
Фіброгенна діяFibrogenic action
Загальнотоксична діяGeneral toxic action
Алергічна діяAllergic action
Канцерогенна діяCarcinogenic action
Подразнююча діяAnnoying action
Терапія
216/1500
У пацієнта, що хворіє на епілепсію, після самостійного припинення вживання протиепілептичних медикаментів раптово виник епілептичний статус генералізованих судомних нападів. Вкажіть препарати першої лінії для лікування цього стану.A patient with epilepsy suddenly developed status epilepticus with generalized seizures after self-discontinuation of antiepileptic drugs. Specify the first-line drugs for the treatment of this condition.
Вагітна 22-х років звернулась до жіночої консультації зі скаргами на кровотечу зі статевих шляхів, що розпочалася 2 дні тому, сильну втому та запаморочення. Термін вагітності 13 тижнів. За день вона використала 6 гігієнічних прокладок. При фізикальному обстеженні: артеріальний тиск - 90/60 мм рт.ст., температура - 37,8^oC, пульс - 125/хв., частота дихання - 15/хв. Розпочата інфузійна терапія. При вагінальному дослідженні: кров у піхві, шийка матки згладжена та розкрита. На УЗД плідне яйце в порожнині матки, серцебиття і рухи плода не визначаються. Який з наступних кроків у веденні пацієнтки буде найбільш доречним?A 22-year-old pregnant woman came to the women's consultation with complaints of bleeding from the genital tract, which began 2 days ago, severe fatigue and dizziness. The pregnancy period is 13 weeks. During the day, she used 6 sanitary napkins.During physical examination: blood pressure - 90/60 mm Hg, temperature - 37.8°C, pulse - 15/min. Infusion therapy was started. During vaginal examination: blood in the vagina, the cervix is smoothed and opened. On ultrasound, the fetal egg in the uterine cavity, the heartbeat and the movements of the fetus are not determined. Which of the following steps in the management of the patient will be most appropriate?
Дилатація та кюретаж маткиDilatation and curettage of the uterus
АнтибіотикотерапіяAntibiotic therapy
Постільний режим та анальгетикиBed rest and analgesics
МетотрексатMethotrexate
Терапія
218/1500
На репрезентативній сукупності хворих на грип проведено вивчення зв'язку між температурою тіла і частотою пульсу. Розрахований коефіцієнт кореляції дорівнює +0,5. Яка характеристика сили і направленості зв'язку між ознаками, що вивчаються є найбільш точною?The relationship between body temperature and pulse rate was studied on a representative population of flu patients. The calculated correlation coefficient is +0.5. What is the characteristic of the strength and directionality of 'is the connection between the signs being studied the most accurate?
--
Середній прямийAverage straight
Середній зворотнійAverage inverse
Слабкий прямийWeak direct
Сильний прямийStrong straight
Акушерство і гінекологія
219/1500
Пацієнтка віком 38 років, друга вагітність терміном 23-24 тижні, хворіє на цукровий діабет 2 типу, з приводу чого отримує метформін в добовій дозі 2500 мг. Лабораторно виявлено: глікований гемоглобін (HbA_1c) - 7,2 %. Яка подальша тактика лікування цієї пацієнтки?38-year-old female patient, second pregnancy at 23-24 weeks, suffers from type 2 diabetes, for which she receives metformin in a daily dose of 2500 mg. Laboratory revealed : glycated hemoglobin (HbA_1c) - 7.2%. What is the further treatment strategy for this patient?
Відмінити метформін і перевести на препарати сульфанілсечовиниCancel metformin and switch to sulfonylureas
Збільшити дозу метформіну до 3000 мг/добуIncrease metformin dose to 3000 mg/day
Призначити інсулінотерапію та відмінити метформінPrescribe insulin therapy and cancel metformin
Залишити лікування без змінLeave treatment unchanged
Додати до метформіну аналоги ГПП-1Add GLP-1 analogs to metformin
Гігієна
220/1500
У структурі населення N-ського регіону питома вага осіб віком від 0 до 14 років склала 31%, а питома вага осіб віком 50 років і старше склала 20%. Який склад населення найбільш влучно характеризує таку демографічну ситуацію?In the population structure of the N region, the share of people aged 0 to 14 was 31%, and the share of people aged 50 and over was 20%. What composition of the population most aptly characterizes such a demographic situation?
Міграція населенняPopulation migration
Стаціонарний склад населенняStationary composition of the population
Еміграція населенняPopulation emigration
Регресивний склад населенняRegressive composition of the population
Прогресивний склад населенняProgressive composition of the population
Терапія
221/1500
Пацієнт віком 35 років четверту добу знаходиться у відділенні інтенсивної терапії після перенесеної кровотечі та геморагічного шоку. Добовий діурез - 50 мл. Другу добу відзначає блювання. ЦВТ - 159 мм вод. ст., під час аускультації в легенях останні 3 години відзначаються поодинокі вологі хрипи, ЧД - 32/хв. За результатами лабораторного аналізу крові виявлено: залишковий азот - 82 ммоль/л, К^+ - 7,1 ммоль/л, Cl^- - 78 ммоль/л, Na^+ - 130 ммоль/л, Ht - 0,32, Hb - 100г/л, креатинін крові - 0,9 ммоль/л. Яке ускладнення найімовірніше розвинулося у пацієнта?A 35-year-old patient is in the intensive care unit for the fourth day after suffering bleeding and hemorrhagic shock. Daily diuresis - 50 ml. Vomiting on the second day. CVT - 159 mm during auscultation in the lungs for the last 3 hours, single wet wheezes are noted, BH - 32/min. According to the results of the blood analysis: residual nitrogen - 82 mmol/l, K^+ - 7.1 mmol/l, Cl^- - 78 mmol/l, Na^+ - 130 mmol/l, Ht - 0.32, Hb - 100 g/l, blood creatinine - 0.9 mmol/l. What complication most likely developed in the patient?
Дегідратація у зв'язку з гіповолемієюDehydration due to hypovolemia
Гостра ниркова недостатністьAcute renal failure
Акушерство і гінекологія
222/1500
На 5-й день після пологів породілля скаржиться на біль в лівій молочній залозі, підвищення температури тіла до 38,1^oC. При огляді: молочна залоза збільшена, болюча при пальпації, є тріщини та набряк соска, гіперемія верхньо-зовнішнього квадранту залози. Яка профілактика ускладнення, що виникло?On the 5th day after childbirth, a woman in labor complains of pain in the left mammary gland, an increase in body temperature to 38.1^oC. On examination: the mammary gland is enlarged, painful on palpation, there are cracks and swelling of the nipple, hyperemia of the upper-external quadrant of the gland. What is the prevention of the complication that has arisen?
Годування немовляти по годинахFeeding the baby by the hour
Годування дитини на вимогу, зціджування залишкового молока, попередження тріщин сосківFeeding a child on demand, expressing residual milk, preventing cracked nipples
Припинення годування дитини при появі тріщинStop feeding the baby when cracks appear
Постійне зціджування молочних залозConstant breast pumping
Годування не довше 10 хвилин через накладкуFeeding no longer than 10 minutes through the pad
Педіатрія
223/1500
Під час скринінг-дослідження у двотижневого новонародженого було виявлено фенілкетонурію. Яке лікування необхідно призначити дитині для запобігання тяжким ускладненням у майбутньому?During the screening examination, phenylketonuria was detected in a two-week-old newborn. What treatment should be given to the child to prevent serious complications in the future?
Гормональну терапіюHormone therapy
Сонячні ванниSunbaths
Спеціальну дієтуSpecial diet
АнтибіотикотерапіюAntibiotic therapy
ВітамінотерапіюVitamin therapy
Педіатрія
224/1500
Дитина 3-х місяців доставлена в лікарню зі скаргами на мляве смоктання та появу задишки, яка посилюється під час годування, неспокій. Симптоми з'явилися протягом останніх годин. Шкіра бліда, з попелястим відтінком, кінцівки холодні. Аксилярна температура - 36,8^oC. Частота дихання - 66/хв. Частоту серцевих скорочень підрахувати не вдає-ться. Пульс слабкий. Печінка пальпується на 4 см нижче реберної дуги. На ЕКГ: ЧСС - 230/хв., ритм правильний, зубець Р на Т, комплекси тривалістю 0,08 с. Який препарат вибору для надання невідкладної допомоги даній дитині?A 3-month-old baby was brought to the hospital with complaints of sluggish sucking and shortness of breath, which worsens during feeding, restlessness. Symptoms appeared during the last hours. The skin is ashen, the axillary temperature is 66/min. The pulse is weak at 4 cm below the costal arch. Heart rate - 230/min., rhythm is correct, wave P on T, complexes lasting 0.08 s. What is the drug of choice for providing emergency care to this child?
ПропранололPropranolol
ДигоксинDigoxin
АденозинAdenosine
ВерапамілVerapamil
ЛідокаїнLidocaine
Терапія
225/1500
У хлопця 18 років за 2 тижня після ангіни з'явилися набряки під очима, на гомілках, підвищився артеріальний тиск, зменшилась кількість сечі і змінився її колір. Загальний аналіз сечі виявив: білок - 1,9 г/л, еритроцити - покривають усе поле зору, лейкоцити - 10-12 у полі зору. Для якого захворювання характерні такі симптоми?2 weeks after angina, an 18-year-old boy developed swelling under his eyes, on his lower legs, increased blood pressure, decreased the amount of urine and changed its color. General analysis urine showed: protein - 1.9 g/l, erythrocytes - covering the entire field of vision, leukocytes - 10-12 in the field of vision. What disease is characterized by such symptoms?
Роділля 28-ми років доставлена до пологового будинку з бурхливою пологовою діяльністю. Пологи перші. Розміри таза: 23-25-28-18 см. Ознака Генкеля-Вастена позитивна. Роділля збуджена, живіт напружений, болісний в нижніх відділах. Контракційне кільце на рівні пупка, розташоване косо. Голівка плода притиснута до входу у малий таз. Серцебиття плоду - 140/хв. Яке ускладнення виникло у роділлі?A 28-year-old woman in labor was brought to the maternity hospital with violent labor. First delivery. Pelvic dimensions: 23-25-28-18 cm. Henkel-Wasten's sign Labor is excited, the abdomen is painful. The fetal head is pressed against the pelvic inlet.
Дискоординація пологової діяльностіDiscoordination of labor
Розпочатий розрив маткиBeginning uterine rupture
Педіатрія
227/1500
У двомісячної дитини спостерігаються судоми, рецидивуючі вірусно-бактеріальні інфекції верхніх дихальних шляхів. Виявлено деформований череп, гіпоплазію тимусу та паращитовидних залоз. Під час лабораторного імунологічного обстеження виявлено: лімфоцитопенія, зниження рівня та проліферативної відповіді Т-лімфоцитів, нормальний рівень імуноглобулінів. Який найімовірніший діагноз?A two-month-old child has convulsions, recurrent viral and bacterial infections of the upper respiratory tract. A deformed skull, hypoplasia of the thymus and parathyroid glands were detected. During the laboratory immunological examination, lymphocytopenia was detected , a decrease in the level and proliferative response of T-lymphocytes, a normal level of immunoglobulins. What is the most likely diagnosis?
Загальний варіабельний імунодефіцитGeneral variable immunodeficiency
Жінка 32-х років скаржиться на посилення спастичного болю внизу живота після психоемоційного напруження. Випорожнення кишечника інтермітуючі: 2-3 випорожнення після пробудження чергуються із закрепами протягом 1-2 днів впродовж 6 місяців. Об'єктивно: маса тіла збережена, помірний біль під час пальпації сигмоподібної кишки. Hb- 130 г/л, лейкоцити - 5,2 Г/л, швидкість осідання еритроцитів - 9 мм/год. Ректороманоскопічне дослідження болісне через спастичний стан кишечника, його слизова оболонка не змінена. В просвіті кишечника багато слизу. Яке захворювання найбільш імовірне у даної пацієнтки?A 32-year-old woman complains of increased spastic pain in the lower abdomen after psycho-emotional stress. Intermittent bowel movements: 2-3 bowel movements after waking up alternate with constipation for 1-2 days for 6 months. Body weight is preserved, moderate pain during palpation of the sigmoid intestine. Hb - 5.2 G/l, erythrocyte sedimentation rate - 9 mm/h spastic condition of the intestine, its mucous membrane is not changed. There is a lot of mucus in the intestinal lumen. What disease is most likely in this patient?
Пацієнтка віком 12 років скаржиться на слабкість, запаморочення, головний біль, підвищення температури тіла до 38^oС. Об'єктивно спостерігається: температура тіла 37,8^oС, слизові та шкіра бліді, зів без змін. Пальпуються збільшені до 2-х см підщелепні та шийні лімфовузли, щільні, неболючі. Патологічних змін внутрішніх органів не виявлено. У загальному аналізі крові виявлено: еритроцити - 2,8·10^12/л, гемоглобін - 85 г/л, кольоровий показник - 0,9, лейкоцити - 10·10^9/л, еозинофіли - 0%, паличкоядерні - 1%, сегментоядерні - 8%, лімфоцити - 47%, ретикулоцити - 0,5%, тромбоцити - 60·10^9/л, бластні клітини - 44%. Який найімовірніший діагноз?A 12-year-old patient complains of weakness, dizziness, headache, an increase in body temperature up to 38°C. Objectively observed: body temperature 37.8°C , mucous membranes and skin are unchanged. Submaxillary and cervical lymph nodes are palpable, painless. No pathological changes in internal organs are detected: erythrocytes - 2.8·10^12/l , hemoglobin - 85 g/l, color index - 0.9, leukocytes - 10·10^9/l, eosinophils - 0%, rod-nuclear - 1%, segmentonuclear - 8%, lymphocytes - 47%, reticulocytes - 0.5 %, platelets - 60·10^9/l, blast cells - 44%. What is the most likely diagnosis?
Інфекційний мононуклеозInfectious mononucleosis
ЛімфогрануломатозLymphogranulomatosis
Гострий еритромієлозAcute erythromyelosis
Гострий лейкозAcute leukemia
Хронічний лімфолейкозChronic lymphocytic leukemia
Хірургія
230/1500
Жінка оперована з приводу дифузно-токсичного зобу ІІ ступеня. За 12 годин після операції скаржиться на захриплість та втрату голосу, утруднення дихання, задишку, тривогу. Післяопераційна рана без особливостей. Яке ускладення виникло у пацієнтки?A woman was operated on for a diffuse-toxic goiter of the II degree. 12 hours after the operation, she complains of hoarseness and loss of voice, difficulty breathing, shortness of breath, anxiety. Postoperative wound without features. What condition did the patient have?
Хлопчик 3 років доставлений до стаціонару в важкому стані. Об'єктивно: сомнолентність, гіперрефлексія, судоми, гіперестезія, невпинне блювання, температура тіла - 39,9^oC, ЧСС - 160/хв., АТ - 80/40 мм рт. ст. Яке дослідження слід провести насамперед?A 3-year-old boy was brought to the hospital in serious condition. Objectively: somnolence, hyperreflexia, convulsions, hyperesthesia, incessant vomiting, body temperature - 39.9^oC , heart rate - 160/min., blood pressure - 80/40 mm Hg. What research should be done first?
Люмбальна пункціяLumbar puncture
РеоенцефалографіяRheoencephalography
Рентгенографія черепаX-ray skull
ЕхоенцефалографіяEchoencephalography
Комп'ютерна томографія мозкуComputed tomography of the brain
Терапія
232/1500
Чоловік віком 40 років скаржиться на ниючий біль у м'язах спини, поперековій ділянці, ногах, плечових і кульшових суглобах, погіршення зору, прискорене серцебиття. Об'єктивно спостерігаються явища увеїту. Рентгенографічно виявлено поодинокі синдесмофіти між тілами хребців і розмитість контурів крижово–клубових суглобів. Під час лабораторного обстеження виявлено антитіла до антигенів HLA–B27, анемію, ШОЕ - 28 мм/год. Яке захворювання зумовлює таку картину?A 40-year-old man complains of aching pain in the muscles of the back, lumbar region, legs, shoulder and hip joints, impaired vision, rapid heartbeat. Objectively the phenomena of uveitis are observed. Single syndesmophytes between the vertebral bodies and blurring of the contours of the sacroiliac joints are detected. During the laboratory examination, antibodies to HLA-B27 antigens, ESR - 28 mm/h are found. What disease causes such a picture?
Дитині 8 місяців. Тиждень тому перенесла ГРВІ. Фізичний та нервово-психічний розвиток відповідають вікові. Скарги на млявість, зниження апетиту, немотивоване повторне блювання, підвищення температури протягом останньої доби до 38^oC, блідість, часте сечовипускання малими порціями. Тахікардія. В аналізі сечі виявлено: білок - 0,099%, лейкоцити - 15-20 в полі зору, бактеріурія - +++, слиз - +++. Яке захворювання можна припустити?The child is 8 months old. A week ago he suffered SARS. Physical and neuropsychological development correspond to the age. Complaints of lethargy, decreased appetite, unmotivated repeated vomiting, increased temperature during the last days up to 38°C, frequent urination in small portions. Urinalysis revealed: protein - 15-20 in the field of vision, bacteriuria - +++. What disease can be assumed ?
Фосфат-діабетPhosphate-Diabetes
Гострий пієлонефритAcute pyelonephritis
Гострий гломерулонефритAcute glomerulonephritis
Дефекти доглядуCare Defects
Дисметаболічна нефропатіяDysmetabolic nephropathy
Терапія
234/1500
Пацієнт віком 29 років хворіє гормонозалежною бронхіальною астмою. Під час флюорографії в С2 правої легені виявлено округлу тінь з чіткими рівними контурами, середньої інтенсивності. Навколо неї кілька поліморфних вогнищевих тіней, у корені - кальцинат. Під час обстеження спостерігається: перкуторний звук над легенями з коробковим відтінком, вислуховуються розсіяні сухі хрипи. Аналіз крові без змін. Реакція на пробу Манту з 2 ТО ППД-Л - папула 22 мм. Який найімовірніший діагноз?A 29-year-old patient suffers from hormone-dependent bronchial asthma. During fluorography, a rounded shadow with clear even contours of medium intensity was detected in C2 of the right lung. Several polymorphic focal shadows were found around it , in the root - calcification. During the examination, there is a percussion sound over the lungs, scattered dry rales are heard. The reaction to the Mantoux test is 22 mm. What is the most likely diagnosis?
Периферичний ракPeripheral cancer
ТуберкульомаTuberculoma
Еозинофільний інфільтратEosinophilic infiltrate
ПневмоніяPneumonia
АспергільомаAspergilloma
Акушерство і гінекологія
235/1500
Пацієнтка віком 30 років на п'яту добу після фізіологічних пологів скаржиться на нагрубання лівої молочної залози, біль, почервоніння шкіри, підвищення температури тіла до 38^oС. Об'єктивно спостерігається: ліва молочна залоза збільшена у розмірі, шкіра червоного кольору, в верхньо-зовнішньому квадранті з ціанозом, пальпується без чітких меж інфільтрат, різко болісний. Зціджування молока не приносить полегшення. Який найімовірніший діагноз?On the fifth day after physiological childbirth, a 30-year-old patient complains of swelling of the left mammary gland, pain, redness of the skin, an increase in body temperature up to 38°C. 'objectively observed: the left mammary gland is enlarged, the skin is red, in the upper-external quadrant, the infiltrate is palpable, sharply painful. Expressing milk does not bring relief. What is the most likely diagnosis?
Інфікована кіста молочної залозиInfected breast cyst
Терапія
236/1500
Чоловік 37 років скаржиться на гострий біль у лівій половині грудної клітки, задишку, що посилюється під час будь-яких рухів. Захворів раптово після значного фізичного навантаження. Під час обстеження встановлено: помірний ціаноз обличчя, ліва половина грудної клітки відстає в акті дихання. Перкуторно виявлено: зліва - тимпаніт, дихання ослаблене. ЧД - 24/хв. Тони серця ослаблені. ЧСС - 90/хв. На рентгенограмі видно лінія вісцеральної плеври. Назовні від неї легеневий малюнок відсутній. Який найімовірніший діагноз?A 37-year-old man complains of sharp pain in the left half of the chest, shortness of breath that worsens during any movement. He fell ill suddenly after significant physical exertion. During examination revealed: moderate cyanosis of the chest. Percussion revealed: heart rate - 24/min. Visceral pleura line she has no lung pattern. What is the most likely diagnosis?
Тромбоемболія легеневої артеріїThromboembolism of the pulmonary artery
Інфаркт міокардаMyocardial infarction
Спонтанний пневмотораксSpontaneous pneumothorax
Лівостороння пневмоніяLeft-sided pneumonia
Терапія
237/1500
Чоловік 70 років за 2 тижні до госпіталізації у стаціонар спостерігав підвищення температури до 38^oС. З діагнозом пневмонія був направлений до лікарні. Під час огляду встановлено: блідість шкіри, тахікардія. У нижніх відділах легенів - дрібнопухирчаті хрипи. У V точці - короткий протодіастолічний шум. АТ - 140/40 мм рт. ст., пальпується нижній край селезінки. Hb - 40 г/л., еритроцити - 2,5·10^12/л, лейкоцити - 12·10^9/л, ШОЕ - 35 мм/год. На ЕКГ спостерігається: депресія ST в V 5,6. У сечі - протеїнурія. Який найімовірніший діагноз?2 weeks before hospitalization, a 70-year-old man observed an increase in temperature to 38°C. He was referred to the hospital with a diagnosis of pneumonia. During the examination, it was found: pale skin , tachycardia. In the lower parts of the lungs - a short protodiastolic murmur. Hb - 40 g/l ^12/l, leukocytes - 12·10^9/l, ESR - 35 mm/h. On the ECG, there is depression in V 5.6. What is the most likely diagnosis?
Ревматична вада серцяRheumatic heart disease
Хронічний мієлолейкозChronic myelogenous leukemia
Гострий міокардитAcute myocarditis
Гострий перикардитAcute pericarditis
Бактеріальний ендокардитBacterial endocarditis
Педіатрія
238/1500
Лікар оглядає недоношену новонароджену дитину. Об'єктивно спостерігалося одиничне апное тривалістю до 5 секунд. Якою основною функціональною особливістю дихальної системи можна пояснити приступи апное у недоношеної дитини?The doctor examines a premature newborn baby. A single apnea lasting up to 5 seconds was objectively observed. What main functional feature of the respiratory system can explain apnea attacks in a premature baby?
Схильність слизових оболонок до набряку та гіперсекреціїPropensity of mucous membranes to edema and hypersecretion
Функціональна незрілість дихального центруFunctional immaturity of the respiratory center
Недостатня дренажна та захисна функція бронхівInsufficient drainage and protective function of the bronchi
М'якість хрящів трахеї та бронхівSoftness of tracheal and bronchial cartilages
Вузкість просвіту дихальних шляхівNarrowness of the airway
Терапія
239/1500
Пацієнта віком 38 років турбує біль у ділянці плеснофалангових суглобів 1-2 пальців правої стопи. Під час огляду виявлено: шкіра над ураженими суглобами багряно-синюшного кольору, на дотик гаряча, у ділянці вушних раковин наявні вузликові утворення, вкриті тонкою блискучою шкірою. Який препарат для уратзнижувальної терапії потрібно призначити пацієнту?A 38-year-old patient is bothered by pain in the area of the metatarsophalangeal joints of 1-2 toes of the right foot. During the examination, the skin over the affected joints is purple-bluish to the touch hot, there are nodular formations in the area of the auricles, covered with thin shiny skin. What drug should be prescribed for urate-lowering therapy?
КолхіцинColchicine
АлопуринолAllopurinol
ФебуксостатFebuxostat
НімесулідNimesulide
МетотрексатMethotrexate
Хірургія
240/1500
До лікаря звернулася жінка 74 років зі скаргами на біль у пахвинній ділянці справа. Ці симптоми виникли раптово близько 2 годин тому. З анамнезу відомо: пацієнтка каже, що ці симптоми вже були у неї близько 3 тижнів тому, але самі зникли, після того як хвора лягла. Об'єктивно спостерігається: нижче пупартової зв'язки пальпується щільне, напружене утворення до 3,5 см у діаметрі, різко болюче, симптом <<кашльового поштовху>> негативний. Який діагноз найімовірніший?A 74-year-old woman came to the doctor with complaints of pain in the inguinal region on the right. These symptoms appeared suddenly about 2 hours ago. It is known from the anamnesis: the patient says that these she already had symptoms about 3 weeks ago, but they disappeared after the patient lay down. Objectively observed: a dense, tense mass up to 3.5 cm in diameter, sharply painful, is palpable below the Pupart ligament. shock>> negative. What is the most likely diagnosis?
Серед населення, що мешкає поблизу підприємства з виробництва пестицидів, динамічно підвищується рівень вроджених вад розвитку, що виражаються в центральному паралічі, ідіотії та сліпоті новонароджених. Сполуки якої хімічної речовини можуть зумовити розвиток цієї патології?Among the population living near a pesticide production plant, the level of congenital malformations, expressed in central paralysis, idiocy and blindness in newborns, is dynamically increasing. Compounds of which chemical substance can cause the development of this pathology?
ХромChrome
ЗалізоIron
КадмійCadmium
СтронційStrontium
РтутьMercury
Терапія
242/1500
Чоловік 32-х років звернувся до сімейного лікаря зі скаргами на нав'язливі спогади про автомобільну аварію, яка була 3 місяці тому, порушення нічного сну, тривогу, зниження настрою і апетиту, загальну слабкість і спустошеність. Що найбільш доцільно призначити?A 32-year-old man presented to his family doctor with complaints of intrusive memories of a car accident 3 months ago, nighttime sleep disturbances, anxiety, decreased mood and appetite, general weakness and desolation. What is the most appropriate to prescribe?
СертралінSertraline
ХлорпромазинChlorpromazine
КарбамазепінCarbamazepine
Літію карбонатLithium carbonate
АмітриптилінAmitriptyline
Терапія
243/1500
Хлопець 27-ми років під час купання в річці раптово знепритомнів та зник з поверхні води. Через 2 хвилини був доставлений до берега. Об'єктивно: непритомний, шкіра вираженого синюшного кольору, пінисті виділення з рота та носа, дихання та пульс на сонних артеріях відсутні. Який вид допомоги необхідно надати в цьому випадку?A 27-year-old boy while swimming in the river suddenly fainted and disappeared from the surface of the water. After 2 minutes, he was brought to the shore. Objectively: unconscious, skin pronounced bluish color, foamy discharge from the mouth and nose, breathing and pulse on the carotid arteries are absent. What kind of help should be provided in this case?
Первинна медична допомогаPrimary medical care
Екстрена медична допомогаEmergency medical assistance
Третинна медична допомогаTertiary medical care
Паліативна медична допомогаPalliative medical care
Вторинна медична допомогаSecondary medical care
Терапія
244/1500
Пацієнт віком 56 років скаржиться на зниження апетиту, слабкість, серцебиття, біль та відчуття печіння язика, відчуття важкості в надчеревній ділянці та почуття оніміння кінцівок. Об'єктивно спостерігається: блідність шкіри з лимонним відтінком, глосит Гантера, збільшення печінки, селезінки. У загальному аналізі крові виявлено: еритроцити - 2,8·10^12/л, гемоглобін - 100 г/л, кольоровий показник - 1,2. Еритроцити великі, часто овальні з тільцями Жоллі та кільцями Кебота. Який найімовірніший діагноз?A 56-year-old patient complains of decreased appetite, weakness, palpitations, pain and a burning sensation in the tongue, a feeling of heaviness in the epigastric area, and a feeling of numbness in the limbs. Objectively observed : pallor of the skin with a lemon hue, Ganter's glossitis, enlargement of the liver, spleen. In the general blood analysis, erythrocytes - 2.8·10^12/l, hemoglobin - 100 g/l, erythrocytes are large. often oval with Jolly bodies and Cabot rings. What is the most likely diagnosis?
Жінка 35-ти років звернулася до лікаря зі скаргами на зростаючий біль під час менструації протягом 1-го року. Тривалість менструального циклу 28 днів. 2 роки тому була проведена лапароскопічна перев'язка маткових труб. При пальпації матка збільшена до 8 тижнів вагітності, м'якої консистенції, болюча. Придатки з обох сторін не збільшені, при пальпації безболісні. Який діагноз є найбільш імовірним?A 35-year-old woman consulted a doctor with complaints of increasing pain during menstruation during the 1st year. The duration of the menstrual cycle is 28 days. 2 years ago she was laparoscopic tubal ligation. On palpation, the uterus is soft, painful. On palpation, the appendages are painless. What is the most likely diagnosis?
Позаматкова вагітністьEctopic pregnancy
АденоміозAdenomyosis
ГідросальпінксHydrosalpinx
Полікістоз яєчниківPolycystic ovary
Зовнішній ендометріозExternal endometriosis
Акушерство і гінекологія
246/1500
До пологового відділення шпиталізовано жінку, яка народжує вперше, зі скаргами на перейми нерегулярного характеру та кров'янисті виділення зі статевих шляхів. Термін вагітності 39 тижнів. Під час зовнішнього акушерського дослідження встановлено: положення плода повздовжнє, головне передлежання. Визначається гіпертонус матки. Під час внутрішнього акушерського дослідження спостерігається: шийка матки довжиною 1 см, цервікальний канал відкритий на 2 см. Передлежить голівка плода, притиснута до входу в малий таз. Виділення кров'янисті, зі згортками. Серцебиття плода аритмічне, 80-100/хв. Яке ускладнення виникло у роділлі?A woman giving birth for the first time was hospitalized in the maternity ward with complaints of irregular periods and bleeding from the genital tract. The term of pregnancy is 39 weeks. During external Obstetric examination revealed: the position of the fetus is longitudinal, the uterus is hypertonic. During the internal obstetric examination, the cervix is 1 cm long. The fetal head is pressed against the entrance to the pelvis , with contractions. The heartbeat of the fetus is arrhythmic, 80-100/min. What complication occurred during childbirth?
У чоловіка на різних ділянках шкірного покриву, переважно симетрично, розташовані депігментовані плями, що не лущаться, різного розміру й обрисів. Частина волосся на них знебарвлена. Захворювання почалося в дитинстві. Появі плям не передували будь-які елементи висипки. Який імовірний діагноз?The man has depigmented spots that do not peel, of different sizes and outlines, on different areas of the skin, mostly symmetrically. Some of the hair on them is discolored. The disease began in childhood. The appearance of spots was not preceded by any elements of the rash. What is the probable diagnosis?
АльбінізмAlbinism
ПсоріазPsoriasis
Рожевий лишайPink lichen
ВітилігоVitiligo
Різнокольоровий лишайMulticolored lichen
Педіатрія
248/1500
Хлопчик 5-ти років прийшов з матір'ю на прийом до лікаря зі скаргами на припухлість обличчя, біль у вухах та підвищену температуру протягом останніх 5 днів. Через релігійні переконання матері вакцинація дитини неповна. Під час фізикального обстеження виявлено двосторонній, болючий набряк у ділянці жувального м'яза та гарячку 39,3^oC. Що з перерахованого буде найбільш доцільно призначити цьому паціє-нту?A 5-year-old boy came with his mother to a doctor's appointment with complaints of facial swelling, ear pain, and high temperature for the past 5 days. Due to religious the mother's conviction that the child's vaccination is incomplete. During the physical examination, bilateral, painful swelling in the area of the masticatory muscle and a fever of 39.3°C were detected. Which of the following would be most appropriate to prescribe for this patient?
ПарацетамолParacetamol
Імуноглобулін людини нормальнийHuman immunoglobulin is normal
ПеніцилінPenicillin
АцикловірAcyclovir
ЦефтриаксонCeftriaxone
Педіатрія
249/1500
Під час огляду новонародженого, який народився в сідничному передлежанні, педіатр установив обмеження пасивного відведення правого стегна і клацання у разі його відведення, асиметрію пахових і стегнових сідничних складок. Яка ознака достовірно свідчить про наявність вродженого вивиху стегна у дитини?During the examination of a newborn who was born in the breech presentation, the pediatrician established a limitation of passive abduction of the right hip and clicking in the event of its abduction, asymmetry of the inguinal and femoral buttock folds. Which does the sign reliably indicate the presence of congenital dislocation of the hip in the child?
Достовірної ознаки немаєThere is no valid sign
Асиметрія пахових і стегнових сіднич-них складокAsymmetry of inguinal and femoral gluteal folds
Народження в сідничному передлежа-нніBirth in breech presentation
Обмеження пасивного відведення стегнаLimit passive abduction of hip
Клацання у разі відведення стегнаClick when abducting hip
Хірургія
250/1500
Чоловік скаржиться на болючу припухлість у ділянці підборіддя, загальне нездужання, головний біль. У цій ділянці виявлено гостро запалений щільний вузол конусоподібної форми. Шкіра над ним напружена, червона. Усередині вузла є виразка з прямовисними краями і некротичним стрижнем брудно-зеленого кольору. Підщелепні лімфатичні вузли справа збільшені і болючі. Який діагноз найімовірніший?A man complains of painful swelling in the chin area, general malaise, headache. In this area, an acutely inflamed, dense, cone-shaped node was found. The skin above it is tense, red. Inside the node there is an ulcer with vertical edges and a necrotic rod of dirty green color. The submandibular lymph nodes on the right are enlarged and painful. What is the most likely diagnosis?
При вивченні середнього рівня та характеру різноманітності деяких лабораторних показників отримані такі дані: для загального білку крові - середнє квадратичне відхилення pm4 г/л, коефіцієнт варіації - 6%; для швидкість осідання еритроцитів відповідно pm2 мм/год, 23%. Яка з ознак, що вивчаються, є найбільш різноманітною?When studying the average level and nature of the diversity of some laboratory indicators, the following data were obtained: for total blood protein - mean square deviation of pm4 g/l, coefficient of variation - 6%; for the sedimentation rate of erythrocytes, respectively, pm2 mm/h, 23%. Which of the studied features is the most diverse?
Відмінності в різноманітності ознак відсутніThere are no differences in the diversity of features
Для вивчення різноманітності потрібні додаткові дослідженняMore research is needed to study diversity
Для вивчення різноманітності потрібні додаткові розрахункиExamination of diversity requires additional calculations
Загальний білок сироватки кровіTotal serum protein
Швидкість осідання еритроцитів (ШОЕ)Erythrocyte Sedimentation Rate (ESR)
Акушерство і гінекологія
252/1500
Пацієнтка 48 років із матковою кровотечею. В анамнезі вказано порушення менструального циклу впродовж року. Під час огляду шийки матки патології не виявлено. Бімануально встановлено: матка нормальних розмірів, неболюча, рухома, придатки без особливостей. Виділення кров'янисті, рясні. Що треба виконати на цьому етапі насамперед?A 48-year-old female patient with uterine bleeding. The anamnesis indicated a violation of the menstrual cycle for a year. During the examination of the cervix, no pathology was found. It was established bimanually: the uterus is of normal size, painless , mobile, appendages without features. Bloody discharge, abundant. What should be done at this stage first?
ГістеректоміюHysterectomy
Вишкрібання стінок маткиScraping the walls of the uterus
Гормональну терапіюHormone therapy
Уведення окситоцинуOxytocin administration
Уведення вітаміну КVitamin K administration
Акушерство і гінекологія
253/1500
Пацієнтка на 3-тю добу після штучного аборту госпіталізована до гінекологічного відділення у тяжкому стані з симптомами інтоксикації, болем у животі, гнійними виділеннями з піхви. Об'єктивно: стан жінки тяжкий, температура тіла - 38,8^oC, пульс - 100/хв., АТ- 110/70 мм рт.ст., матка м'яка, дно її на рівні пупка, симптоми подразнення очеревини позитивні. Який найбільш імовірний діагноз?On the 3rd day after the artificial abortion, the patient was hospitalized in the gynecological department in a serious condition with symptoms of intoxication, abdominal pain, purulent discharge from the vagina. Objectively: the woman's condition is severe, body temperature - 38.8°C, pulse - 100/min, blood pressure - 110/70 mm Hg, the uterus is soft, the bottom is at the level of the navel, the symptoms of peritoneal irritation are positive diagnosis?
Чоловіку 42-х років виконано геміти-реоїдектомію з приводу вузлового еути-реоїдного зоба. Призначення якого лікування найбільш імовірно допоможе запобігти рецидиву захворювання?A 42-year-old man underwent hemit-rheoidectomy for a nodular euthy-rheoid goiter. Which treatment is most likely to help prevent recurrence of the disease?
L-тироксинL-thyroxine
Радіоактивний йодRadioactive iodine
МерказолілMercazolil
АнтиструмінAnticurrent
ТиротропінThyrotropin
Педіатрія
255/1500
Мати прийшла на прийом до педіатра зробити своїй 2-місячній дитині планові щеплення. Для зручності, вона бажає аби дитині було введено комбіновану 6-компонентну вакцину, що містить антигени усіх збудників, проти яких рекомендовано вакцинуватися у цьому віці. Під час збору анамнезу вона зазначила, що іноді після пробудження її дитина починає раптово напружуватися, згинати голівку, ручки та ніжки. Зазвичай такі напади тривають не більше декількох секунд з інтервалами ''спо-кою'' у 10 секунд. Останній напад був декілька днів тому. Температура тіла - 37,2^oC , частота дихання - 25/хв., пульс - 104/хв., артеріальний тиск - 100/70 мм рт.ст. Яке рішення має прийняти лікар щодо подальшої імунізації цієї дитини?A mother came to the pediatrician's appointment to give her 2-month-old child routine vaccinations. For convenience, she wants the child to be given a combined 6-component vaccine containing antigens of all the pathogens that are recommended to be vaccinated at this age. During the anamnesis, she noted that sometimes after waking up, her child begins to tense up, bend the head, arms and legs. Usually such attacks last no more than a few seconds with 'calm' intervals '' in 10 seconds. The last attack was several days ago. Body temperature - 25/min., pulse - 100/70 mm Hg should the doctor accept regarding further immunization of this child?
Провести вакцинацію за схемою гепатит В+Hib-інфекціяVaccinate according to the hepatitis B+Hib infection scheme
Введення будь-яких вакцин на даний момент протипоказаноIntroduction of any vaccines is currently contraindicated
Провести вакцинацію за схемою АКДП+гепатит В+Hib-інфекціяVaccinate according to the scheme AKDP+hepatitis B+Hib-infection
Провести вакцинацію за схемою АДП+поліо+гепатит B+Hib-інфекціяVaccinate according to the scheme ADP+polio+hepatitis B+Hib-infection
Ввести комбіновану 6-компонентну вакцинуEnter a combined 6-component vaccine
Терапія
256/1500
Пацієнта віком 62 роки шпиталізовано зі скаргами на сильний біль за грудниною, що триває 1 годину і не знімається нітрогліцирином. Хворіє на стенокардію, раніше напад знімався нітратами. Інших захворювань немає. Об'єктивно спостерігається: ціаноз губ. Тони серця глухі, ритмічні. На ЕКГ виявлено: елевація сегменту ST у V4-V6. Який лікарський засіб потрібно негайно ввести пацієнту?A 62-year-old patient was hospitalized with complaints of severe chest pain lasting 1 hour and not relieved by nitroglycerin. He suffers from angina pectoris, previously the attack was relieved by nitrates. Other diseases Objectively observed: cyanosis of the heart, rhythmic: elevation of the ST segment in V4-V6 is detected immediately.
АналгінAnalgin
ДімедролDiphenhydramine
Но-шпуNo-shpu
КоргліконCorglycon
АктилізеActilize
Педіатрія
257/1500
У новонародженої дитини на третій день життя з'явилася жовтяниця шкіри та склер, загальний стан не порушений, сон і апетит без відхилень. З анамнезу відомо: дитина від ІІ пологів, у матері група крові - А (ІІ) Rh(+), у дитини - А (ІІ) Rh(+). Який стан виник у новонародженої дитини?On the third day of life, a newborn child developed jaundice of the skin and sclera, the general condition is not impaired, sleep and appetite are normal. It is known from the anamnesis: the child from II childbirth, the mother's blood group is A (II) Rh(+), the child's blood group is A (II) Rh(+). What condition did the newborn have?
Гемолітична хворобаHemolytic disease
Фізіологічна жовтяницяPhysiological jaundice
Інфекційний гепатитInfectious hepatitis
СепсисSepsis
Пологова травмаBirth trauma
Терапія
258/1500
Чоловік віком 35 років був знайдений мертвим у власному гаражі. Під час судово-медичного дослідження виявлено: странгуляційна борозна горизонтальна, замкнута, петехіальні крововиливи шкіри голови та шиї, ціаноз обличчя, трупні плями добре виражені, множинні плями Тардьє. Які ознаки вказують на те, що чоловік помер не внаслідок повішення?A 35-year-old man was found dead in his garage. Forensic examination revealed: horizontal strangulation groove, closed, petechial hemorrhages of scalp and neck, cyanosis face, cadaver spots well defined, multiple Tardieu spots. What signs indicate that the man did not die as a result of hanging?
Множинні плями ТардьєMultiple Tardieu spots
Добре виражені трупні плямиWell defined corpse spots
Роділлю віком 27 років у терміні вагітності 38-39 тижнів шпиталізовано до пологового відділення з приводу кров'янистих виділень зі статевих шляхів, що з'явилися з початком пологової діяльності. Крововтрата склала - 90 мл. Загальний стан пацієнтки задовільний, АТ - 110/70 мм рт. ст. Під час піхвового дослідження спостерігається: шийка матки вкорочена до 0,5 см, розкриття шийки матки 3 см, спереду і праворуч від передлежачої голівки визначається губчаста тканина з нерівною поверхнею, яка доходить до внутрішнього вічка шийки матки. Плідний міхур цілий. Після обстеження кровотеча стала інтенсивнішою. Який найбільш імовірний діагноз?A 27-year-old woman in the 38-39th week of pregnancy was hospitalized in the maternity ward due to bloody discharge from the genital tract that appeared with the onset of labor The blood loss was 90 ml. The patient's blood pressure was 110/70 mm Hg. During the vaginal examination, the cervix was shortened to 3 cm, in front and to the right of the head. spongy tissue that reaches the inner cervix. After examination, the bleeding became more intense?
Під час проведення епідеміологічного дослідження на території міста згідно висунутою гіпотезою вивчалися причино-наслідкові зв'язки виникнення нових випадків захворювань залежно від визначених факторів ризику. Який вид епідеміологічного дослідження був використаний?During an epidemiological study on the territory of the city, according to the proposed hypothesis, the cause-and-effect relationships of the occurrence of new cases of diseases depending on the identified risk factors were studied. What type of epidemiological study was used?
АналітичнийAnalytic
ОписовийDescriptive
Квазі-експериментальнийQuasi-experimental
ЕкспериментальнийExperimental
КлінічнийClinical
Акушерство і гінекологія
261/1500
Вагітна жінка віком 22 роки госпіталізована у важкому стані. Протягом останніх трьох днів з'явилися набряки, головний біль, нудота, одноразове блювання. Об'єктивно спостерігається: свідомість потьмарена, АТ - 160/130 мм рт. ст., дрібні фібрилярні посмикування м'язів обличччя, утруднене носове дихання. Під час транспортування почалося посмикування верхніх кінцівок, тіло жінки витяглося, хребет вигнувся, щелепи щільно стиснулися, дихання припинилося. Після цього з'явилися клонічні судоми, виражений ціаноз. Потім судоми припинилися, з'явився глибокий шумний вдих, на губах виступила піна, забарвлена кров'ю. Який діагноз найімовірніший?A 22-year-old pregnant woman was hospitalized in serious condition. Over the past three days, swelling, headache, nausea, and one-time vomiting appeared. Objectively observed: consciousness darkened, blood pressure - 160/130 mm Hg, small fibrillar twitching of the facial muscles, difficulty breathing 'clonic convulsions appeared, pronounced cyanosis. Then the convulsions stopped, a deep noisy breath appeared, foam appeared on the lips. What is the most likely diagnosis?
Гіпертонічний кризHypertensive crisis
Діабетична комаDiabetic coma
ЕпілепсіяEpilepsy
ХореяChorea
ЕклампсіяEclampsia
Терапія
262/1500
Пацієнт скаржиться на нестерпний кашель з виділенням до 600 мл за добу гнійного мокротиння шоколадного кольору з гнилісним запахом. З анамнезу відомо що захворів гостро, температура тіла - 39^oС, лихоманка неправильного типу. Під час рентгенологічного дослідження виявлено: ділянка затемнення з порожниною у центрі, з неправильними контурами та з рівнем рідини. Який найімовірніший діагноз?The patient complains of a persistent cough with discharge of up to 600 ml per day of chocolate-colored purulent sputum with a putrid smell. It is known from the anamnesis that he was acutely ill, the body temperature is 39^ oC, fever of the wrong type. During the X-ray examination, an area of darkening with a cavity in the center, with irregular contours and a level of fluid was found. What is the most likely diagnosis?
Рак легені з розпадомLung cancer with decay
Кавернозний туберкульозCavernous tuberculosis
Бронхоектатична хворобаBronchoectatic disease
Абсцес легеніLung abscess
Гангрена легеніGangrene of the lung
Терапія
263/1500
Пацієнтка скаржиться на частi, ряснi, без патологiчних домiшок, пiнистi випорожнення, переймоподiбний бiль у навколопупковiй дiлянцi, бурчання в животi, лихоманку. З анамнезу відомо про вживання в їжу качиних яєць, зварених некруто. Який збудник захворювання найiмовiрніший?The patient complains of frequent, abundant, without pathological impurities, foamy stools, cramp-like pain in the peri-umbilical region, stomach rumbling, fever. From the anamnesis, it is known about eating hard-boiled duck eggs. What is the most likely causative agent of the disease?
Escherichia coliEscherichia coli
V. choleraeV. cholerae
Salmonella enteritidisSalmonella enteritidis
Salmonella typhiSalmonella typhi
Shigella sonneiShigella sonnei
Хірургія
264/1500
Хлопчик 12-ти років звернувся до хірурга зі скаргою на нестерпний біль кінчика II пальця правої кисті. Лікарем з'ясовано, що 4 дні тому він вколов це місце дротом. Протягом останніх трьох днів з'явився різкий пульсуючий біль, набряк тканин та гіперемія дистальної фаланги пальця. Об'єктивно: температура тіла - 38,1^oC. Який з перерахованих діагнозів є найбільш імовірним?A 12-year-old boy turned to the surgeon complaining of excruciating pain in the tip of the II finger of the right hand. The doctor found out that 4 days ago he pricked this place with a wire During the last three days, sharp throbbing pain and swelling of the distal phalanx of the finger appeared. Objectively, the body temperature is 38.1°C.
ЕризипелоїдErysipeloid
ПароніхіяParonychia
ЦелюлітCellulite
--
ПанариційPanaricius
Педіатрія
265/1500
Дитина народилася з терміном гестації 30 тижнів, із масою - 1100,0 г, через 3 години після народження з'явилися часті апное, дихання за типом <>, ЧСС - 98/хв. Оцінка за шкалою Сільвермана - 9 балів. Додаткова оксиногенація не привела до поліпшення стану. Що потрібно зробити?The child was born with a gestation period of 30 weeks, with a weight of 1100.0 g, frequent apnea appeared 3 hours after birth, breathing in the type <>, heart rate - 98/min. Score on the Silverman scale - 9 points. Additional oxygenation did not improve the condition?
Почати штучну вентиляцію легень під позитивним тискомStart positive pressure ventilation
Почати інтубувати трахеюStart tracheal intubation
Почати штучну вентиляцію легеньStart CPR
Не класифіковано
266/1500
До лікарні шпиталізовано постраждалого в ДТП чоловіка віком 42 роки з діагнозом: політравма, розрив селезінки, перелом правого стегна. Об'єктивно спостерігається: загальмований, шкіра вкрита липким холодним потом, бліда. Дихання поверхневе, часте. Пульс - 140/хв, слабкого наповнення, АТ - 65/40 мм рт. ст., ЦВТ - 0. Планується операційне втручання під внутрішньовенною анестезією зі штучною вентиляцією легень. Який анестетик необхідно вибрати для проведення загальної анестезії?A 42-year-old man injured in a road accident was admitted to the hospital with the diagnosis: polytrauma, rupture of the spleen, fracture of the right hip. Objectively observed: slowed down, skin covered with sticky cold sweat, shallow breathing. Pulse - 140/min, blood pressure - 65/40 mmHg. An operation under intravenous anesthesia is planned general anesthesia?
ІзофлюранIsoflurane
КетамінKetamine
СевофлуранSevoflurane
ПропофолPropofol
Тіопентал натріюSodium thiopental
Терапія
267/1500
У чоловіка 59 років протягом місяця спостерігається короткочасне зниження сили у лівих кінцівках. Потім уранці після сну розвинулась стійка слабкість у цих кінцівках. Об'єктивно спостерігається: свідомість не втрачена, центральний парез VII і XII пар черепних нервів зліва. З того ж боку центральний геміпарез і гемігіперстезія. Які препарати вибору для диференційованного лікування?A 59-year-old man experienced a short-term decrease in strength in his left limbs during the month. Then in the morning after sleep, persistent weakness developed in these limbs. Objectively observed: consciousness is not lost , central paresis of the VII and XII pairs of cranial nerves on the left side. Central hemiparesis and hemihyperesthesia. What are the drugs of choice for differential treatment?
ДиуретикиDiuretics
КортикостероїдиCorticosteroids
ГіпотензивніHypotensive
АнтикоагулянтиAnticoagulants
ГемостатикиHemostatics
Терапія
268/1500
Жінка 35-ти років надійшла до відділення інтенсивної терапії з нападами судом з періодичністю 2-3 хвилини, між якими не відбувається покращення свідомості, реакція зіниць на світло відсутня. З дитинства хворіє на епілепсію. Зазвичай напади виникають 1-2 рази на місяць у нічний час та супроводжуються мимовільним сечовиділенням та дефекацією. Почастішання нападів та вищевказані ускладнення виникли на тлі перенесеного грипу. Який стан найбільш імовірно розвинувся у хворої?A 35-year-old woman came to the intensive care unit with convulsions every 2-3 minutes, between which there is no improvement in consciousness, and the reaction of the pupils to light is absent. She has been suffering from epilepsy since childhood. Usually, seizures occur 1-2 times a month and are accompanied by involuntary urination and defecation.
Істеричний неврозHysterical neurosis
Епілептичний психозEpileptic psychosis
Гіпокальціємічний кризHypocalcemic crisis
--
Епілептичний статусStatus epilepticus
Терапія
269/1500
У хворого 46-ти років з болем стискального характеру в ділянці серця настала зупинка кровообігу та дихання. На ЕКГ моніторі: великохвильова фібриляція шлуночків. Що потрібно зробити першочергово?A 46-year-old patient with constrictive pain in the heart area has stopped blood circulation and breathing. On the ECG monitor: large-wave ventricular fibrillation. What should be done first?
Хлопчик 8-ми років, звернувся до лікаря зі скаргами на підвищену втомлюваність, зниження апетиту, іктеричність склер, біль у животі. У періоді новонародженості була затяжна жовтяниця. У фізичному розвитку не відстає. Об'єктивно: шкіра бліда, слизові оболонки і склери іктеричні. Печінка +2 см, злегка болюча при пальпації. Випорожнення і сечовиділення в нормі. При лабораторному дослідженні крові: еритроцити - 4,5·10^12/л, гемоглобін - 115 г/л, лейкоцити - 7·10^9/л, швидкість осідання еритроцитів - 8 мм/год, білірубін прямий - 10 мкмоль/л, непрямий - 39 мкмоль/л, аспартатамінотрансфераза (АСТ) - 17 Од/л, аланінамінотрансфераза (АЛТ) - 21 Од/л. Який діагноз є найбільш імовірним?An 8-year-old boy turned to the doctor with complaints of increased fatigue, decreased appetite, icterus of the sclera, abdominal pain. In the newborn period, he had prolonged jaundice. In Physical development is not lagging behind. Skin is pale, mucous membranes are icteric. Liver is slightly painful on palpation. In laboratory examination, erythrocytes are 4.5·12/l , hemoglobin - 115 g/l, leukocytes - 7·10^9/l, erythrocyte sedimentation rate - 8 mm/h, direct bilirubin - 10 μmol/l, indirect - 39 μmol/l, aspartate aminotransferase (AST) - 17 Units/ l, alanine aminotransferase (ALT) - 21 units/l. What is the most likely diagnosis?
У відділення надійшов хворий 30-ти років зі скаргами на часті рідкі випорожнення з кров'ю і слизом, підвищення температури до 37,7^oC, переймоподібний біль у животі. Пальпаторно відзначається болючість по ходу товстої кишки. При ректороманоскопії: стінка кишечника набрякла, легкоранима, ерозії і виразки, значна кількість крові та слизу. Імовірний попередній діагноз:A 30-year-old patient came to the department with complaints of frequent loose stools with blood and mucus, temperature rise to 37.7^oC, spasm-like pain in Abdominal tenderness is noted along the course of the colonoscopy: the intestinal wall is swollen, with erosions and ulcers. A preliminary diagnosis is:
У чоловіка 73 років, який упродовж 5 років хворіє на доброякісну гіперплазію простати, вранці настала гостра затримка сечі. Закатетеризувати сечовий міхур неможливо. Якою буде невідкладна допомога?A 73-year-old man who has been suffering from benign prostatic hyperplasia for 5 years has acute urinary retention in the morning. It is impossible to catheterize the bladder. What will be the emergency care?
Призначення спазмолітиків та аналгетиківPrescription of antispasmodics and analgesics
Призначення теплових процедурAssignment of thermal procedures
Призначення адреноблокаторівPrescription of adrenoblockers
Пункція сечового міхураBladder puncture
Призначення сечогінних препаратівPrescription of diuretics
Терапія
273/1500
Пацієнт віком 32 роки, дивлячись на малюнок на шпалерах, бачить, як лінії починають рухатися, утворюючи силуети химерних тварин. Замість люстри, що висить на стелі, бачить гігантського восьминога. Встановіть психопатологічний симптом.A 32-year-old patient, looking at a picture on the wallpaper, sees the lines begin to move, forming silhouettes of whimsical animals. Instead of a chandelier hanging from the ceiling, he sees a giant Octopus. Establish a psychopathological symptom.
До жіночої консультації звернулась породілля на 18-й день після пологів зі скаргами на біль та ущільнення у правій молочній залозі. Температура тіла - 38,5^oС. Під час огляду встановлено: у ділянці соска - тріщина, болючість під час пальпації. Яке ускладнення виникло в післяпологовому періоді?On the 18th day after giving birth, a woman in labor came to the women's consultation with complaints of pain and tightness in the right mammary gland. The body temperature is 38.5°C. Under at the time of the examination it was established: there was a crack in the area of the nipple, tenderness during palpation. What complication occurred in the postpartum period?
Рак молочної залозиBreast cancer
Кіста правої молочної залозиRight breast cyst
ЛактостазLactostasis
Додаткова долька молочної залозиAdditional breast lobe
Лактаційний маститLactation mastitis
Терапія
275/1500
Чоловік 48 років звернувся до лікаря зі скаргами на біль у попереку. Хворіє 3 дні після переохолодження. Після огляду лікар поставив діагноз: гострий попереково-крижовий радикуліт. Які ліки слід призначити хворому?A 48-year-old man went to the doctor with complaints of lower back pain. He has been sick for 3 days after hypothermia. After the examination, the doctor diagnosed acute lumbosacral sciatica. What medicines should be prescribed to the patient?
Хлопчик 15-ти років скаржиться на нападоподібний біль в животі під час дефекації, діарею до 6 разів на добу з домішками гною та темної крові у калових масах. Об'єктивно: відставання у фізичному та статевому розвитку. Шкіра бліда, суха. Живіт здутий, болісний біля пупка та у правій здухвинній ділянці. Припущено хворобу Крона. Яке дослідження треба провести для підтвердження діагнозу?A 15-year-old boy complains of acute abdominal pain during defecation, diarrhea up to 6 times a day with impurities of pus and dark blood in the feces. About' objectively: retardation in physical and sexual development. The skin is swollen, painful in the right navel area. What research should be done to confirm the diagnosis?
КопроцитограмаCoprocytogram
УЗД органів черевної порожниниUltrasound of abdominal organs
РектороманоскопіяRectoromanoscopy
КолоноскопіяColonoscopy
ФГДСFGDS
Хірургія
277/1500
Хвора 42-х років скаржиться на наяв-ність болючого утворення у лівій сідниці, підвищення температури тіла до 38,2^oC. Тиждень тому завершила курс лікування з приводу поперекового радикуліту. У верхньо-зовнішньому квадранті лівої сідниці гіперемія, набряк. При пальпації - різко болюче ущільнення до 6 см в діаметрі з розм'якшенням у центрі. Ваш попередній діагноз:A 42-year-old patient complains of the presence of a painful lump in the left buttock, an increase in body temperature to 38.2^oC. A week ago, she completed a course of treatment for of lumbar radiculitis. In the upper-external quadrant of the left buttock, there is a sharp, painful induration up to 6 cm in diameter. Your previous diagnosis:
Підшкірний парапроктитSubcutaneous paraproctitis
Абсцедивний фурункулAbscess boil
Нагноєна атеромаSuppurative atheroma
Карбункул сідниціCarbuncle of the buttock
Післяін'єкційний абсцесPost-injection abscess
Акушерство і гінекологія
278/1500
У породіллі вагою 70 кг за 40 хвилин після пологів великим плодом (4500 г) з'явилися значні кров'янисті виділення зі статевих шляхів. Проведено випорожнення сечового міхура та зовнішній масаж матки; після чого матка скоротилася, але за 5 хвилин кровотеча відновилася. Після введення утеротоніків, ручного обстеження порожнини матки крововтрата склала 1200 мл. Якою повинна бути подальша тактика?In a woman in labor weighing 70 kg, 40 minutes after giving birth to a large fetus (4500 g), significant bloody discharge appeared from the genital tract. The bladder was emptied and external massage of the uterus, after which the bleeding resumed in 5 minutes. After the administration of the uterine cavity, the blood loss was 1200 ml. What should be the next tactics?
Кюретаж стінок порожнини маткиCurettage of the walls of the uterine cavity
Продовжити введення утеротоніківContinue administration of uterotonics
Компресія черевного відділу аортиCompression of the abdominal aorta
Локальна гіпотерміяLocal hypothermia
Лапаротомія. Екстирпація матки без додатківLaparotomy. Extirpation of the uterus without attachments
Педіатрія
279/1500
У дитини 4,5 років з вираженими ознаками зневоднення свідомість на рівні сопору, загальний ціаноз, токсичне дихання, анурія, АТ- 60/20 мм рт.ст., ембріокардія, відсутність пульсу на променевій артерії. Який болюс розчину NaCl 0,9% необхідно ввести на першому етапі невідкладної допомоги?A 4.5-year-old child with pronounced signs of dehydration has consciousness at the level of sedation, general cyanosis, toxic breathing, anuria, blood pressure - 60/20 mm Hg. , embryocardia, absence of a pulse on the radial artery. What bolus of NaCl solution 0.9% should be administered at the first stage of emergency care?
100 мл/кг100 ml/kg
30 мл/кг30 ml/kg
60 мл/кг 60 ml/kg
50 мл/кг50 ml/kg
80 мл/кг80 ml/kg
Терапія
280/1500
Дівчина 22 років, хвора на системний червоний вовчак, скаржиться на лихоманку, схуднення, біль у дрібних суглобах кистів. Об'єктивно встановлено: дифузна алопеція, генералізована лімфаденопатія, еритематозна висипка на щоках та спинці носа, ексудативний плеврит та перикардит. Температура тіла - 38,4^oC. У крові виявлено: лейкоцити - 3,1·10^9/л, Нb - 102 г/л, ШОЕ - 56 мм/год, С-реактивний протеїн - (++++). Яка доза преднізолону є доцільною у цьому разі?A 22-year-old girl with systemic lupus erythematosus complains of fever, weight loss, pain in the small joints of the hands. Objectively established: diffuse alopecia, generalized lymphadenopathy, erythematous rash on the cheeks and back of the nose, exudative pleurisy and pericarditis. Body temperature - 38.4°C. Leukocytes - 3.1·10^9/l, Hb - 102 g/l, ESR - 56 mm/l. hours, C-reactive protein - (++++). What dose of prednisolone is appropriate in this case?
0,25 мг/кг0.25 mg/kg
0,75 мг/кг0.75 mg/kg
0,5 мг/кг0.5 mg/kg
1 мг/кг1 mg/kg
0,1 мг/кг0.1 mg/kg
Педіатрія
281/1500
У 9-місячної дитини ВІЛ-інфіко-ваної жінки визначені гіпотрофія 2-го ступеня, полілімфаденопатія, гепатоспленомегалія, рецидивуючий орофарингеальний кандідоз, гіпохромна анемія (Нb - 80 г/л), нейтропенія; виявлено ДНК ВІЛ у крові. Поставте діагноз:In a 9-month-old child of an HIV-infected woman, hypotrophy of the 2nd degree, polylymphadenopathy, hepatosplenomegaly, recurrent oropharyngeal candidiasis, hypochromic anemia (Hb - 80 g /l), neutropenia; HIV DNA was detected in the blood. Diagnose:
Безсимптомне носійство ВІЛAsymptomatic HIV carrier
ВІЛ-серопозитивна дитинаHIV-seropositive child
Хірургія
282/1500
У чоловіка 52 рокiв, який страждає на постхолецистектомiчний синдром, під час ретроградної холецистопанкреатографії виявлено стеноз сфiнктера Оддi з помiрною дилатацiею загального жовчного протока i рефлюксом контраста у Вiрсунгiв проток. Конкрементiв у протоках немає. Дiагноз хронiчного панкреатита безсумнiвний. Яка операцiя адекватна для корекцiї цієї патологiї?In a 52-year-old man suffering from post-cholecystectomy syndrome, stenosis of the sphincter of Oddi with moderate dilatation of the common bile duct and reflux of contrast into the duct of Wirsung was revealed during retrograde cholecystopancreatography. there are no ducts. The diagnosis of chronic pancreatitis is unquestionable. What operation is adequate for the correction of this pathology?
У пацієнта віком 22 роки об'єктивно спостерігається: на шкірі грудей, шиї, бокових поверхонь тулуба, плечей - чисельні незапальні плями жовтувато-коричневого та блідо-рожевого кольору, у разі пошкрябування яких виявляється 'симптом стружки'. Хворіє протягом 2-х років, висипи помітив після літнього відпочинку на морі. Який найімовірніший діагноз?A 22-year-old patient objectively observes: on the skin of the chest, neck, lateral surfaces of the body, shoulders - numerous non-inflammatory spots of yellowish-brown and pale pink color , in the case of scraping, it turns out to be a 'chip symptom'. He has been sick for 2 years, he noticed rashes after a summer vacation at the sea. What is the most likely diagnosis?
ЕритразмаErythrasma
Рожевий лишай ЖибераZhiber's pink lichen
Висівкоподібний лишайLichen bran
Сифілітична розеолаSyphilitic roseola
СтрептодерміяStreptoderma
Хірургія
284/1500
Пацієнт віком 32 роки під час фізичних вправ зненацька відчув нестачу дихання, слабкість, біль в правій половині грудей з іррадіацією в праве плече, задишку, серцебиття. Об'єктивно спостерігається: стан тяжкий, тахікардія до 100/хв, АТ - 90/60 мм рт. ст. ЧД - 28/хв, права половина грудної клітки відстає при диханні. Перкуторно справа тимпанічний звук, дихальних шумів немає. Температура тіла - нормальна. Який найімовірніший діагноз?A 32-year-old patient suddenly experienced shortness of breath, weakness, pain in the right half of the chest with radiation to the right shoulder, shortness of breath, palpitations during physical exercises. Objectively the condition is severe, BP - 90/60 mmHg, right sided chest sound, no respiratory sounds the most likely diagnosis?
ПневмоніяPneumonia
Інфаркт легеніPulmonary infarction
Спонтаний пневмотораксSpontaneous pneumothorax
Судинний колапсVascular collapse
Інфаркт міокардаMyocardial infarction
Терапія
285/1500
У сухому піщаному ґрунті був знайдений труп чоловіка 45-50 років, шкіра якого буро-брунатного кольору, пергаментної щільності. Під час обстеження трупа встановлено: травматичні ушкодження відсутні, довжина тіла 172 см, внутрішні органи зменшені, сухі, безформні, окремі органи плівчасті. В якому стані трупних змін перебуває труп?The corpse of a 45-50-year-old man was found in the dry sandy soil, the skin of which is brownish-brown in color, parchment-like. During the examination of the corpse, it was found: there are no traumatic injuries, body length 172 cm, internal organs are reduced, dry, shapeless, some organs are membranous. What state of cadaveric changes is the corpse in?
МуміфікаціїMummifications
ГниттяRot
Торф'яного дубленняPeat tanning
Штучного консервуванняArtificial preservation
ЖировоскуTallow wax
Педіатрія
286/1500
Мати 22-місячного хлопчика звернулася до лікаря зі скаргами на погане набирання ваги та худобу у дитини. Годує грудним молоком на вимогу, не менше 5 разів на день. Вага та зріст хлопчика нижче 3-го стандартного відхилення для його віку. У нього дугоподібна деформація ніг та потовщені зап'ястя. При лабораторному дослідженні підвищений рівень лужної фосфатази. Дефіцит якого вітаміну найбільш імовірно спричинив подібний стан у дитини?The mother of a 22-month-old boy went to the doctor with complaints of poor weight gain and cattle in the child. Breastfeeds on demand, at least 5 times a day. Weight and the boy's height is below the 3rd standard deviation. He has bowed legs and thickened wrists. On laboratory examination, the level of alkaline phosphatase is most likely to cause this condition in the child?
Вітаміну EVitamin E
Вітаміну AVitamin A
Вітаміну DVitamin D
Вітаміну B_6Vitamin B_6
Вітаміну PPVitamin PP
Терапія
287/1500
У лісі виявлено труп чоловіка 35 років, який висів у петлі на дереві. Тканини обличчя та шиї синюшного забарвлення, нижні кінцівки звичайного забарвлення. Странгуляційна борозна добре виражена, замкнута. Установити причину та рід смерті:The corpse of a 35-year-old man was found in the forest, hanging from a noose on a tree. The tissues of the face and neck are bluish in color, the lower limbs are of a normal color. The strangulation furrow is well defined, closed To establish the cause and type of death:
Механічна асфіксія внаслідок задушення петлею, рід смерті - ненасильницькаMechanical asphyxiation due to strangulation with a noose, type of death - non-violent
Механічна асфіксія внаслідок здавлення грудей і живота, рід смерті - насильницькаMechanical asphyxiation due to compression of the chest and abdomen, the manner of death is violent
Механічна асфіксія внаслідок задушення петлею, рід смерті - насильницькаMechanical asphyxiation due to strangulation with a noose, type of death - violent
Механічна асфіксія внаслідок закриття дихальних шляхів, рід смерті - насильницькаMechanical asphyxiation due to closing of the respiratory tract, type of death - violent
Механічна асфіксія внаслідок повішення, рід смерті - ненасильницькаMechanical asphyxiation due to hanging, type of death - non-violent
Терапія
288/1500
64-річний чоловік прийшов до лікаря на щорічний профілактичний огляд. Скарг не має. Хворіє на артеріальну гіпертензію, ХОЗЛ та глаукому. Викурює одну пачку сигарет на день протягом 30-ти років (індекс паління - 30 пачко/років). При фізикальному обстеженні: температура - 37,2^oC, артеріальний тиск - 140/70 мм рт.ст., пульс - 79/хв., частота дихання - 16/хв., пряма, співдружня реакція зіниць на світло. При аус-культації серця акцент II тону над аортою, аускультація легень без патологічних змін, грудна клітка ''бочкоподібна''. Який з перерахованих методів скринінгу є найбільш доречним для цього пацієнта?A 64-year-old man came to the doctor for an annual preventive examination. He has no complaints. He suffers from arterial hypertension, COPD and glaucoma. He smokes one pack of cigarettes a day for 30 years (smoking index - 30 pack/years). During physical examination: temperature - 37.2°C, blood pressure - 140/70 mm Hg, pulse - 79/min, respiratory rate - 16/min ., direct pupillary reaction to light. During auscultation of the heart, the accent of the II tone over the aorta, auscultation of the lungs is 'barrel-shaped'. Which of the listed screening methods is the most appropriate for this patient?
Легеневі функціональні тестиPulmonary function tests
Бронхоальвеолярний лаважBronchoalveolar lavage
МРТ легеньLung MRI
Низькодозова КТ легеньLow-dose lung CT
Рентгенографія ОГКRoentgenography of OGK
Терапія
289/1500
Чоловік віком 63 роки звернувся до лікаря зі скаргами на виражену загальну слабкість, поганий апетит, схуднення, біль в суглобах, відчуття важкості в правому підребер'ї. У клінічному аналізі крові спостерігається: еритроцити - 3,4·10^12/л, Hb - 102 г/л, КП - 0,9, тромбоцити - 640·10^9/л, лейкоцити - 138·10^9/л, бласти -1 %, промієлоцити - 2 %, мієлоцити - 13 %, юні - 12 %, паличкоядерні - 16 %, сегментоядерні - 31 %, базофіли - 3 %, еозинофіли - 8 %, лімфоцити - 9 %, моноцити - 5 %, ШОЕ - 30 мм/год. Який попередній діагноз?A 63-year-old man consulted a doctor with complaints of severe general weakness, poor appetite, weight loss, pain in the joints, a feeling of heaviness in the right hypochondrium. In the clinical blood analysis shows: erythrocytes - 3.4·10^12/l, Hb - 102 g/l, CP - 0.9, platelets - 640·10^9/l, leukocytes - 138·10^9/l, blasts -1%, promyelocytes - 2%, myelocytes - 13%, young - 12%, rod-nuclear - 16%, segmentonuclear - 31%, basophils - 3%, eosinophils - 8%, lymphocytes - 9%, monocytes - 5%, ESR - 30 mm/h. What is the previous diagnosis?
ЕритреміяErythremia
Лейкемоїдна реакціяLeukemoid reaction
Хронічний мієлолейкозChronic myelogenous leukemia
Гострий лейкозAcute leukemia
Хронічний лімфолейкозChronic lymphocytic leukemia
Терапія
290/1500
Робітник цеху з виробництва акумуляторів для автомобілів звернувся до лікаря зі скаргами на нудоту, втрату апетиту, різкий біль у черевній порожнині, закрепи. Під час огляду лікар виявив підвищення артеріального тиску, брадикардію, збільшення печінки, біль в правому підребер'ї, сірувато-синю смужку на яснах, сірий колір шкірних покривів. Результати лабораторного аналізу крові свідчать про наявність еритроцитів з базофільною зерністістю та знижений вміст гемоглобіну, а у сечі виявлено амінолевулінову кислоту і копропорфірин. Який попередній діагноз найімовірніший?A worker at a workshop for the production of batteries for cars turned to the doctor with complaints of nausea, loss of appetite, sharp pain in the abdominal cavity, constipation. During the examination, the doctor found an increase in blood pressure pressure, bradycardia, liver enlargement, pain in the right hypochondrium, grayish-blue stripe on the gums, gray color of the skin. The results of the laboratory blood test indicate the presence of erythrocytes with basophilic granularity and a reduced hemoglobin content, and aminolevulinic acid and coproporphyrin were detected in the urine . What is the most likely preliminary diagnosis?
Отруєння алюмініємAluminum poisoning
Отруєння кадміємCadmium poisoning
Отруєння ртуттюMercury poisoning
Харчове отруєнняFood poisoning
Отруєння свинцемLead poisoning
Терапія
291/1500
Жінка, яка хворіє на полікістоз нирок, спостерігає зростання добового діурезу до 2-2,5 л. За даними динамічної нефросцинтиграфії виявлено, що загальна клубочкова фільтрація становить 34 мл/хв., креатинін сироватки крові - 84 мкмоль/л, сечовина - 8,0 ммоль/л. Яка стадія хронічної ниркової недостатності у пацієнтки?A woman suffering from polycystic kidney disease observes an increase in daily urine output to 2-2.5 liters. According to dynamic nephroscintigraphy, it was found that the total glomerular filtration rate is 34 ml /min., serum creatinine - 84 μmol/l, urea - 8.0 mmol/l. What is the patient's stage of chronic renal failure?
КомпенсованаCompensated
ІнтермітуючаIntermittent
ПоліуричнаPolyuric
ЛатентнаLatent
ТермінальнаTerminal
Педіатрія
292/1500
При проведенні медичного огляду учнів середнього та старшого шкільного віку лікарі визначали відповідність біологічного розвитку та календарного віку за наступними критеріями: щорічне збільшення довжини тіла, осифікація кісток кисті, кількість постійних зубів. Який додатковий показник розвитку у ці вікові періоди найбільш імовірно мають включити лікарі?During the medical examination of middle and high school students, doctors determined the correspondence of biological development and calendar age according to the following criteria: annual increase in body length, ossification of hand bones, number of permanent teeth. What additional developmental indicator should doctors most likely include in these age periods?
М'язова сила кистіHand muscle strength
Життєва ємність легеньVital lung capacity
Розвиток вторинних статевих ознакDevelopment of secondary sexual characteristics
Маса тілаBody weight
Обвід грудної кліткиChest Circumference
Терапія
293/1500
Дівчина 21-го року звернулася до лікаря з грипоподібною картиною захворювання, субфебрильною температурою, нездужанням та легкою жовтяницею протягом 2-х днів. Лабораторні показники сироватки крові: аспартатамінотрансфераза (АСТ) - 456 Од/л, аланінамінотрансфераза (АЛТ) - 745 Од/л, IgM до HAV - позитивний результат. Яка тактика по відношенню до контактних осіб буде найбільш доречною?A 21-year-old girl consulted a doctor with a flu-like illness, low-grade fever, malaise, and mild jaundice for 2 days. Serum laboratory parameters: aspartate aminotransferase ( AST) - 456 units/l, alanine aminotransferase (ALT) - 745 units/l, IgM to HAV - a positive result. What tactics will be most appropriate in relation to contact persons?
Профілактичний прийом софосбувіру/ледіпасвіру протягом 1 місяцяProphylactic sofosbuvir/ledipasvir for 1 month
Введення інтерферону альфа-2b протягом першого тижняIntroduction of interferon alfa-2b during the first week
Введення імуноглобуліну людини нормального протягом 24 годинInjection of normal human immunoglobulin within 24 hours
Вакцинація від гепатиту В протягом першого тижняHepatitis B vaccination during the first week
Вакцинація від гепатиту А протягом першого тижняVaccination against hepatitis A during the first week
Терапія
294/1500
Контроль міжнародного нормалізованого співвідношення (МНС) потрібно проводити у пацієнтів із фібриляцією передсердь, які приймають препарат із групи антитромботичних засобів. Укажіть цей препарат.International normalized ratio (INR) control should be performed in patients with atrial fibrillation who are taking a drug from the group of antithrombotic drugs. Specify this drug.
Варфарин (антагоніст вітаміну К)Warfarin (vitamin K antagonist)
Ривароксабан (інгібітори фактора Ха)Rivaroxaban (factor Xa inhibitors)
Чоловік 31-го року доставлений до відділення невідкладної допомоги зі скаргами на утруднене дихання, відчуття стороннього тіла в горлі, осиплість, сильний набряк обличчя та шиї. Вищевказані симптоми швидко наростають та з'явились протягом декількох хвилин після куштування меду. Який стан найбільш імовірно розвинувся у хворого?A 31-year-old man was brought to the emergency department with complaints of difficulty breathing, feeling of a foreign body in the throat, dizziness, severe swelling of the face and neck. The above symptoms quickly build up and appeared within a few minutes after tasting the honey. Which condition most likely developed in the patient?
Гострий ларинготрахеїтAcute laryngotracheitis
Запальний набряк гортаніInflammatory swelling of the larynx
Ангіоневротичний набряк КвінкеAngioedema of Quincke
Педіатрія
296/1500
Хвора 12 років із хронічним панкреатитом, період реконвалесценції. Стоїть питання про подальше продовження замісної терапії препаратом панкреатину та розширення режиму харчування. За яким методом дослідження можна найкоректніше оцінити екзокринну функцію підшлункової залози?12-year-old patient with chronic pancreatitis, convalescent period. There is a question about further continuation of pancreatin replacement therapy and expansion of the diet. Which research method can be used to assess exocrine function most correctly pancreas?
Визначення рівня амілази в сироватці крові та діастази в сечіDetermining the level of amylase in blood serum and diastase in urine
Рівень стеатореї під час копро логіч-ного дослідженяLevel of steatorrhea during coprologic examination
Визначення еластази-1 у каліDetermination of elastase-1 in feces
Визначення рівня трипсину в сироватці кровіDetermination of trypsin level in blood serum
Визначення еластази-2 у сироватці кровіElastase-2 determination in blood serum
Терапія
297/1500
У хворого, що 1 рік тому переніс Q-інфаркт міокарда задньої стінки лівого шлуночка, протягом останніх 2-х тижнів щодня виникають напади фібриляції передсердь та епізоди брадикардії, що супроводжуються запамороченнями. Яка найбільш доцільна тактика?A patient who suffered a Q-myocardial infarction of the posterior wall of the left ventricle 1 year ago, during the last 2 weeks, attacks of atrial fibrillation and episodes of bradycardia occur every day, which are accompanied by dizziness. What is the most appropriate tactic?
Призначити дигоксинPrescribe digoxin
Імплантація кардіостимулятораImplantation of a pacemaker
Призначити новокаїнамідPrescribe novocaine
Призначити бісопрололPrescribe bisoprolol
Призначити аміодаронPrescribe amiodarone
Терапія
298/1500
Протягом останніх трьох років хвора жінка страждає на цукровий діабет. Сімейний лікар, з метою профілактики ускладнень цукрового діабету, регулярно призначає їй превентивне лікування. До якого виду профілактики належать вказані заходи?For the past three years, a sick woman has been suffering from diabetes. The family doctor, in order to prevent complications of diabetes, regularly prescribes preventive treatment for her. What kind of prevention do the specified measures?
Громадська профілактикаPublic prevention
Первинна профілактикаPrimary prevention
Індивідуальна профілактикаIndividual prevention
Вторинна профілактикаSecondary prevention
Третинна профілактикаTertiary prevention
Терапія
299/1500
Хворому 65-ти років встановлено діагноз рак головного мозку IV стадії. Чоловік з вираженими епілептичними нападами та больовим синдромом. Який вид медичної допомоги необхідний для полегшення стану хворого?A 65-year-old patient has been diagnosed with stage IV brain cancer. A man with severe epileptic seizures and pain syndrome. What type of medical care is needed to alleviate the patient's condition?
Санаторно-курортна допомогаSanatorium and resort assistance
Третинна медична допомогаTertiary medical care
Первинна медична допомогаPrimary medical care
Паліативна медична допомогаPalliative medical care
Вторинна медична допомогаSecondary medical care
Педіатрія
300/1500
У новонародженого на п'ятий день життя відмічається надмірне здуття живота з контуруванням кишкових петель на передній черевній стінці. Меконій відійшов після клізми. На оглядовій рентгенограмі органів черевної порожнини було виявлено розширення петель товстого кишківника. Яка вада розвитку ймовірно у дитини?A newborn on the fifth day of life has excessive abdominal distension with contouring of intestinal loops on the anterior abdominal wall. Meconium has passed after an enema. On the X-ray examination of the abdominal organs, there was the expansion of the loops of the large intestine was detected. What malformation is likely in the child?
Стеноз клубової кишкиIleal stenosis
Інвагінація кишківникаIntussusception
Хвороба ГіршпрунгаHirschsprung's disease
Портальна гіпертензіяPortal hypertension
Синдром ЛедаLed syndrome
Гігієна
301/1500
Для характеристики стану здоров’я населення в районі розраховувалися показники народжуваності, смертності, природного приросту, поширеності хвороб і первинної захворюваності, загальної інвалідності та інвалідизації населення. До якого виду статистичних величин відносяться ці показники?In order to characterize the state of health of the population in the district, indicators of birth rate, mortality, natural increase, prevalence of diseases and primary morbidity, general disability and disability of the population were calculated. To what type statistical values to which these indicators belong?
ЕкстенсивнихExtensive
НаочностіVisibility
СтандартизованихStandardized
СпіввідношенняRatio
ІнтенсивнихIntensive
Хірургія
302/1500
Хлопчик 9-ти років упав з дерева, вдарився потиличною ділянкою, спостерігалась короткочасна втрата свідомості. Стан дитини задовільний, турбують головний біль, запаморочення. На рентгенограмах черепа виявлено втиснений уламчастий перелом потиличної кістки в ділянці потиличного горба. Яка лікувальна тактика показана хворому?A 9-year-old boy fell from a tree, hit the occipital region, a short-term loss of consciousness was observed. The child's condition is satisfactory, he is bothered by headache, dizziness. X-rays of the skull revealed an imprinted fragmentary fracture of the occipital bone in the area of the occipital hump. What treatment tactics are indicated for the patient?
Дитина віком 6 років захворіла гостро: підвищилася температура тіла, з'явилися головний біль і біль під час ковтання. Через 3 години з'явилися яскраво-червоний дрібно-крапчастий висип на гіперемованій шкірі, рясніший на боковій поверхні тулуба та в природних складках, відмежована гіперемія слизової оболонки ротоглотки, гнійний наліт на мигдаликах. Яке захворювання можна припустити у дитини?A 6-year-old child became acutely ill: body temperature rose, headache and pain during swallowing appeared. After 3 hours, bright red small spotted rash on the hyperemic skin, more abundant on the lateral surface of the body and in the natural folds, limited hyperemia of the mucous membrane of the oropharynx, purulent plaque on the tonsils. What disease can be assumed in the child?
ДифтеріяDiphtheria
КраснухаKrasnukha
КірMeasles
Інфекційний мононуклеозInfectious mononucleosis
СкарлатинаScarlatina
Педіатрія
304/1500
Дитині 7 років. Скаржиться на біль у животі нападоподібного характеру, який виникає після психічного навантаження, уживання холодних напоїв, морозива. Після клініко-інструментального обстеження поставлено діагноз: дискінезія жовчного міхура за гіпертонічним типом. Які препарати слід призначати для лікування насамперед?The child is 7 years old. He complains of abdominal pain of an attack-like nature, which occurs after mental stress, consumption of cold drinks, ice cream. After a clinical and instrumental examination, the diagnosis was made: dyskinesia gall bladder according to the hypertensive type. What drugs should be prescribed for treatment first of all?
Холеретики та холекінетикиCholeretics and cholekinetics
Спазмолітики та холеретикиSpasmolytics and choleretics
АнтибіотикиAntibiotics
Седативні та холекінетикиSedatives and cholekinetics
АнтиоксидантиAntioxidants
Педіатрія
305/1500
У хлопчика 3-х років під час ходьби та бігу збільшується права половина мошонки, яка в положенні лежачи зменшує-ться до нормальних розмірів. Діаг-ноз:In a 3-year-old boy, the right half of the scrotum increases during walking and running, which decreases to normal size when lying down. Diagnosis:
Сполучна водянка оболонок правого яєчкаConjunctive dropsy of the membranes of the right testicle
ПарафімозParaphimosis
ОрхіепідидимітOrchiepididymitis
ЦистоцелеCystocele
ФімозPhimosis
Терапія
306/1500
Пацієнт віком 38 років скаржиться на кашель з виділенням гнійного мокротиння (до 60-80 мл на добу), підвищення температури тіла до 39^oС. Захворювання пов'язує з переохолодженням. Об'єктивно спостерігається: пульс - 96/хв, ритмічний. Артеріальний тиск - 110/60 мм рт. ст. Під час огляду виявлено: відставання правої половини в диханні. ЧД - 30/хв. Перкуторно локальне притуплення біля кута лопатки. Під час аускультації вислуховується: вологі різнокаліберні хрипи, амфоричне дихання. Який найімовірніший діагноз?A 38-year-old patient complains of a cough with the release of purulent sputum (up to 60-80 ml per day), an increase in body temperature to 39°C. The disease is associated with Objectively observed: pulse - 110/60 mm Hg. During the examination, the right side was found to be dull - 30/min . During auscultation: moist rales of various calibers, amphoric breathing are heard. What is the most likely diagnosis?
Вогнищева пневмоніяFocal pneumonia
Гострий бронхітAcute bronchitis
Емпієма плевриEmpyema of the pleura
Бронхіальна астмаBronchial asthma
Гострий абсцес легеніAcute lung abscess
Педіатрія
307/1500
Одинадцятирічна дівчинка скаржиться на біль у суглобах, підвищення температури тіла до 38^oС, слабкість. Захворіла 5 днів тому. Об'єктивно спостерігається: плямисто-папульозний висип на переніссі та обличчі, суглоби не змінені, тони серця послаблені. АТ - 100/80 мм рт. ст. У загальному аналізі крові: еритроцити - 2,6 т/л, лейкоцити - 3,7 г/л, тромбоцити - 12г/л, ШОЕ - 45 мм/год, СРБ (+++). У загальному аналізі сечі: білок - 0,063 г/л, еритроцити - 15-20 в п/з, лейкоцити - 10-14 в п/з. Вкажіть імовірний діагноз.An eleven-year-old girl complains of pain in the joints, an increase in body temperature up to 38°C, weakness. She fell ill 5 days ago. Objectively observed: a spotted-papular rash on nose and face, the joints are not changed, blood pressure - 100/80 mm Hg: erythrocytes - 3.7 g/l, platelets - 12 g/l. , ESR - 45 mm/h, CRP (+++). In the general analysis of urine: protein - 0.063 g/l, erythrocytes - 15-20 in p/z. Specify the probable diagnosis .
Громадянин заключив декларацію з сімейним лікарем приватного закладу. Для проходження чергового медичного огляду, лікар дав направлення хворому на біохімічний аналіз крові. Хто є платником медичної послуги у цьому разі?The citizen made a declaration with the family doctor of a private institution. To undergo another medical examination, the doctor referred the patient for a biochemical blood test. Who is the payer of the medical service in this case?'
НСЗУNSZU
Спеціальний фонд лікарніSpecial hospital fund
Благодійні внескиCharity contributions
ПацієнтPatient
За рахунок місцевого бюджетуAt the expense of the local budget
Терапія
309/1500
У пацієнта протягом декількох днів з'явилося обмеження периферичного зору ззовні, схоже на заслінку. Причину не знає. Око спокійне, оптичні середовища прозорі. Під час офтальмоскопії з носової сторони при рухах ока коливається сірий «парус» із судинами. Диск зорового нерва та судини не змінені. Який діагноз найімовірніший?For several days, the patient had a limitation of peripheral vision from the outside, similar to a valve. The cause is unknown. The eye is calm, the optical media is transparent. During ophthalmoscopy from the nose sides with eye movements, a gray 'sail' with vessels. The optic nerve disc and vessels are not changed. What is the most likely diagnosis?
Підвивих кришталикаSubluxation of the lens
Початкові прояви глаукомиInitial manifestations of glaucoma
Геміанопсія при неврологічній патологіїHemianopsia in neurological pathology
Пацієнта віком 30 років шпиталізовано до лікарні. З анамнезу відомо що пацієнт захворів гостро, хвороба почалась з частих водянистих випорожнень, потім з'явилось рясне блювання без нудоти, спрага. Об'єктивно спостерігається: температура тіла - 35,4^oC. Стан важкий, шкіра холодна. Тургор тканин та еластичність шкіри значно знижені. Риси обличчя загострені. Голос сиплий, акроціаноз. Пульс - 130/хв, слабкий. АТ - 60/30 мм рт. ст. Язик сухий. Живіт запалий, неболючий, під час пальпації виражене гуркотіння в кишечнику. Анурія. Який найімовірніший діагноз?A 30-year-old patient was admitted to the hospital. It is known from the anamnesis that the patient became acutely ill, the disease began with frequent watery stools, then profuse vomiting appeared without nausea, thirst Objectively observed: body temperature - 35.4 °C, skin turgor is increased. Voice - 130/min. 30 mm Hg. The tongue is inflamed, during palpation there is pronounced rumbling. What is the most likely diagnosis?
Чоловік 45-ти років, звернувся до лікаря зі скаргами на дратівливість, підвищену втомленість, схуднення, серцебиття, перебої в роботі серця. При пальпації лівої долі щитоподібної залози пальпується утворення, щільно-еластичної консистенції, безболісне, яке при ковтанні зміщується разом з щитоподібною залозою. Позитивні очні симптоми, екзофтальм, порушення конвергенції. Артеріальний тиск - 135/80 мм рт.ст., пульс - 110/хв. При ультразвуковому дослідженні щитоподібної залози у лівій долі візуалізується гіперехогенне округле утворення розмірами 2х3 см, з чіткими контурами, однорідної структури. Який висновок лікаря після проведеного обстеження буде найбільш коректним?A 45-year-old man consulted a doctor with complaints of irritability, increased fatigue, weight loss, palpitations, interruptions in the work of the heart. When palpating the left lobe of the thyroid gland, palpable formation, of a dense and elastic consistency, which is displaced with the thyroid gland. Positive eye symptoms, violation of convergence. Arterial pressure - 135/80 mm Hg. During ultrasound examination of the thyroid gland In the left lobe, a 2x3 cm hyperechoic mass with clear contours and uniform structure is visualized. What conclusion of the doctor will be the most correct after the examination?
Дифузно-токсичний зобDiffuse toxic goiter
Кіста щитоподібної залозиThyroid cyst
Рак щитоподібної залозиThyroid cancer
Гострий тиреоїдитAcute thyroiditis
Вузол щитоподібної залозиThyroid nodule
Хірургія
312/1500
Пацієнт госпіталізований у нейрохірургічне відділення з приводу закритої черепно-мозкової травми, перелому скроневої кістки справа. Через 5 годин його стан різко погіршився, виникли порушення дихання, періодичні тонічні судоми, анізокорія (розширення правої зіниці). Яке ускладнення можна припустити у пацієнта?The patient was hospitalized in the neurosurgery department due to a closed craniocerebral injury, a fracture of the temporal bone on the right. After 5 hours, his condition deteriorated sharply, breathing disorders, periodic tonic convulsions occurred , anisocoria (enlargement of the right pupil). What complications can be assumed in the patient?
Особливості перебігу струсу головного мозкуFeatures of concussion
Ішемічний інсульт в ділянці правої ніжки мозкуIschemic stroke in the area of the right leg of the brain
Абсцес головного мозкуBrain abscess
Виникнення епідуральної гематомиEmergence of epidural hematoma
Педіатрія
313/1500
Новонароджена доношена дитина з масою тіла 4500 г народилася в асфіксії з оцінкою за шкалою Апгар 4-6 балів. В пологах утруднене виведення плечового поясу. У неврологічному статусі: загальномозкові розлади, виявлений тотальний верхній млявий парез - ручка атонічна, пронована, не викликаються рефлекси - хапальний, Бабкіна. Вкажіть рівень ураження спинного мозку:A newborn full-term child with a body weight of 4500 g was born asphyxiated with an Apgar score of 4-6 points. During delivery, the removal of the shoulder girdle was difficult. In the neurological status: cerebral disorders, a total upper limb paresis was detected - the handle is atonic, pronated, reflexes are not evoked - grasping, Babkin. Specify the level of spinal cord damage:
Грудні сегменти TVI-TVІІThoracic segments TVI-TVII
Шийні сегменти СIII-С IVCervical segments CIII-C IV
Шийні сегменти СI-СIICervical segments CI-CII
Педіатрія
314/1500
У дівчинки 12 років упродовж 2 місяців відзначається періодичне підвищеня температури тіла до 39^oС, спостерігаються веретеноподібна припухлість міжфалангових суглобів, біль у верхній частині грудини і шиї, скутість зранку. Який діагноз найімовірніший?A 12-year-old girl has had a periodic increase in body temperature up to 39°C for 2 months, spindle-shaped swelling of the interphalangeal joints, pain in the upper part of the sternum and neck, morning stiffness . What is the most likely diagnosis?
Після видобування корисних копалин і швидкому піднятті на поверхню з глибини 80 м працівники відчули сильні болі у суглобах і м'язах, м'язах живота, парез нижніх кінцівок, судоми. Укажіть ймовірне захворювання:After extracting minerals and quickly rising to the surface from a depth of 80 m, the workers felt severe pains in the joints and muscles, abdominal muscles, paresis of the lower limbs, convulsions. Specify the probable disease:
Горна хворобаMountain sickness
Кесонна хворобаcaisson disease
Шумова хворобаNoise sickness
Висотна хворобаAltitude sickness
Тромбоемболія судин нижніх кінцівокThromboembolism of vessels of the lower extremities
Педіатрія
316/1500
Хлопчик 3-х років надійшов з вираженим набряковим синдромом. Об'єктивно: блідий. Артеріальний тиск - 90/60 мм рт.ст. Олігурія. Загальний аналіз сечі: колір - жовтий, відносна густина - 1020, білок - 3,5 г/л, еритроцити - вилужені, 4-5 в п/з, лейкоцити - 2-3 в п/з. Добова протеїнурія - 6,6 г. Холестерин - 9,8 ммоль/л. Який наступний крок у веденні пацієнта буде найбільш доречним?A 3-year-old boy was admitted with severe edematous syndrome. Objectively: pale. Blood pressure - 90/60 mm Hg. Oliguria. General analysis of urine : color - yellow, relative density - 1020, protein - 3.5 g/l, erythrocytes - leached, 4-5 in p/z, leukocytes - 2-3 in p/z. Daily proteinuria - 6.6 g. Cholesterol - 9.8 mmol/l. What would be the most appropriate next step in the management of the patient?
Проба ЗимницькогоZymnytsky's trial
Комп'ютерна томографія без контрастуComputed tomography without contrast
Біопсія ниркиKidney biopsy
Аналіз сечі за НечипоренкоUrine analysis according to Nechiporenko
Спостереження протягом тижняObservations during the week
Терапія
317/1500
У хворого 40-ка років ранковий кашель з виділенням слизово-гнійного харкотиння, підвищення температури тіла до 37,6^oC. Тютюнопаління з 17-ти років. Об'єктивно: при аускультації в легенях жорстке дихання, розсіяні сухі хрипи. ЗАК: Л- 12 Г/л, ШОЕ- 18 мм/год. При бронхоскопії: катарально-гнійний ендобронхіт. Поставте діагноз:A 40-year-old patient has a morning cough with mucus-purulent sputum discharge, an increase in body temperature to 37.6^oC. He has been smoking since the age of 17. About Objectively: during auscultation in the lungs, scattered dry wheezes. АК - 12 G/l. At bronchoscopy: make a diagnosis of purulent endobronchitis.
Для проведення інтенсивної інфузійної терапії пацієнту з гострою дихальною недостатністю виконано катетеризацію підключичної вени за Сельдінгером. Після введення 600 мл інфузійного розчину стан пацієнта різко погіршився, збільшилося тахіпное з 26 до 40/хв., ДО зменшився з 400 мл до 250 мл. Під час аускультації дихання справа різко ослаблене. Перкуторно спостерігається тупість звуку. Яке ускладнення виникло у пацієнта?For intensive infusion therapy, a patient with acute respiratory failure underwent Seldinger's subclavian vein catheterization. After administration of 600 ml of infusion solution, the patient's condition worsened sharply, tachypnea increased from 26 to 40/min., DO decreased from 400 ml to 250 ml. During auscultation, the breath is sharply weakened. Dullness of sound is observed in the patient?
Гостра серцева недостатністьAcute heart failure
Тромбоемболія легеневої артеріїThromboembolism of the pulmonary artery
Набряк мозкуBrain edema
ГідротораксHydrothorax
Набряк легенівPulmonary edema
Педіатрія
319/1500
У новонародженого терміном гестації 31 тиждень спостерігаються гіпотонія та пригнічення свідомості. Гематокрит - 35%, а в загальному аналізі ліквору вия-влено підвищену кількість еритроцитів, білка та знижений вміст глюкози. Ці дані відповідають клінічній картині:A newborn with a gestational age of 31 weeks has hypotonia and depression of consciousness. Hematocrit is 35%, and in the general analysis of the cerebrospinal fluid, an increased number of erythrocytes, protein and a decreased content of glucose. These data correspond to the clinical picture:
Жінка 53-х років, звернулася до лікаря зі скаргами на розпираючий біль в м'язах лівої гомілки та набряк, що з'явилися тиждень тому. Раніше подібного стану не виникало. Температура тіла - 37,6^oC. При огляді гомілка в середній третині збільшена в об'ємі на 5 см. Шкіра гомілки та ступні синюшна, блискуча, відмічається місцева гіпертермія. При пальпації гомілки - різка болючість. Позитивні симптоми Хоманса та Мозеса. Пульс на артеріях стоп задовільний. Який діагноз є найбільш імовірним?A 53-year-old woman turned to the doctor with complaints of excruciating pain in the muscles of the left leg and swelling that appeared a week ago. Previously, a similar condition did not occur. Body temperature - 37.6°C. When examining the lower leg, the volume is increased by 5 cm. The skin of the lower leg and foot is bluish. When palpating the lower leg, there is a sharp pain Moses. The pulse on the arteries of the feet is satisfactory. What is the most likely diagnosis?
Гострий тромбоз підколінної вениAcute thrombosis of popliteal vein
Гострий тромбоз підколінної артеріїAcute thrombosis of popliteal artery
Педіатрія
321/1500
Матір восьмимісячного хлопчика скаржиться на здуття живота у дитини, рясні, пінисті випорожнення 3-4 рази на добу з неприємним запахом, зниження маси тіла. Шкірні покриви дитини під час огляду бліді і сухі, визначається збільшення в об'ємі живота, гепатомегалія. У загальному аналізі крові анемія. У біохімічному дослідженні крові - гіпопротеїнемія, гіпоальбумінемія, зниження холестерину, загальних ліпідів і alpha-ліпопротеїдів. Встановіть діагноз за результатами дослідження.The mother of an eight-month-old boy complains of abdominal distension in the child, copious, foamy stools 3-4 times a day with an unpleasant odor, weight loss. The child's skin during the examination is pale and dry, an increase in the volume of the abdomen is determined. In the general blood analysis, hypoproteinemia, hypoalbuminemia, a decrease in total lipids and alpha-lipoproteins are established.
Виразковий ентеритUlcerative enteritis
Хвороба КронаCrohn's disease
Лактозна недостатністьLactose deficiency
Кишкова форма муковісцидозуIntestinal form of cystic fibrosis
ЦеліакіяCeliac
Терапія
322/1500
У чоловіка 35 років виникає задишка під час фізичного навантаження, він періодично втрачає свідомість. Аускультативно встановлено: систолічний шум у ІІІ міжребер'ї зліва від грудини. Під час УЗД серця встановлено: симетрична гіпертрофія ЛШ, його діастолічна дисфункція з деяким зменшенням порожнини. Спостерігається передньо-систолічний рух передньої стулки мітрального клапану. Яка патологія у хворого?A 35-year-old man develops shortness of breath during physical exertion, he periodically loses consciousness. Auscultation revealed: systolic murmur in the third intercostal space to the left of the sternum. During ultrasound of the heart: symmetric LV hypertrophy, its diastolic dysfunction with some reduction of the cavity. Ante-systolic movement of the anterior leaflet of the mitral valve is observed. What is the patient's pathology?
Дівчина 22 років скаржиться на дратівливість, плаксивість, головний біль, нудоту, іноді блювоту, серцебиття, нагрубання молочних залоз, набряки на руках і ногах, метеоризм. Скарги виникають за 6-10 днів до менструації та зникають із початком місячних. Бімануально встановлено: матка та додатки без змін. Який імовірний діагноз?A 22-year-old girl complains of irritability, tearfulness, headache, nausea, sometimes vomiting, palpitations, swelling of the mammary glands, swelling of the arms and legs, flatulence. Complaints occur 6-10 days before menstruation and disappear with the onset of menstruation. Bimanually established: the uterus and appendages are unchanged. What is the probable diagnosis?
Потерпілий, робочий хімзаводу, пра-цюючи з агресивною рідиною, по необережності вилив її собі на спецодяг, отримавши хімічний опік правого стегна та гомілки. Після зняття одягу виявлено: на передньо-внутрішній поверхні правого стегна, передній поверхні правої гомілки з переходом на тил стопи ділянки жовтаво-сірого кольору, місцями обривки епідермісу. Тактильна та больова гіпестезія уражених ділянок шкіри. Який з наступних кроків невідкладної допомоги має бути виконаний першочергово?The victim, a worker at a chemical plant, while working with an aggressive liquid, carelessly spilled it on his work clothes, receiving a chemical burn on his right thigh and lower leg. After removing the clothes, it was found: on the front-inner surface of the right thigh, with the transition to the back of the foot, a patch of epidermis. Tactile and painful hypoesthesia of the affected skin. Which of the following steps of emergency care should be performed as a priority?
Накладання асептичної пов'язкиApplying an aseptic bandage
Обробка етиловим спиртомEthyl alcohol treatment
Промивання проточною водоюRinsing with running water
Хлопчик віком 10 років лікувався в кардіологічному відділенні з приводу ревматизму, І атака, активна фаза, активність ІІ ступеня. Виписаний у задовільному стані. Який препарат найдоцільніше призначити для профілактики вторинного ревматизму?A 10-year-old boy was treated in the cardiology department for rheumatism, 1st attack, active phase, 2nd degree activity. He was discharged in a satisfactory condition. What drug is most appropriate to prescribe for prevention secondary rheumatism?
Біцилін-5Bicillin-5
ЕритроміцинErythromycin
АмпіцилінAmpicillin
ОксацилінOxacillin
Біцилін-1Bicilin-1
Терапія
326/1500
У жінки віком 35 років за 30 хвилин після внутрішньо-м'язової ін'єкції ампіциліну виникли різка слабкість, свербіж обличчя та рук, нудота, кашель, задишка, біль у грудній клітці. Об'єктивно спостерігається: ціаноз, набряк повік, обличчя, шиї, шкіра волога з червоним висипом. Пульс - 120/хв, АТ - 70/20 мм рт. ст., тони серця глухі, дихання часте, поверхневе з великою кількістю різнокаліберних вологих хрипів. З введення якого препарату необхідно негайно розпочати терапію?A 35-year-old woman developed sudden weakness, itching of the face and hands, nausea, cough, shortness of breath, 30 minutes after an intramuscular injection of ampicillin pain in the chest. Objectively observed: cyanosis, swelling of the face, neck, skin with a red rash. Pulse - 70/20 mm Hg, heart sounds are shallow with a large number of wet rales. With the introduction of which drug should the therapy be started immediately?
ЕуфілінуEuphilina
АстмопентуAstmopentu
ДопамінуDopamine
АдреналінуAdrenaline
ПреднізолонуPrednisolone
Терапія
327/1500
Хвора 48-ми років доставлена в лікарню з неадекватною поведінкою. Зі слів чоловіка, постійно вживала знеболювальні препарати у зв'язку з мігренню та болем у спині. Об'єктивно: жовтяничність склер, чутливість у правому верхньому квадранті живота та астериксис. БАК: АЛТ- 649 Од/л, загальний білірубін - 95,8 мкмоль/л та МНС- 6,8. Який найбільш імовірний діа-гноз?A 48-year-old patient was brought to the hospital with inappropriate behavior. According to her husband, she was constantly taking painkillers due to migraines and back pain. About' objectively: sclera, sensitivity in the right upper quadrant and asterixis. BAC - 649 U/l, total bilirubin - 95.8 μmol/l. What is the most likely diagnosis?
Гостра печінкова недостатністьAcute liver failure
Інтоксикація знеболювальними засобамиIntoxication with painkillers
Гостре порушення мозкового кровообігуAcute cerebrovascular accident
Акушерство і гінекологія
328/1500
Жiнка віком 25 рокiв була госпіталізована в терміні вагітності 11 тижнів зі скаргами на біль внизу живота. З анамнезу відомо, що 2 тижнi тому вона перехворiла на краснуху в тяжкiй формi. Під час бiмануального дослiдження виявлено: шийка матки сформована, вiчко шийки матки закрите, матка збільшена до 11 тижнів вагiтності, придатки матки без особливостей. Яка лікарська тактика є найдоцiльнішою в цій ситуації?A 25-year-old woman was hospitalized at 11 weeks of pregnancy with complaints of pain in the lower abdomen. It is known from the anamnesis that 2 weeks ago she was ill with rubella in a severe form During the bimanual examination, it was found that the cervix is formed, the cervix is closed, the uterus is enlarged to 11 weeks of pregnancy, and the appendages are unremarkable.
Введення утеротонікiвIntroduction of uterotonics
Малий кесарiв розтинSmall caesarean section
Введення спазмолiтикiвIntroduction of antispasmodics
Переривання вагітностіTermination of pregnancy
Пролонгування вагiтностiProlongation of pregnancy
Терапія
329/1500
Чоловік 27-ми років звернувся до лікаря зі скаргами на біль та набряк у II та III пальцях лівої стопи, почервоніння та гнійні виділення з очей, різь та біль при виділенні сечі. Зі слів пацієнта має декількох статевих партнерів, зрідка користується презервативами. При фізикальному обстеженні лікарем виявлено двосторонній кон'юнктивіт та дактиліт пальців стопи. Обстеження інших органів і систем патології не виявило. У загальному аналізі крові ШЗЕ- 40 мм/год, біохімічний аналіз без відхилень. На рентгенограмах пальців стопи збільшення об'єму м'яких тканин, звуження суглобової щілини. Який діагноз є найбільш імовірним?A 27-year-old man consulted a doctor with complaints of pain and swelling in the II and III toes of the left foot, redness and purulent discharge from the eyes, tearing and pain in according to the patient, he occasionally uses condoms. During the physical examination, the doctor revealed dactylitis of the feet. In the general analysis of the blood, it was 40 mm/h analysis without deviations. On X-rays of the toes, an increase in the volume of soft tissues, narrowing of the joint space. What is the most likely diagnosis?
Реактивний артритReactive arthritis
СифілісSyphilis
ПодаграGout
Синдром ШегренаSjogren's syndrome
Ревматоїдний артритRheumatoid arthritis
Терапія
330/1500
Провідник потягу 39 років госпіталізований на 4-й день хвороби зі скаргами на головний біль, слабкість, запаморочення, пітливість, безсоння, лихоманку. Обличчя гіперемійоване, набрякле, кон'юнктивіт. На перехідній складці кон'юнктиви - поодинокі петехії. На шкірі тулуба, грудної клітки, живота, кінцівок - рясна розеольозно-петехіальна висипка. Тахікардія. АТ - 100/60 мм рт. ст. Спостерігається тремор язика. Пальпується печінка, селезінка. Випорожнення затримане. Який найімовірніший діагноз?A 39-year-old train conductor was hospitalized on the 4th day of illness with complaints of headache, weakness, dizziness, sweating, insomnia, fever. The face is hyperemic, swollen, con Conjunctivitis - single petechiae on the trunk, chest, extremities. Tachycardia - 100/60 mmHg. Palpable liver . Defecation is delayed. What is the most likely diagnosis?
Висипний тифTyphoid
ЛептоспірозLeptospirosis
Черевний тифTyphoid
ГрипFlu
МенінгококцеміяMeningococcemia
Педіатрія
331/1500
Дитина віком 5 років захворіла гостро, спостерігаються підвищення температури тіла до 39,2^oC, одноразове блювання, скарги на переймоподібний біль у животі, тенезми, часті випорожнення: малокалові, з великою кількістю слизу, домішками гною та прожилок крові. Під час огляду виявлено: ущільнена сигмоподібна кишка, болюча під час пальпації. Встановіть діагноз.A 5-year-old child became acutely ill, there was an increase in body temperature to 39.2^oC, one-time vomiting, complaints of spasmodic abdominal pain, tenesmus, frequent bowel movements: scanty, with a large amount of mucus, pus and streaks of blood. During the examination, it was found that the sigmoid colon was painful during palpation.
ХолераCholera
ШигельозShigelosis
Ротавірусна інфекціяRotavirus infection
Гострий апендицитAcute appendicitis
СальмонельозSalmonellosis
Педіатрія
332/1500
Дитина 2 років була в контакті з хворим на оперізуючий герпес. На 15-й день контакту в неї підвищилась температура тіла до 39,3^oC, з'явився висип на тулубі, кінцівках, обличчі та волосистій частині голови, який проходив стадії: пляма, папула, визикула, кірочка. Який імовірний діагноз?A 2-year-old child was in contact with a patient with herpes zoster. On the 15th day of contact, her body temperature rose to 39.3^oC, appeared a rash on the trunk, limbs, face and scalp, which went through the stages: spot, papule, vesicle, crust. What is the probable diagnosis?
Ентеровірусна екзантемаEnterovirus exanthema
Вітряна віспаChicken Pox
Простий герпесHerpes simplex
Натуральна віспаSmallpox
Оперізуючий герпесHerpes zoster
Терапія
333/1500
Чоловік 45 років скаржиться на втрату апетиту, запаморочення, виражену загальну слабкість, субфебрильну температуру. Шкіра бліда, синці на тулубі та кінцівках. ЗАК виявив: еритроцити - 2,9·10^12/л, Нb - 96 г/л, КП - 0,99, ретикулоцити - 0,02%, лейкоцити - 2,9·10^9/л, тромбоцити - 85·10^9/л, ШОЕ - 45 мм/год. Яке додаткове дослідження є найбільш доцільним для верифікації діагнозу?A 45-year-old man complains of loss of appetite, dizziness, pronounced general weakness, subfebrile temperature. The skin is pale, bruises on the trunk and limbs. ZAK revealed: erythrocytes - 2, 9·10^12/l, Hb - 96 g/l, CP - 0.99, reticulocytes - 0.02%, leukocytes - 2.9·10^9/l, platelets - 85·10^9/l, ESR - 45 mm/h. What additional research is most appropriate for diagnosis verification?
Стернальна пункціяSternal puncture
Рівень сироваткового залізаSerum iron level
КоагулограмаCoagulogram
Осмотична резистентність еритроцитівOsmotic resistance of erythrocytes
Вміст В_12 в кровіB_12 content in blood
Терапія
334/1500
Пацiєнтка віком 45 рокiв скаржиться на дискомфорт пiд час читання, почервонiння краю повiк, бiлi пiнистi видiлення в кутах очних щiлин протягом останнiх двох мiсяцiв. Об'єктивно спостерiгається: гiперемiя та потовщення вiльного краю повiк, розширення вивiдних протокiв залоз хряща повiк. Який найімовірніший діагноз?A 45-year-old patient complains of discomfort while reading, reddening of the edge of the eyelids, white foamy discharge in the corners of the eye slits for the past two months. Objectively observed: hyperemia and thickening of the free edge of the eyelids, expansion of the excretory ducts of the eyelid cartilage. What is the most likely diagnosis?
Пацієнт 72 років надійшов до відділення судинної хірургії зі скаргами на біль та мерзлякуватість нижніх кінцівок. УЗД артерій нижніх кінцівок виявило атеросклеротичне ураження судинної стінки. Який метод діагностики потрібний для уточнення локалізації та протяжності процесу?A 72-year-old patient came to the department of vascular surgery with complaints of pain and numbness of the lower extremities. Ultrasound of the arteries of the lower extremities revealed atherosclerotic lesions of the vascular wall. What diagnostic method is needed to clarify localization and duration of the process?
ЕКГECG
Рентгенографія кінцівкиExtremity X-ray
АнгіографіяAngiography
ТермометріяThermometry
Рентгенографія органів грудної порожниниX-ray of chest cavity
Педіатрія
336/1500
Хворий 26-ти років звернувся до лікаря зі скаргами на озноб, нежить, сухий кашель, лихоманку до 38^oC. В ході огляду: стан середньої тяжкості, на шкірі спини, живота, та кінцівок блідо-рожеві дрібні незливні плями. Пальпуються збільшені потиличні, аксилярні лімфовузли. Відомостей про щеплення немає. Яка ймовірна етіологія даного захворювання?A 26-year-old patient turned to the doctor with complaints of chills, runny nose, dry cough, fever up to 38°C. During the examination: a condition of moderate severity, on the skin of the back, abdomen, and limbs are pale pink. There are enlarged occipital and axillary lymph nodes. There is no information about the etiology of this disease.
МенінгококMeningococcus
Вірус Епштейна-БаррEpstein-Barr virus
СтрептококStreptococcus
Вірус краснухиRuellas virus
Вірус епідемічного паротитуEpidemic mumps virus
Не класифіковано
337/1500
Чоловік 30 років скаржиться на висип та свербіння шкіри стоп. Хворіє 3 роки. Об'єктивно в ділянці шкіри підошов спостерігаються групи пухирців, схожих на розварені сагові зерна, а також ерозії з уривками мацерованого епідермісу на периферії вогнищ. У міжпальцевих складках обох стоп видно тріщини, ерозії. Яка патологія найімовірніша?A 30-year-old man complains of a rash and itching of the skin of his feet. He has been ill for 3 years. Objectively, in the area of the skin of the soles, groups of blisters similar to boiled sago grains are observed, and also erosions with fragments of macerated epidermis on the periphery of the foci. Cracks, erosions are visible in the interdigital folds of both feet. What is the most likely pathology?
Вторинний сифілісSecondary syphilis
Епідермофітія стопEpidermophyta of the feet
Руброфітія стопRubrophytia of the feet
ДерматитDermatitis
ПсоріазPsoriasis
Акушерство і гінекологія
338/1500
Жінка 22-х років звернулася до лікаря з метою планування вагітності. В анамнезі цукровий діабет протягом двох років. Діа-бетичних ускладнень при опитуванні та огляді не виявлено. Глікемія натще в межах 6,0-7,0 ммоль/л. Який аналіз серед перерахованих слід призначити, щоб адекватно оцінити контроль глікемії?A 22-year-old woman consulted a doctor to plan a pregnancy. She has a history of diabetes for two years. Diabetic complications were not detected during the survey and examination. Glycemia fasting in the range of 6.0-7.0 mmol/l. Which analysis among the listed should be prescribed to adequately assess glycemic control?
Глюкозуричний профільGlucosuric profile
Глікемічний профільGlycemic Profile
Рівень С-пептидуC-peptide level
Рівень глікозильованого (глікованого) гемоглобіну (НbА1c)The level of glycosylated (glycated) hemoglobin (HbA1c)
Постпрандіальна глікеміяPostprandial glycemia
Терапія
339/1500
Під час проходження допризивної комісії у хлопця 17 років виявлена артеріальна гіпертензія - АТ на руках 190/110 мм рт. ст. Скарг немає. Звертає на себе увагу непропорційна будова тіла - добре розвинений плечовий пояс і недорозвинені нижні кінцівки. Який метод дослідження, найвірогідніше, найбільш інфор-мативний для постановки діагнозу?During the pre-conscription commission, a 17-year-old boy was diagnosed with hypertension - blood pressure in the arms of 190/110 mm Hg. There are no complaints. The disproportionate structure attracts attention bodies - a well-developed shoulder girdle and underdeveloped lower limbs. Which research method is most likely to be the most informative for making a diagnosis?
Доплер-сонографія судинDoppler sonography of vessels
Вимірювання АТ на нижніх кінцівкахBP measurement on lower limbs
РеносцинтіграфіяRenoscintigraphy
УЗД нирок і наднирниківUltrasound of kidneys and adrenal glands
Визначення катехоламінів у сечіDetermination of catecholamines in urine
Терапія
340/1500
Жінка 36 років скаржиться на біль у надчеревній ділянці, що виникає після вживання їжі, нудоту, відрижку, нестійкість випорожнень. Хвороба поступово прогресує впродовж 2 років. Об'єктивно спостерігаються блідість і сухість шкіри, язик обкладений і вологий з відбитками зубів по краях. Під час пальпації живота спостерігається розлитий біль у надчеревній ділянці. Який найінформативніший метод дослідження слід застосувати в цьому разі?A 36-year-old woman complains of pain in the epigastric region, which occurs after eating, nausea, belching, stool instability. The disease gradually progresses over 2 years. Objectively paleness and dryness of the skin are observed, the tongue is coated and wet with imprints of the teeth. During palpation of the abdomen, diffuse pain is observed in the epigastric region. What is the most informative method of examination in this case?
Фракційне дослідження шлункової секреціїFractional study of gastric secretion
Розгорнутий клінічний аналіз кровіExpanded clinical blood analysis
Комп'ютерне дослідження черевної порожниниComputer examination of the abdominal cavity
Рентгенологічне дослідження шлунка і кишечникаX-ray examination of the stomach and intestines
Фіброгастроскопія з біопсією слизової шлункаFibrogastroscopy with gastric mucosa biopsy
Терапія
341/1500
Хвора 45-ти років звернулася зі скаргами на прогресуюче збільшення маси тіла за останній рік. При огляді: місяцеподібне обличчя, ламке волосся, гірсутизм, на животі смуги розтягу, кінцівки непропорційно тонкі. Зріст - 162 см, маса тіла - 94 кг, ІМТ - 35,8 кг/м^2. Який тип ожиріння притаманний для хворої?A 45-year-old patient complained of a progressive increase in body weight over the past year. On examination: a moon-shaped face, brittle hair, hirsutism, stretch marks on the abdomen, limbs are disproportionately thin. Height - 162 cm, body weight - 94 kg, BMI - 35.8 kg/m^2. What type of obesity is characteristic of the patient?
АндроїднеAndroid
Аліментарно-конституційнеAlimentary and constitutional